Reasoning Answer

You might also like

Download as pdf or txt
Download as pdf or txt
You are on page 1of 132

P

Numerical Ability

UNIT 1  Quantitative Aptitude 1.21

A
UNIT 2  Reasoning1.51

R
t
B
Unit I_Chapter 1.indd 19 27/04/2017 15:27:22
Unit I_Chapter 1.indd 20 27/04/2017 15:27:22
U
Quantitative Aptitude

  Chapter 1 
Simple Equations 1.23

n
  Chapter 2  Ratio–Proportion–Variation1.31

  Chapter 3 Numbers 1.39

  Chapter 4  Percentage, Profit, and Loss 1.52

  Chapter 5 
Simple Interest and
Compound Interest 1.62

i
  Chapter 6 
Averages, Mixtures,
and Alligations 1.68

  Chapter 7  Time and Work 1.76

  Chapter 8  Time and Distance 1.84

  Chapter 9 
Indices, Surds, and

t
Logarithms1.95

Chapter 10  Quadratic Equations 1.103

Chapter 11  Inequalities1.110

Chapter 12  Progressions1.117

Chapter 13 
Permutations and

1
Combinations1.125

Chapter 14  Data Interpretation 1.131

Unit I_Chapter 1.indd 21 27/04/2017 15:27:22


Unit I_Chapter 1.indd 22 27/04/2017 14:16:35
Chapter 1 Simple Equations
Hints/Solutions
Practice Problems I b = 4 and a = 5
\ Product of digits = 20
1. Given that P, Q and R are successive even natural num-
bers in ascending order. As we need Q, we express P Hence, the correct option is (B).
and R in terms of Q. 7. Let the present ages of Ram and Shyam be r years and
P = Q - 2 and R = Q + 2 s years respectively.
\ 5(Q + 2) = 7(Q - 2) + 8 ⇒ Q = 8  r - 6 = 4(s - 6) ⇒ r = 4s - 18
Hence, the correct option is (B). And   r + 6 = 3(s + 6) ⇒ r = 3s + 12
2. Let the two parts be x grams and y grams \ 4s - 18 = 3s + 12 ⇒ s = 30, r = 102
x + y = 1000 \ their present combined age is 132 years. It has to
5 increase by 18 years to become 150 years. This will
x + y = ( x − y ) happen if the age of each increases by 9 years.
4
x – y = 800 ⇒ x = 900 Hence, the correct option is (B).
 y = 100 8. Let the age of Bharat be A and Sharat be B.
Hence, the correct option is (C). A + B = 2 (A + B - 14) ⇒ A + B = 28
A+ B  A + B2 = 400
2

3. = A− B (A + B)2 - 2AB = 400


3
3A - 3B = A + B \ AB = 192
2A - 4B = 0 Hence, the correct option is (A).
A - 2B = 0 (32) 9. Let the number of `20 notes with Alok be x. Number of
B + 40 = 2A + 10 `5 notes with him = 30 - x. Given that,

2A - B = 30 (33) 20x + 5(30 - x) = 300 ⇒ x = 10


⇒ 4A - 2B = 60 Hence, the correct option is (B).
3A = 60 x
10. Let the fraction be
⇒ A = 20 and B = 10 y
x=y-5  (37)
Hence, the correct option is (C).
and 4x = y + 1  (38)
4. Let the number be x.
Solving (37) and (38)
9 2
x − x = 4235 ⇒ x = 990 x = 2, y = 7
2 9 2
\ the fraction is
Hence, the correct option is (C). 7
5. Let the shares of A, B and C be A, B and C respectively. Hence, the correct option is (C).
then, A + B + C = 5600 (34) 11. The difference between any two digit number and the
B + C = 3A  (35) number formed by reversing its digits is equal to 9
times the difference of its digits
A + C = 9/5B  (36)
\ the required difference = 9 (3) = 27
Solving (34) and (35)
Hence, the correct option is (B).
4A = 5600 ⇒ A = `1400
12. Let the costs of each chocolate, each milk shake and
Solving (34) and (36)
each cake be `ch `m and `c respectively.
14 B
= 5600 ⇒ B = `2000 2ch + 3m + 4c = 190
5 4ch + 8c = 320
Share of C = 5600 - 1400 - 2000 = `2200
The coefficients of ch and c are proportional, we can
Hence, the correct option is (C). determine m, but not ch or c.
6. Let the number be ‘ab’. 2ch + 4c = 160
4(a + b) = 10a + b - 18 ⇒ b + 6 = 2a \ 3m = 190 - (2ch + 4c) = 30
Also, 10a + b - 18 = 10b + a - 9 ⇒ a - b = 1 \ m = 10.
\ b + 6 = 2(b + 1) Hence, the correct option is (A).

Unit I_Chapter 1.indd 23 27/04/2017 14:16:36


1.24  |  Unit 1  •  Quantitative Aptitude

13. Let the three consecutive numbers be 2x, 2(x + 1), 19. Cost of 1 pen = x
2(x + 2) Cost of 1 pencil = y
1 1
then × 3 (x + 1) = × 2(x + 2) ⇒ x = 2 Cost of 1 eraser = z
3 4
2x + 5y + 7z = 37 (39)
\ the three numbers are 4, 6, 8
7x + 2y + z = 49 (40)
Hence, the correct option is (A).
Multiplying equation (40) by 7 and subtracting equa-
14. Let the amount with Amar be `a tion (39) gives the required value.
Total amount with the others = `(150 - a) ⇒ 49x + 14y + 7z = 343
1
a = (150 − a ) ⇒ a = 30   2x + 5y + 7z = 37
4 --------------------------
Hence, the correct option is (C). 47x + 9y = 306
15. Let the present ages of Ramesh and Suresh be r years --------------------------
and S years respectively Hence, the correct option is (D).
r = 3s 20. Let the larger number be a.
r + 2 = 2 (s + 2) Smaller number = 250 - a
⇒ r = 2s + 2 = 3s a2 - (250 - a)2 = 12500
s = 2   r = 6 (a + 250 - a) (a - (250 - a)) = 12500
\ Ramesh’s present age is 6 years. ⇒ 2a - 250 = 50; \ a = 150
Hence, the correct option is (D). Hence, the correct option is (C).
16. Let the number of 50 np coins with Nalini be x. 21. Let the number N  be ‘abc’.
Number of `1 coins with her will be (30 - x) Let the sum of the remaining numbers be R.
Amount = x × 50 + (30 - x) 100 = 2000 R + cba = R + abc + 11(a + b + c)
⇒ x + 60 - 2x = 40 ⇒ x = 20 ⇒ 99(c - a) = 11(a + b + c)
\ number of `1 coins with her is (30 - 20) = 10 ⇒ 8c - 10a = b (41)
Hence, the correct option is (B). 5
As b ≥ 0 and c ≥ a , c > a
17. Let the number be ‘ab’ i.e. or 10a + b. 4
Given that 10a + b = 6(a + b) + 1 ⇒ 4a = 5b + 1 \ 8 (Difference of a and c) = 8(c - a) = 6 + 2a
b +1 8c - 10a = 6 (42)
a = b +
4 From (41) and (42), b = 6
As a is a natural number, b + 1 must be divisible by 4. Hence, the correct option is (B).
0 ≤ b ≤ 9. 22. Let the cost of each pen, each eraser and each sharp-
\ b can be only 3 or 7 and correspondingly a = 4 or 9 ener be `p, `e and `s respectively.
\ ab = 43 or 97 2p + e + 3s = 23  (43)
The second condition is satisfied only if ‘ab’ = 97. 6p + 3e + s = 45 (44)
\ ‘ab’ = 97 14p + 7e + 21s = 161 (45)
Hence, the correct option is (B). But (43), (45) are equivalent.
18. As the system has infinite solutions, the coefficients are \ We effectively have only 2 equations (43) and (44)
proportional or (44) and (45).
k In order to find the cost of each pen, the ratio of the
+2
3 3 ⎛k ⎞ coefficients of erasers and sharpeners must be the
i.e., = ; k ⎜ −4⎟=0
k 2k ⎝3 ⎠ same. As this is not the case, Choice (D) follows.
k = 0 or 12. If k = 0, the second equation becomes Hence, the correct option is (D).
inconsistent. 23. If cost of a pencil is `5 then the only feasible values for
\ k = 12 cost of a pen and cost of a ruler are 6 and 4 respectively.
Hence, the correct option is (D).

Unit I_Chapter 1.indd 24 27/04/2017 14:16:37


Chapter 1  •  Simple Equations  |  1.25

\ Cost of 10 rulers = 40 i.e. 2n - 2s + 4 = 0


\ Number of pencils he purchased with `40 is 2n - s - 3 = 0
40 -------------------------
= =8
5 ⇒  s = 7, n = 5
Hence, the correct option is (C). Hence, the correct option is (B).
24. Let the number of marbles with Nitya and Satya be n 25. Let the speeds of John and Peter be x and y respectively
and s respectively. (in kilometres per hour)
n+1=s-1 ⇒ x = y + 10  (48)
⇒ n-s+2=0 (46) x + 15 = 2y  (49)
s + 1 = 2(n - 1) y = 25; x = 35
⇒ 2n - s - 3 = 0 (47) \ John’s speed is 35 km/hr
Solving (46) and (47) Hence, the correct option is (D).

Practice Problems 2
Solutions for questions 1 to 5: 2. The sum of two numbers is 4 and the reciprocal of one
1. In a four-digit number, the sum of the middle two digits exceeds the reciprocal of the other by twice the product
is twice the units digit. The sum of the hundreds digit of their reciprocals. What is the product of the recipro-
and six times the thousands digit is twice the sum of cals of the two numbers?
the other two digits. The sum of the units digit and five (A) 3 (B) 2 (C) 1/3 (D) 1/2
times the thousands digit is twice the hundreds digit. 1 1
How many values can the four-digit number assume? Sol. Let the two numbers be and
x y
(A) 1 (B) 2 (C) 3 (D) 4 1 1
+ =4  (54)
Sol. Let the number be ‘abcd’ x y
Given that, and x - y = 2xy
b + c = 2d(50) 1 1
⇒ − = 2 (55)
b + 6a = 2(c + d)(51) y x
d + 5a = 2b (52) 2 1
(54) + (55) gives = 6 ⇒ y =
Let us that the equations (50), (51) and (52) as the y 3
­linear equations in a, b and c and express the values of (54) - (55) gives
a, b and c in terms of b. 2
c = 2 ⇒ x = 1
By (51) - (50), we get 3c = 6a ⇒ a = x
2
By substituting c = 2a in (50), it becomes \ Product of the reciprocals = (x) (y) = (1) (1/3) = (1/3)
2d - 2a = b (53) Hence, the correct option is (C).
Subtracting (53) from 2 × (52), we get 3. Ashok went to a casino to play a card game. In each round,
b he happened to double the amount with himself and in
12a = 3b ⇒ a = each round he gave `x to his friend. At the end of three
4
rounds, he was left with no money. If the amount he gave
b
As c = 2a, c = to his friend in each round was `20 more than the amount
2 he started with, find the amount (in `) that he started with.
b (A) 110 (B) 120 (C) 130 (D) 140
By substituting a = in (53), we get
4
Sol. Suppose Alok started with `A. He gave `x to his friend
5 in each round. The amounts with him at the end of the
2d = x + y = ( x − y )
4 first, second and third rounds are (in `)
3b
⇒ d = 2A - x, 4A - 3x and 8A - 7x respectively.
4
Given that, 8A - 7x = 0
\ a : b : c : d = 1 : 4 : 2 : 3
\ ‘abcd’ can be 1423 or 2846. 8
⇒ x = A
Hence, the correct option is (B). 7

Unit I_Chapter 1.indd 25 27/04/2017 14:16:38


1.26  |  Unit 1  •  Quantitative Aptitude

As x = A + 20,
8 p + e + s = 38
A = A + 20
7 p < e < s
⇒ A = 140 \ ( p, e, s) can be only (11, 12, 15) or (11, 13, 14).
Hence, the correct option is (D). 6. How many pens did he purchase?
4. The cost of printing the first fifteen hundred copies of (A) 11 (B) 12 (C) 13 (D) 14
a book, is `1500. It costs y rupees to print each subse- Sol. In either of the above combinations p = 11
quent copy. The cost of printing the first 7500 copies of
the book is `7080. Find y. Hence, the correct option is (A).
(A) 0.91 (B) 0.89 (C) 0.93 (D) 0.87 7. If each pen cost `2, each eraser cost `3, and each sharp-
Sol. Number of sequence copies made = 7500 - 1500 = ener cost `4, find the minimum expenditure be could
6000. have incurred on the items (in `).
Cost of making these copies (in `) = 7080 - 1500 = (A) 116 (B) 118 (C) 117 (D) 119
5580. Sol. ( p, e, s) = (11, 13, 14) results in the minimum
5580 expenditure.
y= = 0.93
6000 Minimum expenditure = ` 117.

Hence, the correct option is (C). Hence, the correct option is (C).
5. Mohan and Sohan went to a bakery shop. Mohan Solutions for questions 8 to 15:
bought six puffs, four burgers, and three cakes and used 8. A test had 200 questions. Each correct answer carried
up all the money he had. Sohan bought three puffs, 2 marks. Each wrong answer carried -1/2 mark and
eight burgers, and six cakes and spent 20% more than unanswered questions carried no mark. Ajay attempted
Mohan. What percent of his money did Mohan spend all the questions in the test and scored 360 marks.
on puffs? What would his marks be, if for each correct answer
1 3 he got only 1/2 mark and for each wrong answer he lost
(A) 53 % (B) 58 %
3 4 2 marks?
4 1 (A) 80 (B) 60 (C) 40 (D) 100
(C) 59 % (D) 57 %
9 7 Sol. Let the number of questions correctly answered by
Sol. Let the costs of each puff, each burger and each cake Ajay be x. Number of questions wrongly answered by
be `p, `b and `c respectively. Let the amount spent by him = 200 - x.
1 5
Mohan be `x. His mark = 2x - (200 - x) = x −100 = 360
2 2
6p + 4b + 3c = x (56) ⇒ x = 184
6x Upon interchanging of the marks, his marks would be
3p + 8b + 6c = (57)
5  = 184(1/2) - 16(2) = 60
Multiplying (56) by 2 and subtracting (57) from it, Hence, the correct option is (B).
4x
9p = 9. Anand bought a total of 30 white and brown pens for
5 a total of `32. The cost of each white pen is 70 paise
4x more than the cost of each brown pen. Which of the
⇒ p =
45 following represents a possible value of the cost of each
brown pen (in paise)?
8
\ 6p = x (A) 40 (B) 35 (C) 45 (D) 60
15
Sol. Let the cost of each brown pen be b paise, cost of each
1 white pen = (b + 70) paise. Suppose Anand bought B
\ Mohan spent 53 % of his money on puffs. brown pens. He must have bought (30 - B) white pens.
3
Hence, the correct option is (A). Total cost = bB + (30 - B) (b + 70)
Solutions for questions 6 to 7: 30b - 70B + 2100 = 3200
3b − 110
Rakesh went to a stationery shop to purchase a total of B= . Among the given choices, B has a feasi-
38 pens, erasers, and sharpeners. He purchased at least 7
11 items of each. He purchased more sharpeners than ble value only when b = 60.
­erasers and more erasers than pens. Hence, the correct option is (D).
Let the number of pens, erasers and sharpeners that Rakesh 10. Runs scored by Bhangar in a match are 10 more than
purchased be p, e and s respectively. the balls faced by Karthik. The number of balls faced

Unit I_Chapter 1.indd 26 27/04/2017 14:16:40


Chapter 1  •  Simple Equations  |  1.27

by Bhangar is 5 more than the number of runs scored Sol. Let the initial amount with Gopi, Murthy and Hari in
by Karthik. Together they have scored 50 runs and rupees be x, y and z respectively.
Bhangar has faced 15 balls less than Karthik. What is x
the number of runs scored by Bhangar? Gopi gives to Murthy
2
(A) 30 (B) 20 (C) 40 (D) 50
x x
Sol. Data is tabulated below They now have , + y and z
2 2
Bhangar Karthin
Murthy gives half of his amount to Hari
Runs x + 10y
They now have
Balls y + 5x
x x y x y
x + y + 10 = 50 ; + ; + + z
2 4 2 4 2
x + y = 40 (58)
Hari gives half his amount to Gopi.
y + 5 = x - 15
⎛x x y x z⎞
x - y = 20 (59) Gopi now has ⎜ + + + + ⎟ and this is equal
to x. ⎝ 2 8 4 4 2⎠
⎛ 40 + 20 ⎞
⇒ x = ⎜ ⎟ = 30
⎝ 2 ⎠ x x y z
Runs scored by Bhangar = 30 + 10 = 40 ⇒ x = + + +
2 8 4 2
Hence, the correct option is (C).
3x y z
11. An amount of `9000 is divided among four people A, ⇒ = +
8 4 2
B, C, and D. The sum of the shares of A, C, and D is
⇒ 3x = 2y + 4z = 2 x 45 = 90, as (y + 2z)
four times the share of B. The sum of the shares of B
and D is equal to four-fifths the sum of shares of A and is given equal to 45.
C. Find the share of D ⇒ x = 30
(A) `1,800 Hence, the correct option is (C).
(B) `2,400
(C) `2,200 13. A housewife, with a given amount, can buy either
(D) Cannot be determined 10 apples or 15 oranges or 2 watermelons. Find the
maximum number of oranges which she can buy with
Sol. A + B + C + D = 90000 (60)
six times the initial amount such that she gets each of
A + C + D = 4B (61) the three varieties of fruits.
also, B + D = 4/5 (A + C) (62) (A) 75 (B) 81
From (60) and (61) (C) 60 (D) 72
4B + B = 9000 Sol. Let the initial amount = A
⇒ B = `1800 Cost of 1 apple = (A/10)
From (61) and (62) Cost of 1 orange = (A/15)
B + D = 4/5 (4B - D) Cost of 1 watermelon = (A/2)
5B + 5D = 16B - 4D As all three types of fruits are bought, the minimum
shall be one.
⇒ 11B = 9D
As oranges are to be maximum, others are one each.
11
⇒ D = × 1800 = `2200 Amount spent = 6A (as per data).
9
Hence, the correct option is (C). Hence, the number of oranges
= [6A – {(A/10) + (A/2)}] / (A/15)
12. Gopi, Murthy, and Hari had some amount of money.
Gopi gives half his amount to Murthy, who then gives = [6A – (6A/10)] / (A/15) = (54A/10) / (A/15) = 81
half of what he now has to Hari. Hari gives half of what Hence, the correct option is (B).
he now has to Gopi, who, now has exactly what he 14. A bag has balls of four colours—red, blue, pink, and
started with. If the sum of Murthy’s initial amount and green. Half the total number of balls are pink. One-
twice Hari’s initial amount is `45, what was the amount fourth of the number of green balls equals one-third
(in rupees) Gopi started with? of the number of blue balls. The number of pink balls
(A) 10 (B) 15 is 4 less than twice the total number of green and blue
(C) 30 (D) 40 balls. The number of green balls is 22 less than the total

Unit I_Chapter 1.indd 27 27/04/2017 14:16:41


1.28  |  Unit 1  •  Quantitative Aptitude

number of blue and pink balls. Find the total number of Given that,
balls in the bag. 10a + b - (10b + a) = 81
(A) 48 (B) 36 (C) 40 (D) 54
a - b = 9
Sol. Let the number of balls of he colours red, blue, pink,
\ a = 9 and b = 0
green be r, b, p, g respectively.
16. Find the actual number of toys sold.
Half the total number of balls in the box are pink
(A) 92 (B) 81
\   p = b + r + g(63) (C) 90 (D) 29
g b 4b Sol. ab = 90
= ⇒∴g = (64)
4 3 3 Hence, the correct option is (C).
p = 2 (g + b) – 4 (65)
17. If the faulty calculations show a total sale of `882, find
g = b + p – 22 (66) the actual selling price of each toy (in `).
\ g = b + 2 (g + b) – 4 – 22 (A) 89 (B) 98
4b 13b (C) 97 (D) 79
⇒ 26 = 3b + g = 3b + =
3 3 882 882
Sol. Mistaken sale price = = = 98 = `98
b = 6 ’ba ’ 09
\ g = 8, p = 24, r = 10 Actual sale price = `89
Total number of balls = b + g + p + r = 2p = 48 Hence, the correct option is (A).
Hence, the correct option is (A). Solutions for questions 18 to 25:
15. The cost of four apples, six bananas, and eight oranges 18. A three-digit number is eleven times the two-digit num-
is p. The cost of five apples, eight bananas, and eleven ber formed by using the hundreds and the units digit
oranges is q. The cost of eight apples, fourteen bananas, of the three-digit number, respectively, in the tens and
and twenty oranges is r. Which of the following is units place of the two-digit number. If the difference
always true? between the digit in tens place and the digit in hundreds
3r r place is 1, then what is the digit in the units place?
(A) 3p + 4q = (B) –3p + 4q =
4 2 (A) 2 (B) 3 (C) 4 (D) 1
r Sol. Let the hundreds, tens and units digits be x, y, z
(C) –4q + 3p = (D) –3p + 4q = r
2 respectively.
Sol. Let the costs of an apple, a banana and an orange be (100x + 10y + z) is the 3-digit numbers = 11(10x + z)
`a, `b and `c respectively.
= (100x + z) + (10x + 10z)
4a + 6b + 8c = p \ y = x + z
5a + 8b + 11c = q Given y - x = 1
8a + 16b + 20c = r \ z = 1
Going back from the options, only Choice (D) satisfy Note: If (x – y) is taken as 1, z = –1, which is
the given equation. inadmissible.
Hence, the correct option is (D). Hence, the correct option is (D).
Solutions for questions 16 and 17: 19. A person on an exercise regime decides to lose
A shopkeeper sold a certain number of toys. The number of 500 ­calories in a day. For every 30 minutes of exercise,
toys as well as the price of each toy (in `) was a two digit he loses 80 calories, but gains 25 calories per cake that
number. By mistake, he reversed the digits of both the num- he eats. In the morning, he exercises for a certain time
ber of toys he sold and the price of each toy. As a result, he and he eats a certain number of cakes. In the afternoon,
found that his stock account at the end of the day showed he exercises for double the time but eats thrice the num-
81 items more than it actually was. ber of cakes. In the evening, he exercises for 1.5 times
Let the number of toys sold by the shopkeeper be ‘ab’. the time he spends in the morning session and eats 4
Let the selling price of each toy be ` ‘cd’. Let his initial times the number of cakes he eats in the morning. Had
stock be s. he not eaten any cake, he would have exceeded his tar-
Actual stock left = s - ‘ab’. get by 220 calories. If he actually falls short of his tar-
get by 180 calories, then, what is the number of cakes
Stock as shown on record = s - ‘ba’ he has eaten in the evening?
s - ba = s - ab + 81 (A) 12 (B) 8 (C) 16 (D) 10

Unit I_Chapter 1.indd 28 27/04/2017 14:16:42


Chapter 1  •  Simple Equations  |  1.29

Sol. Calories lost by him on the particular day = 500 - 180 ⇒ 13a = 26
= 320 ⇒ a=2
If he exercises for x hours, he loses (2x) × (80) calories y=5
= 160 x. a=x-z
If he eats y cakes he gains = 25y calories ⇒  y + x - z = 5 + 2 = 7
\ In the morning he loses = 160x - 25y calories Hence, the correct option is (A).
In the afternoon he loses = (320x - 75y) calories 22. Classes A and B have 35 students each. If seven girls
In the evening he loses = (240x - 100y) calories shift from class A to class B, then the number of girls
\ Total calories lost = 720x - 200y = 320 calories (1) in the classes would interchange. If four girls shift from
class B to class A, then the number of girls in class A
The second relation, as per the data is,
would be twice the original number of girls in class B.
720x = 500 + 220 What is the number of boys in class A and in class B?
x = 1, y = 2 (A) 18 and 11 (B) 24 and 17
\ Cakes eaten in the evening = 2 × 4 = 8 (C) 18 and 27 (D) 17 and 24
Hence, the correct option is (B). Sol. Let the number of girls in class A and class B be x and
y respectively
20. A test consists of 120 questions. Each correct answer,
each wrong answer, and each unanswered question in x=y+7
the test carry 1 mark, –½ mark, and –¼ mark, respec- ⇒ x – y = 7 (74)
tively. Find the maximum number of questions that the  x + 4 = 2y
candidate could have answered wrongly in the test, if
x – 2y = – 4 (75)
he scores 50 marks in it.
(A) 40 (B) 42 (C) 45 (D) 46 Solving (74) and (75), we get
Sol. Let the number of questions correctly answered, y = 11, x = 18
wrongly answered and unanswered by the candidate be \ Number of boys in A = 35 – 18 = 17
C, W and U respectively. Given that Number of boys in B = 35 – 11 = 24
C + W + U = 120 (67) Hence, the correct option is (D).
1 1 23. Ten years ago, the age of a man was 35 years more than
C - W − U = 50 (68)
2 4 twice his son’s age. After how many years from now
Subtracting (68) from (67), will the man be
3 5 (i) twice his son’s age?
W + U = 70
2 4 (A) 15
280 − 6W 6 (B) 20
U= = 56 - W (69) (C) 25
5 5
(D) Cannot be determined
2 Sol. Let the present ages of the man and the son be m years
As U ≥ 0, W ≤ 46 .
3 and s years respectively.
To satisfy (69), W must be divisible by 5. m - 10 = 2 (s - 10) + 35
\ W = 45. m = 2s + 25
Hence, the correct option is (C). (i) let us say after x years the father will be twice his
21. If 3x + y – 3z = 11 and 2x – 2z + 5y = 29 what is the son’s age.
value of x + y – z? m + x = 2 (s + x)
(A) 7 (B) 14 (C) 9 (D) 8
2s + 25 + x = 2s + 2x
Sol.  3x + y - 3z = 11
25 = x
⇒ 3(x - z) + y = 11 (70)
Hence, the correct option is (C).
2(x - z) + 5y = 29 (71)
(ii) Let us say after x years the father will be thrice his
Let x-z=a
son’s age
⇒ 3a + y = 11 (72)
2s + 25 + x = 3 (s + x)
2a + 5y = 29 (73)
25 = s + 2x
(72) × 5 - (73)

Unit I_Chapter 1.indd 29 27/04/2017 14:16:42


1.30  |  Unit 1  •  Quantitative Aptitude

We have only 1 equation but 2 unknowns. So x more than between the unit’s digit. Both these condi-
cannot be found. tions are satisfied for (A), not for choices (B) and (C).
Hence, the correct option is (D). Hence, the correct option is (D).
24. A grocer uses a weighing balance in which one pan 25. There are ten children (aged 1 to 10 years) who have
weighs 0.5 kg and the other 0.75 kg. He puts a cer- equal amounts of money. In the first transaction, the
tain quantity of food grains in 0.5 kg pan and finds eldest child gives one rupee to every child younger to
the weight (in kg) as a two-digit number. However, as him. In the second transaction, the second eldest child
the customer insists, he puts it in 0.75 kg pan. Now the gives one rupee to every child younger to him. This
indicated weight is 9.5 kg more than the weight which type of distribution continues for the next two transac-
is obtained by reversing the digits of previous weight. tions. In the end, the total sum with the children who
Which of the following cannot be the actual weight have given money to other children is half the sum of
(in kg) of the food grains? money with the children who did not give any money.
(A) 43.25 What was the original amount with each child?
(B) 36.25 (A) `36 (B) `48 (C) `50 (D) `32
(C) 41.5 Sol. Let the children be C10, C9, …, C1 in decreasing order of
(D) More than one of the above their ages.
Solutions for question 24: Let each child have `x at the beginning. The following
Let the actual weight of food grains be w. table gives the amounts with each of them at the end of
Case I: If it is kept in the pan of weight 0.5 kg each transaction.
Total weight (including that of food grains) = w + 0.5 1 → (x – 9), (x + 1) (x + 1), …, (x + 1)
2 → (x – 9), (x – 7) (x + 2), (x + 2), …, (x + 2)
On the other side of the balance let an indicated weight ‘xy’
be kept, 3 → (x – 9), (x – 7), (x – 5), (x + 3), (x + 3), …, (x + 3)
4 → (x – 9), (x – 8), (x – 5), (x – 3), (x + 4), (x + 4), …,
w + 0.5 = 10x + y + 0.75 (76)
(x + 4)
Case II: If food grains are kept in the other pan of the At the end, the total sum with children who have given
balance some money
w + 0.75 = 10y + x + 9.5 + 0.5 (77) = (x - 9) + (x - 7) + (x - 5) + (x - 3) = 4x - 24
(77) - (76) gives The total sum with those children who have not given
0.25 = 9(y - x) + 9.25 any money = 6 – (x + 4) = 6x + 24
9(x - y) = 9 \ 6x + 24 = 2(4x - 24)
x - y = 1 (78) ⇒ 2x = 72
Sol. w = 10x + y + 0.25 x = 36
So, the actual weight w = 10x + y + 0.25 Hence, the correct option is (A).
So, in the actual weight after the decimal part 0.25 must
appear, and in the integral part, the ten’s digit should be

Unit I_Chapter 1.indd 30 27/04/2017 14:16:43


Chapter 2 Ratio-Proportion–Variation
Hints/Solutions
Practice Problems 1 Ratio of amounts spent = (5 × 5) : (8 × 4) : (13 × 3)
1. Let p = 5x   q = 4x = 25 : 32 : 39
5x = a + b(3) Total number of parts of the ratio = 25 + 32 + 39 = 96
4x = a - b(4) Total amount spent, as per data = `768
Value of each part of the amounts’ ratio = 768/96 = 8
From (3) and (4),
Amount spent on costliest variety
2a = 9x and 2b = x
= Value of 39 parts = 39 × 8
\ a : b = 2a : 2b = 9 : 1
Number of costliest variety balls Paul purchased is
Hence, the correct option is (B).
= (39 × 8)/13 = 24
2. Let the number of marbles with A and B be 10x and 11x
respectively Hence, the correct option is (D).
Total number of marbles = 21x 8. a : b = 2 : 3 = (2 × 4) : (3 × 4) = 8 : 12
\ the total number of marbles must be divisible by 21. b : c = 4 : 3 = (4 × 3) : (3 × 3) = 12 : 9
Only Choice (D) violates this condition. ⎛ 9⎞ ⎛ 9⎞ ⎛ 27 ⎞
c : d = 2 : 3 = ⎜ 2 × ⎟ : ⎜ 3 × ⎟ = 9 : ⎜ ⎟
Hence, the correct option is (D). ⎝ 2⎠ ⎝ 2⎠ ⎝ 2 ⎠
3. Let the numbers of students in A, B and C be 3x, 7x and 27
8x respectively. a : b : c : d = 8 : 12 : 9 : = 16 : 24 : 18 : 27
2
If 10 students leave C and join B, C and B would have Hence, the correct option is (B).
8x - 10 students and 7x + 10 students respectively. 9. Let a, b and c be the weights of the Bimal, Basu and
7x + 10 = 80; x = 10 Bali respectively.
Total number of students = 18x = 180 a/b = 2/3 = 8/12; c/b = 3/4 = 9/12; a : b : c = 8 : 12 : 9
Hence, the correct option is (C). Given the sum of the weights (8 + 12 + 9 = 29 parts)
q+r p+r p+q is 203 kg i.e. one part is 7 kg. So Basu’s weight i.e.,
4. Given that = = =k 12 parts is 12 × 7 = 84 kg
p q r
Hence, the correct option is (A).
q + r = pk, p + r = qk and p + q = rk
10. 3x - 4y + 2z = 0 (5)
q + r + p + r + p + q = k(p + q + r)
4x - 2y - z = 0 (6)
(p + q + r) (2 - k) = 0
From (6)
\ p + q + r ≠ 0,
z = 4x - 2y
\2-k=0⇒k=2
Substituting z = 4x – 2y in (5), we get
Hence, the correct option is (D).
3x - 4y + 8x - 4y = 0
p−x 1
5. = 11x = 8y
q+ x 3
Again, z = 4x - 2y
3p − q
x = multiply by 4 on both sides, we get
4
Hence, the correct option is (B). 4z = 16x - 8y; substituting 8y = 11x
6. Let the present age of Kishore be x years, 4z = 16x - 11x = 5x
Vipin’s present age is 2(x – 1) years 5 11
x : z : y = x : x : x = 8 : 10 : 11
Given the ratio of the sum of their present ages to the 4 8
difference of their present age is 19 : 5 Hence, the correct option is (A).
11. Let a + b - c = 5x
3 x − 2 19
i.e. = \x=7 b + c - a = 6x
x−2 5
a + c - b = 7x
Sum of present ages = (k + v) = 19 years
(a + b - c) + (b + c - a) = 11x
Hence, the correct option is (B).
\ 2b = 11x
7. Ratio of prices = 5 : 8 : 13
(a + b - c) + (a + c - b) = 12x
Ratio of number of balls = 5 : 4 : 3

Unit I_Chapter 2.indd 31 27/04/2017 14:18:23


1.32  |  Unit 1  •  Quantitative Aptitude

\ 2a = 12x 18. Let the scores of Ajay be a and v respectively.


(b + c - a) + (a + c - b) = 13x a + 2v = 310 (9)
\2c = 13x v + 2a = 290 (10)
\ a : b : c = 2a : 2b : 2c = 12 : 11 : 13 Solving (9) and (10)
Hence, the correct option is (B). a = 90 and v = 110
20 p 2 − 40 pq \ a : v = 9 : 11
12. Given, = 20 ⇒ 20p2 - 60pq - 80q2 = 0
pq + 4 q 2 Hence, the correct option is (A).
Solutions for questions 19 and 20:
⇒ p2 - 3pq - 4q2 = 0
19.
⇒ (p - 4q) (p + q) = 0
p Number of Time per Total time
\ = 4 or − 1 \ Choice (C) is possible. questions question
q

Hence, the correct option is (C). Mathematics a 2x 8y

13. Let the first and second parts be a and b. English b X 7y

a 25 Given 15y = 60
Given, ⇒ =
b 6 y = 4   \ 8y = 32 min    7y = 28 min
25
a = (93) = 75 (a ) ( 2 x) 8
31 =   [a/b = 16/28 = 4/7]
(b ) ( x ) 7
Hence, the correct option is (B).
\ Ratio of the number of English questions to the
14. Let the fraction be x/y number of Maths = b/a = 7/4
x −3 1 Hence, the correct option is (B).
= ⇒ 2x – y = 11 (7)
y+5 2
20. Total number of questions is 22.
x+2 Number of English questions = (7 × 22)/11 = 14
= 1;  x – y = –2 (8)
y Time taken for English questions = (60 × 7)/15 = 28

Subtracting (8) from (7), we get x = 13; y = 15 minutes
\ Fraction = 13/15 Number of questions that can be answered in 18 ­minutes
Hence, the correct option is (C). = 18/(28/14) = 9
15. Let the number of students in three classes A, B and C Hence, the correct option is (D).
be 3x, 7x and 8x respectively. Solutions for questions 21 to 25:
7 x + 10 8 PV PV xy (3 x ) ( 2 y )
Given, = ⇒ x = 10 21. 1 1
= 2 2; = , T2 = 600 K
8 x −10 7 T1 T2 100 T2
Total = 18x = 180
\ T2 – T1 = 500 K
Hence, the correct option is (C).
Hence, the correct option is (C).
16. Number of weighing stones of 500 gms = 5000/500 =
10 d1 d2 48 d2
22. = ; = ⇒ d2 = 147
\ Number of 100 gm weights = (3/5) × 10 = 6 t12 t 2 2 16 49
Hence, the correct option is (A). Hence, the correct option is (D).
17. Let the price of tea last year be `5x per kg. Let the price 23. A a (B + C) ⇒ A = k1 (B + C)
of coffee last year be `7y per kg. B a x ⇒ B = k2 x
6 8 1 k
( 5 x ) + ( 7 y ) = 48 Ca ⇒C= 3
5 7 x x
⎡ k ⎤
( 7 y ) ⇒ y = x ⇒ 6x = ⎛⎜ x ⎞⎟ = 48
20 3 3
5x = \ A = k1 ⎢ k2 x + 3 ⎥
21 4 ⎝4 ⎠ ⎣ x⎦
⇒ x = 4 ⇒ 5x = 20 As A = 3, when x = 1,
Hence, the correct option is (B). 3 = k1 [ k2 + k3 ]  (11)

Unit I_Chapter 2.indd 32 27/04/2017 14:18:25


Chapter 2  •  Ratio-Proportion–Variation  |  1.33

As per data, A = 3, when x = 2 ‘0’ represents the unstretched position of the end of
the spring. By the application of force F1, the spring is
⎡ k ⎤
\ 3 = k1 ⎢ 2k2 + 3 ⎥ (12) stretched from position ‘0’ to ‘1’ and due to additional
⎣ 2⎦ force F2, the spring is additionally stretched from posi-
(11) – (12) gives tion ‘1’ to ‘2’.
k1 k3 ⎛k ⎞ F1 = kx
- k1 k2 + = 0 ⇒ k1 ⎜ 3 − k 2 ⎟ = 0
2 ⎝2 ⎠ F1 + F2 = k (x + 2x)
⇒ k1 = 0 F1 + F2 = (k) (3x) ⇒ F2 = 2kx
k3 (k ) (2 x)
or k2 = are the possible cases F2 as a fraction of F1 = =2:1
2 (k ) ( x)
k1 cannot be 0 (as A can’t be 0). Hence, the correct option is (B).
k
Hence k2 = 3 25. Let k, M and S be the kinetic energy, mass and speed of
2 a body respectively.
From (11)
Given: k ∝ S ² (when M is kept constant) and k ∝ M
3k1 k3 (when S is kept constant)
3 =
2
⇒ k ∝ MS² ⇒ k = CMS²
k1 k3 = 2. ⇒ k1 k2 = 3 -2 = 1.
where C is the constant of proportionality.
k1k3 2
Hence A = k1k2 x + =x+ Given that when M = 2 kg, S = 10 m/s, k = 100 joules
x x
⇒ 100 = C × 2 × 10² ⇒ C = 1/2 ⇒ k = 1/2 MS²
2 9
When x = 4, A = 4 + = = 4.5 When M = 20 kg and S = 1 m/s
4 2
Hence, the correct option is (B). k = 1/2 × 20 × 1² = 10
24. k x 2x \ A body of mass 20 kg moving with a speed of 1 m/s
has a kinetic energy of 10 joules.
0 1 2
Hence, the correct option is (C).

Practice Problems 2 Since k is an integer a + b must be a factor of 70; from


the options
Solutions for questions 1 to 8:
The sum of the ratio in options (A), (B), (C) are factors
1. The ratio of the ages of four members of a family is
of 70. But the sum of ratio 9 : 2 is not a factor of 70.
9 : 8:3:2. The average age of the family is 22 years.
What is the age of the eldest person in the family? \ 9 : 2 is not a possible ratio.
(A) 36 years (B) 32 years Hence, the correct option is (D).
(C) 12 years (D) 6 years 3. The ratio of the number of chocolates with A and B
Sol. Let the ages of the four members be 9k, 8k, 3k and 2k is 3:4. If A gives four chocolates to B, the ratio of the
years respectively. number of chocolates with them becomes 5:9. How
9k + 8k + 3k + 2k many chocolates did A have initially?
Average age = = 22 ⇒ k = 4 (A) 32 (B) 24 (C) 20 (D) 36
4
\ The age of the eldest family member = 9 × 4 = 36 Sol. Let the number of chocolates A and B be 3k, 4k
Hence, the correct option is (A). 3k − 4 5
=
2. The strength of a class is 70. Which of the following 4k + 4 9
cannot be the ratio of the number of boys and girls in \ A had 3k = 24 chocolates initially.
the class?
Hence, the correct option is (B).
(A) 2:5 (B) 3:2 (C) 11:3 (D) 9:2
4. The ratio of the monthly incomes of A and B is 3:4.
Sol. Let the ratio of the number of boys and girls in the class
The ratio of the monthly expenditures of A and B is
be a : b. Number of boys and girls will be ak and bk.
4:5. Which of the following represents a possible value
Given ak + bk = 70 of the ratio of their savings?
70 (A) 9:10 (B) 3:4
K = .
a+b (C) 13:20 (D) 4:5

Unit I_Chapter 2.indd 33 27/04/2017 14:18:27


1.34  |  Unit 1  •  Quantitative Aptitude

s1 e s s +e e a d a b
Sol. If < 1 then 1 < 1 1 < 1 = or =
s2 e2 s2 s2 + e2 e2 b c d c
Hence, the correct option is (A).
e1 + s1 3 e 4
In the given problem, = and 1 = 7. A certain number is added to each of a pair of num-
e2 + s2 4 e2 5
bers which are in the ratio 4:7. The sum of the result-
s1 3 ing numbers is 75 and their ratio (taken in the same
\ < .
s2 4 order as mentioned above) is 8:17. What is the number
added?
s1 13
Only Choice (C), i.e. = satisfies this condition. (A) –12 (B) 9 (C) –13 (D) 8
s2 20
Sol. Two numbers are in the ratio 4 : 7
Hence, the correct option is (C). Let the numbers be 4k and 7k
5. The ratio of the present ages of a husband and a wife is Let the number added to each be L.
5:4. Given that, (4k + L) + (7k + L) = 75
(i) Which of the following can be a possible ratio of ⇒ 11k + 2L = 75 (13)
their ages 20 years ago?
(A) 5:4 (B) 6:5 4k + L 8
It is also given that, =
(C) 23:20 (D) 13:10 7k + L 17
(ii) Which of the following can be a possible ratio of ⇒ 12k + 9L = 0 (14)
their ages 20 years hence? −9 L −3
(A) 5:4 (B) 6:5 ⇒k= = L
12 4
(C) 7:5 (D) 13:10
Substituting the value of k in (13) we get L = –12
Sol. Let the present ages of the husband and the wife be 5x
years and 4x years respectively. Alternate method:
(i)  Ratio of their ages 20 years ago Ratio of the resulting numbers = 8 : 17
Sum of the resulting numbers = 75
5 x − 20 5 ( x − 5 ) + 5 5
= = > (some positive quantity) Hence, the numbers obtained after addition are
4 x − 20 4 ( x − 5) 4
(8/25) × 75 = 24 and (17/25) × 75 = 51
5
 \ the ratio must exceed . Only choice (D) satis- i.e., 4k + L = 24 and 7k + L = 51
fies this condition 4
On subtraction,
Hence, the correct option is (D).
7k - 4k = 51 - 24, ⇒ 3k = 27, k = 9 and L = -12
(ii) Ratio of their ages 20 years hence
Hence, the correct option is (A).
5 x + 20 5 ( x + 5 ) − 5 5 8. Ajay went to a market to buy a total by 90 apples,
= = <
4 x + 20 4 ( x + 5) 4 oranges and bananas. He bought an equal number of
oranges and bananas. The ratio of the number of apples
5 and oranges he bought is 5:2. If the price of each
\ the ratio must be less than
4 orange was equal to that of each apple, he could have
Only Choice (B) satisfies this condition. skipped the purchase of bananas and instead purchased
Hence, the correct option is (B). the same number of apples and oranges as he actually
6. There are 2 two-digit numbers. Their product equals bought for the same total amount. If the prices (in `) are
the product of the numbers formed by reversing their all integers, find the minimum possible total expendi-
digits. Which of the following holds true if the numbers ture he could have incurred (in `).
are denoted by ab and cd? (A) 120 (B) 130 (C) 140 (D) 150
(A) a, b, d, and c are in proportion. Sol. The data is tabulated below
(B) a, b, c, and d are in proportion.
Apples Oranges Banana Total
(C) a, d, c, and b are in proportion.
(D) a, c, b, d are in proportion. Number 50 20 20 90
Price a r b
Sol. (10a + b) (10c + d) = (10b + a) (10d + c)
100ac + 10bc + 10ad + bd = 100bd + 10ad + 10bc + ac Actual amount spent = 50a + 20r + 20b
ac = bd If r = a, amount spent = 70a

Unit I_Chapter 2.indd 34 27/04/2017 14:18:29


Chapter 2  •  Ratio-Proportion–Variation  |  1.35

Given 50a + 20r + 20b = 70a ⇒ r + b = a Using the relationship each ratio =
Amount spent = 70a. Minimum value of a is 2, sum of numerators
­corresponding to b = 1, r = 1. k=
sum of denominator
\ Minimum possible amount = `140
( a + c) + (c + e) + ( a + e) 2( a + c + e) a + c + e
Hence, the correct option is (C). k= = =
(b + d ) + ( d + f ) + (b + f ) 2(b + d + f ) b + d + f
Solutions for questions 9 and 10:
The amount used to purchase one litre of petrol can be used a+c a+c+e ( a + c + e) − ( a + c) e
Now = =k = =
to purchase three litres of diesel or five litres of k­ erosene. = k b+d b+d + f (b + d + f ) − (b + d ) f
Out of a certain amount, `510 is spent on diesel.
c+e a+c+e
9. How much is spent on kerosene if equal volumes of the Similarly = =k
d + f b+d + f
three liquids are purchased with the total amount?
(A) `300 (B) `306 (C) `382 (D) `354 a + c + e) − (c + e) a
k= = and lastly,
Sol. Quantity of diesel purchased = x lts. (b + d + f ) − ( d + f ) b
\ Cost per liter of diesel = 510/x
( a + c + e) − ( a + e) c
510 × 3 k= = , hence all the choices (A),
Cost per liter of kerosene = = 306/x (b + d + f ) − (b + f ) d
x×5
Since equal volumes are purchased, amount spent on (B) and (C) are true.
306 Hence, the correct option is (D).
kerosene = × x = ` 306
x 12. P and Q are distinct two-digit numbers. Ps and Qs
Hence, the correct option is (B). denote the sums of the digits in P and Q, respectively. If 
10. What will be the amount spent on petrol if the total P Q
= , then find the minimum possible value of Ps +
amount referred in the above question is instead spent Ps Qs
to purchase equal volumes of petrol and kerosene only? Qs.
(A) `1250 (B) `1275 (C) `1955 (D) `1360
(A) 8 (B) 9 (C) 6 (D) 3
Sol. Amount spent on kerosene = x ol. Let P = ab and Q = cd Ps = a + b and Qs = c + d
S
Amount spent on petrol = 5x
10 a + b 10c + d
\ Cost involved = 6x(15) =
a+b c+d
Ratio of prices per liter of Petrol, diesel and kerosene is
(10a + b) (c + d) = (10c + d) (a + b)
15 : 5 : 3
10ac + bc + 10ad + bd = 10ac + ad + 10bc + bd
When equal volumes of all are purchased, the amounts
are also in the ratio 15 : 5 : 3. ad = bc (17)
Amount spent on diesel, in this case = 510 Let a = 1, b = 0, c = 2 d = 0
Total amount spent = (23 × 510)5 = 23 × 102 (16) This would correspond to the minimum value of Ps +
Qs, which is 3.
From (15) and (16), 6x = 2346
Hence, the correct option is (D).
x = 391
\ Amount spent on petrol = 391 × 5 = `1955 a3 c 2 + b3 d 2
13. If a:b = 3:4 and c:d = 2:3, then find
.
Hence, the correct option is (C). ab 2 d 2 + a 2 bcd
Solutions for questions 11 to 25: 19 19 19 19
(A) (B) (C) (D)
a+c c+e a+e 3 4 9 18
11. If k = = = , when all quantities are Sol. We will find it convenient to change the notation
b+d d + f b+ f
positive, then which of the following must be true? slightly.
e a Let A : B = 3 : 4 and c : d = 2 : 3
(A) k = (B) k=
f b Let A = 3x \ B = 4x
c Let c = 2y \ d = 3y
(C) k= (D) All of the above
d A3 c 2 + B 3 d 2
To find E =
a+c c+e a+e AB 2 d 2 + A2 Bcd
Sol. = = =k As each term has 3 upper case letters and 2 lower case,
b+d d + f b+ f
E is homogeneous and we can evaluate it.

Unit I_Chapter 2.indd 35 27/04/2017 14:18:32


1.36  |  Unit 1  •  Quantitative Aptitude

33 ( 22 ) + 43 (32) x3 y 2 16. In a three digit number, the units digit is the sum of
E= the other two digits, and it exceeds the tens digit by
3( 4 )(3 ) + 3 ( 4)( 2)(3) x 3 y 2
2 2 2
as much as the latter exceeds the hundreds digit. How
3 + 4 2 19 many such three digit numbers exist?
= = (A) 1 (B) 2 (C) 3 (D) 4
3( 4) + 6 18
Hence, the correct option is (D). Sol. Let the numbers satisfying the condition be denoted by
14. Amar, Bhavan, and Chetan have some coins with them. xyz.
The total number of coins with Bhavan and Chetan z = x + y, z - y = y - x, z = 2y - x = x + y
and those with Amar and Chetan are in the ratio 4 : 5. y = 2x
The total number of coins with Bhavan and Chetan z = 3x
and those with Amar and Bhavan are in the ratio 4 : 3.
Which of the following is the ratio of the number of \x:y:z=1:2:3
coins with Bhavan and Chetan? \ (x, y, z) can be (1, 2, 3), (2, 4, 6) or (3, 6, 9)
(A) 7:4 (B) 3:2 (C) 7:5 (D) 1:3 Hence, the correct option is (C).
Sol. Let the number of coins with Amar, Bhavan and Chetan 17. Mr. Ram has five sons – Ajay, Balu, Charan, Eswar and
be A, B and C respectively. Ganesh. He had some chocolates with him which he
The data is tabulated below distributed in the following manner. Twice the num-
A B C AB AC BC 2(ABC) ber of chocolates received by Ajay, thrice the number
of chocolates received by Charan and four times the
3 5 4 12 number of chocolates received by Eswar are equal.
\ A = 6 - 4 = 2, B = 6 - 5 = 1 and C = 6 - 3 = 3 Five times the number of chocolates received by Balu,
\B:C=1:3 six times the number of chocolates received by Eswar
Hence, the correct option is (D). and eight times the number of chocolates received by
Ganesh are equal. Find the minimum number of choco-
15. Manoj and Shiva, who are colleagues in an office, have lates that could have been distributed by Mr. Ram.
their monthly savings in the ratio of 2:3. Manoj spends (A) 377 (B) 661 (C) 441 (D) 551
two-thirds of his income every month. If the ratio of
their monthly incomes is 3:4, what is the ratio of their Sol. Let the number of chocolates distributed to Ajay, Balu,
expenditures? Charan, Eswar and Ganesh be a, b, c, e and g.
(A) 3:2 (B) 4:5 (C) 4:3 (D) 5:3 2a = 3c = 4e(22)
Sol. Data can be tabulated as follows: 5b = 6e = 8g(23)
Manoj Shiva Multiplying (22) by 3 and (23) by 2, we get
1. Income Ratio 3 : 4 6a = 10b = 9c = 12e = 16g
2. Savings Ratio 2 : 3 Let each of these equal k
3. Spending k k k k k 377k
(Expenditure) – – a + b + c + e + g = + + + + =
6 10 9 12 16 720
Manoj’s expenditure is 2/3 of his income. Let Manoj’s
rds
This has a minimum value of 377 when k = 720
monthly income be `36. (an arbitrary number is chosen
Hence, the correct option is (A).
in such a way that no fractions are encountered in the
calculation) a2 + c2 b2 + c2
18. If = = k and a ≠ b, which of the follow-
Hence, Manoj’s expenditure is 36 × (2/3) = 24 (18) a+c b+c
Hence, Manoj’s savings is 36 - 24 = 12 ing is equal to k?
Ratio of savings of Manoj and Shiva = 2 : 3 (A) a + c (B) b + c (C) a + b (D) a–c
Hence, Shiva’s savings = 3 × (12/2) = 18 (19)
m p
Income Ratio = 3 : 4 Sol. When two ratios like and are equal, each of them
n q
As Manoj’s income = `36, Shiva’s shall be
m− p
4 × (36/3) = `48(20) is equal to , provided n ≠ q.
n−q
Shiva’s expenditure = Income – Savings
= 48 - 18 = 30 (21)
\ k =
(
(a2 + c2 ) − b2 + c2 ) =a+b
From (18) and (21); Ratio of expenditures of Manoj ( a + c) − ( b + c )
and Shiva is 24 : 30 = 4 : 5
Hence, the correct option is (C).
Hence, the correct option is (B).

Unit I_Chapter 2.indd 36 27/04/2017 14:18:33


Chapter 2  •  Ratio-Proportion–Variation  |  1.37

19. The mean proportional between two numbers is 9. The Let the volumes of three chambers be V1, V2, V3
third proportional of the same numbers is 6561. Find respectively.
the greater of the 2 numbers.
V1 V2 V3
(A) 18 (B) 27 (C) 36 (D) 81
Sol. Let the numbers be a and b Let pressures in 3 chambers be P1, P2 and P3
a : 9 = 9 : b ⇒ ab (24) P1V1 = P2 V2 = P3 V3(27)
a : b = b : 6561 ⇒ b = 6561a 2
(25) [Since PV = KQ and as Q is same for all chambers, KQ
Solving (24) and (25) is constant]
a = 1 and b = 81 V1 : V2 : V3 = 1 : 2 : 3
Hence, the correct option is (D). So, P1 = 2P2 = 3P3
20. In a three digit number, the tens digit is the average of 6 6 6
\ P1 P2 : P3 = : :
the other two digits. The ratio of the number formed by 1 2 3
its first two digits and their sum equals the ratio of the P1 : P2 : P3 = 6 : 3 : 2
number formed by its last two digits and their sum. How
many three digit numbers satisfy these conditions? Hence, the correct option is (C).
(A) 7 (B) 8 (C) 9 (D) 6 22. The volume of a cone varies as the square of the
Sol. Let the three-digit numbers satisfying the given condi- radius of its base when its height is fixed and also var-
tions be denoted by abc ies as its height when radius of its base is fixed. If a
a+c radius of 3 cm and height of 7 cm give it a volume
b = (26) of 66 cubic ­centimetre, then what will be the radius,
2
(in ­centimetres), if the cone has a height of 6 cm and
ab bc ­volume of 308 cubic centimetres?
=
a+b b+c (A) 9 (B) 15 (C) 7 (D) 11

10 a + b 10b + c Sol. V a r h ⇒ V = k.r h ⇒ V1/( r1 h1 )
2 2 2

= = V2 / ( r2 2 h2 ) = k
a+b b+c

Hence, 66/(9 × 7) = 308/(6r²);
(10a + b) (b + c) = (10b + c) (a + b)
⇒r=7
10ab + b2 + 10ac + bc = 10ab + ac + 10b2 + bc
Hence, the correct option is (C).
ac = b2 2
⎛ a+c ⎞ 23. The amount collected per month from a consumer of
From (26), ac = ⎜ ⎟ electrical power consists of two parts—a fixed charge
⎝ 2 ⎠
(a - c)2 = 0 for providing the service and a variable charge, which
\a=c is directly proportional to the number of units of power
consumed. An amount of `700 is collected from the
From (26), b = c
consumer when he consumed 50 units in a month. It is
\a=b=c also noticed that when the consumption increases from
\ Nine numbers satisfy these conditions. 100 units per month to 200 units per month, the bill
Hence, the correct option is (C). amount increases to 5/4 times that of the former. How
much is the fixed charge per month?
21. Two diaphragms partition a cylinder into three cham-
(A) `600 (B) `350 (C) `500 (D) `400
bers, whose volumes are in the ratio of 1:2:3. Pressure
of a gas, which is in joint variation with its mass and Sol. Fixed charge = x, unit rate = y, x and y are in Rupees.
volume, is directly proportional to its mass and inversely 5/4(x + 100y) = x + 200y
proportional to its volume. If the masses of the gas in the 5x + 500y = 4x + 800y(28)
three chambers are same, find the ratio of the pressures
x + 50y = 700  (data) (29)
of the gases in the three chambers taking them in the
same order as the volumes have been taken. (Assume no y = 2, x = 600
factors other than those mentioned play a role.) Hence, the correct option is (A).
(A) 3:2:1 (B) 1:2:3 24. The distance (in metres) to which a boy can throw a
(C) 6:3:2 (D) 1:1:1 stone is inversely proportional to its weight (in kg).
Q He breaks the stone into 3 pieces whose weights (in
Sol. P a , other factors being constant where Q and V are kg) are in the ratio 1:3:2. He then throws the stones
V
mass and volume of the gas. one by one. The sum of the distances they cover is

Unit I_Chapter 2.indd 37 27/04/2017 14:18:34


1.38  |  Unit 1  •  Quantitative Aptitude

22 metres. To what distance can he throw the unbro- (900 - 300) = 600 soldiers, consumed at the rate of
ken stone? 3 kg/day/soldier, for 25 days (30)
(A) 3 m (B) 4 m (C) 6 m (D) 2 m If the soldiers were not transferred, 900 soldiers would
Sol. Distance = d weight = w have consumed it at the rate of 2.5 kg/day/soldier, the
Given d a (1/w) same food. (31)
dw = k (constant) The data can be tabulated as:
Let weights of 3 pieces be x, 3x, 2x Soldiers Consumption Number of
rate days
k k k 11 k
\ Sum of distances = + + = = 22 600 3.0 25
x 3x 2 x 6 x
k = 12x 900 2.5 How many?
Weight of unbroken stone is
Number of soldiers and the number of days for which
w = 6x = k/2  as  d. w = k food lasts are inversely proportional. The number of
d × k/2 = k   ⇒  d = 2 m soldiers increased; hence, number of days decreases.
Hence, the correct option is (D). Hence multiplication factor is (600/900).
25. A garrison of 900 soldiers had food-stock sufficient for Consumption rate and number of days are also inversely
30 days when the rate of consumption is 2.5 kg/day/ proportional. Hence, multiplication factor is 3.0/2.5
soldier. After some days of consumption at that rate, Applying the above rates of variation, the number of
300 soldiers were transferred to another garrison and days = 25 × (600/900) × (3.0/2.5) = 20 days
the balance food lasted for 25 days for the remaining The initial stock was to last for 30 days.
soldiers. If the rate of consumption of the remaining
⇒ Soldiers were transferred after 30 - 20 = 10 days
soldiers was 3.0 kg/day/soldier, after how many days
from the start, were the soldiers transferred? Hence, the correct option is (B).
(A) 12 (B) 10 (C) 8 (D) 15
Sol. The quantity of the balance of food after the transfer is
such that

Unit I_Chapter 2.indd 38 27/04/2017 14:18:35


Chapter 3 Numbers
Hints/Solutions
Practice Problems 1 7. If the odd natural number is more than or equal to 3 its
factorial’s parity would be even
1. If x = 2 and y = 3 , x + y – xy = 2 + 3 – 6
1! = 1.\ 1! is the only odd number satisfying the given
In this case, x + y – xy is irrational. condition.
If x = 2 and y = – 2 , Hence, the correct option is (A).
x + y – xy = 2 + (− 2 ) – ( 2 ) (– 2 ) = 2 8. For any perfect number, the sum of its factors is twice
In this case, x + y – xy is rational. the number.
\ We can only conclude that x + y – xy is real Hence, the correct option is (B).
(Q Any real number is one which is either rational or 9. Let x = 0. 255 = 0.25
irrational)
Hence, the correct option is (A). 10 x = 2.5 (7)
2. Choice (A) 100 x = 25.5 (8)
851 = 302 – 72 = (23) (37) Subtracting (7) from (8)
\ Choice (A) is not prime 23
x =
Choice (B) 90
589 = 252 – 62 = (19) (31) Hence, the correct option is (A).
\ Choice (B) is not prime. 10. Let x = 0. 321
Choice (C) is divisible by 3. 10 x = 3.21 (9)
Choice (D) is prime. 1000 x = 321.21  (10)
Hence, the correct option is (D).
318 53
3. Twin primes are prime numbers, which differ by 2. Subtracting (9) from (10), x = =
990 165
In Choice (A), 133 is divisible by 7 and hence it is not Hence, the correct option is (A).
a prime
11. Let x = 0.321
In Choice (B), the numbers are twin primes.
In Choice (C), 159 is divisible by 3 and hence it is not 100 x = 32.1 (11)
prime. 1000 x = 321.1 (12)
Hence, the correct option is (D). Subtracting (11) from (12),
4. Choice (A) 289
x =
Sum of the digits in the odd places= 32 900
Hence, the correct option is (A).
Sum of the digits in the even places = 21
12. Least natural number = (Least perfect cube greater than
(Sum of the digits in the odd places) – (Sum of the
599) – 599 = 729 – 599 = 130.
digits in the even places) is divisible by 11.
Hence, the correct option is (C).
\ Choice (A) is divisible by 11.
13. Let the least odd number be x.
Choices (B) and (C) are not divisible by 11.
Given, x + x + 2 + … + x + 18 = 200 ⇒ x = 11
Hence, the correct option is (A).
Hence the sum of the last five terms ⇒ 5x + 120
5. The number formed by the last three digits of a number
i.e., 175.
must be divisible by 8 for the number to be divisible
by 8. The least natural number which should be added Hence, the correct option is (B).
to the number formed by the last 3 digits of the given 14. 19019 = 19 (1001) = (19) (13) (11) (7)
number to make it divisible by 8 is 3. \ 19019 has 4 prime factors.
Hence, the correct option is (A). Hence, the correct option is (D).
6. The product of any N consecutive natural numbers is 15. The number of numbers less than N and coprime to it
divisible by N!, any for all values of N.
⎛ 1⎞ ⎛ 1⎞ ⎛ 1⎞ 4
\ When N = 7, any such product is divisible by = N ⎜1 − ⎟ ⎜1 − ⎟ ⎜1 − ⎟ = N
7! = 5040. ⎝ 2⎠ ⎝ 3⎠ ⎝ 5 ⎠ 15
Hence, the correct option is (A). Hence, the correct option is (B).

Unit I_Chapter 3.indd 39 27/04/2017 14:18:44


1.40  |  Unit 1  •  Quantitative Aptitude

16. 40 = 5 × 8; 72 = 8 × 9 its enough to check out the divis-


ibility of the number by 5, 8 and 9 \ 17689 = 133.
\ the number divisible is 7560. Hence, the correct option is (C).
Hence, the correct option is (A). 21. 349247 is odd,
\ All the powers of 2 are co-prime to it. There are an
17. 723111
infinite number of powers of 2.
units sum = 4 An infinite number of positive integers are co-prime
tens sum = 11 to it.
difference = 7 Hence, the correct option is (D).
units sum < tens sum, add difference i.e., 7 N ( N + 1)
22. Sum of the first N natural numbers =
Hence, the correct option is (C). 2
18. (a) 9000 = 9 × 1000 N ( N + 1) N ( N + 1)
= = x2 ⇒ is a perfect square.
= 32 × 103 = 23 × 32 × 53 2 2
Hence, the correct option is (C) Let us go by the choices.
N ( N + 1)
(b) 1936 = 11 × 176 = 11 × 11 × 16 Choice (A): When N = 1, = 1 which is a
­perfect square 2
= 11² × 24
Hence, the correct option is (C) N ( N + 1)
When N = 9, = 45 which is not a perfect
(c) 3969 = 9 × 441 square. 2
= 3² × 3² × 7² = 34 × 7² \ Choice (A) is ruled out.
Hence, the correct option is (C) N ( N + 1)
(d) 14553 = 11 × 1323 Choice (B): When N = 1, is a perfect square.
2
= 11 × 3 × 21² = 11 × 33 × 72
N ( N + 1)
Hence, the correct option is (C). When N = 7, = 28 which is not a perfect
square. 2
19. (a) 248 × 555 +148 × 445
= (100+ 148) 555 + 148 × 445 \ Choice (B) is ruled out.
= 100 × 555 + 148 (555 + 445) N ( N + 1)
Choice (C): When N = 1, which is a perfect
= 203500 square. 2
Hence, the correct option is (A) N ( N + 1) 8 (9)
(b) 4 1/2 + 3 1/5 – 2 1/10 – 4 1/20 = 111/20 When N = 8, = = 36 which is a perfect
square 2 2
Hence, the correct option is (B)
N ( N + 1)
( 3.37 ) + 3 × 3.37 ( 6.63) + 3 × 6.63 ( 3.37 ) + ( 6.63) When N = 48, = (24) (49) which is not a
3 2 2 3

(c) ­perfect square 2


( 3.37 ) + 2 × 3.37 × 6.63 + ( 6.63)
2 2

\ Choice (C) is ruled out.


( 3.37 + 6.63) N ( N + 1)
3

= = 3.37 + 6.63 = 10 Choice (D): When N = 1 or 8, is a perfect


( 3.37 + 6.63)
2
square. (proved above) 2
Hence, the correct option is (C). N ( N + 1)
20. 1 1 76 89 133 When N = 49, = (49), (25) which is a perfect
square 2
1
------ Choice (D) follows.
23 76 Hence, the correct option is (D).
69
23. For any value of n, unit’s place of 5n is 5.
---------
263 789 As 2n is always even, unit’s digit of 42n is 6.
789 As 4n is a multiple of 4, 74n ends with the digit 1.
-------- 5n + 42n + 74n ends with (5 + 6 + 1) i.e., 12
0
Hence 24n ends with 6.
--------
Hence, the correct option is (D).

Unit I_Chapter 3.indd 40 27/04/2017 14:18:46


Chapter 3  •  Numbers  |  1.41

24. Highest power of 5 in 240! is given by 31. Choice (A)


240 240 240 As q and r are odd, q2 and r3 are odd.
+ 2 + 3 i.e., 48 + 9 + 1 = 58
5 5 5 \ pq2r3 is odd
Hence, the correct option is (A). \ Choice (A) is always true.
25. 10 7 6 Choice (B)
(p + q)2 is even.
8 4 5
\ (p + q)2 r3 is even.
i.e, [(5 × 7) + 4] 10 + 8 = 398 is the least number satis-
fying the given condition. \ Choice (B) is always true.
Hence, the correct option is (C). Choice (C)
26. As the HCF is 14, the numbers are of the form 14a, q + r is even.
14b, where a, b are co-primes. \ (p - q + r)2 (q + r) is even.
We know that the product of two numbers will be \ Choice (C) is always true.
always product of their LCM and HCF. Choice (D)
14a × 14b = 1232 × 14 ⇒ ab = 88 If p = 1, q = 3 and r = 5, pqr leaves a remainder of 3
The possible pairs of a and b are: (1, 88), (2, 44), (4, 22) when divided by 4.
and (8, 11) If p = 3, q = 5 and r = 7, pqr leaves a remainder of 1
But the pairs which are co primes are (1, 88) and (8, 11). when divided by 4.
Hence two pairs are possible. \ Choice (D) is not always true.
Hence, the correct option is (B). Hence, the correct option is (D).
27. For number of this type (i.e., 1234 . . . nn – 1 … 1), the 32. abcde = 10000a + 1000b + 100c + 10d + e
number of digits in the square root of the number will
acdbe = 10000a + 1000c + 100d + 10b + e
be equal to the middle digit of the number.
The difference of abcde and acdbe
Hence, the correct option is (C).
= (10000a + 1000b + 100c + 10d + e) – (10000a +
28. Let the four prime numbers be a, b, c and d.
1000c + 100d + 10b + e) = (990b – 900c – 90d)
Given a × b × c = 2431 and b × c × d = 4199
18 (55b – 50c – 5d) which is always divisible by 9
a × b × c 2431 a 11 and 18
\ = ⇒ =
b × c × d 4199 d 19 Hence, the correct option is (D).
\ d = 19
33. The given expression is of the form
Hence, the correct option is (B).
a3 + b3 − c 3 + 3abc
29. A minimum of 5 coins are required to pay 69 paise. =
(1 50 p, 1 10 p, 1 5 p and 2 2 p). a 2 + b 2 + c 2 − ab + bc + ca
A minimum 3 coins are required to pay H1.05 (2 50 p where a = 0.68, b = 0.67 and c = 0.5
and 1 5 p) a3 + b3 + c 3 − 3abc
=a+b−c
A minimum of 3 coins are required to pay 85p (1 50 p, a 2 + b 2 + c 2 − ab − bc − ca
1 25 p and 1 10 p)
= 0.68 + 0.67 - 0.5 = 0.85
\ Minimum number of coins required in total = 11
Hence, the correct option is (D).
Hence, the correct option is (D).
30. b = a + 2 and c = a + 4 34. 22 and 32 when divided by 6 leave remainders of 4 and
If a is even, b and c are also even. As a, b and c are 3 respectively. All other primes are of the farm (6k ± 1)2
prime, this is not possible. when divided by 6 leave a remainder of 1. Hence the
sum of the distinct possible remainders is 8.
\ a, b and c are odd.
Least possible value of a is 3. Hence, the correct option is (B).
If a = 3, b = 5 and c = 7 35. 2 3 7
If a > 3, it must be in the form 6k ± 1 where k is a natu-
ral number. If a is of the form 6k - 1, c is not prime. 1 2 3
(a, b, c) = (3, 5, 7) is the only possibility. The least possible number is (3 × 3 + 2) × 2 + 1 = 23
Hence, the correct option is (D). Hence, the correct option is (D).

Unit I_Chapter 3.indd 41 27/04/2017 14:18:46


1.42  |  Unit 1  •  Quantitative Aptitude

36. p = 23 . 32. 72. 116 40. Two different dividends and two remainders,
q = 22. 31. 54. 112. 132 \ take HCF of difference of dividends and remainders
The common prime factors of p and q are 2, 3 and 11. 6850 – 50 = 6800
\ GCD (p, q) must have only these prime factors. 2575 – 25 = 2550
\ GCD (p, q) = 2 min (3, 2). 3min (2, 1). 11min (6, 2) 6800 = 17 × 24 × 52
= 22. 31. 112 = (4) (3) (121) = 1452 2550 = 17 × 150 = 17 × 2 × 3 × 52
Hence, the correct option is (B). HCF = 17 × 2 × 52 = 850
37. Two numbers having no common factor except one are \HCF = 850 is the greatest number
called as relative primes. Among the options the pairs Hence, the correct option is (B).
which are relative primes are 57,61; 396,455; and 6561, 41. It is a case of three divisors and the same remainder
1024
Least number = LCM of (12,18 and 33) + 5
Hence, the correct option is (B).
= 396 + 5 = 401
38. (a) 
The units digit of 8 repeats after every four Hence, the correct option is (D).
­powers. Expressing 173 in terms of 4, we have
8173 = 84 × 43 + 1 and hence the last digit of 8173 and 81 42. LCM of 38 and 57 = 114
should be the same. Hence 8173 has the units digit Remainder when 1994 is divided by 114 is 56. Number
of 8. to be added to 1994 to make it a multiple of 114 is 58. in
Hence, the correct option is (C). order to leave a remainder of 28, the number to be added is
58 + 28 = 86
(b) Last digit of 518163 is the same as the last digit
Hence, the correct option is (C).
of 8163; 8163 = 8(4 × 40) + 3. Since the last digit of the
power of 8 is 3, 8163 will have the same units digit 43. Take HCF of difference of dividers and remainders.
as 83 whose last digit is 2, 142 will have same units HCF of 3300 – 23 = 3277 and 3640 –­ 24 = 3616
digit as 2157 2157 = 2(4 × 39 +1) cycle for the last digit of 3616 = 25 × 113
power of 2 is also 4. Checking divisibility of 3277 by 113
 Hence 2157 will have the same units digit as 21 3277 = 29 × 113
whose units digit is 2. Hence 518163 + 142157 will HCF = 113
have the last digit of 2 + 2 = 4
Hence, the correct option is (B).
Hence, the correct option is (B). 44. Take difference of two pairs of dividends and find their
(c) 1567143 has the same last digit as 7143 HCF
7143 = 7(4 × 35) + 3 140 – 68 = 72
Since the last digit of the power of 7 has a cycle of 248 – 140 = 108
4, 7143 will have the same last digit as 73 i.e., 3. HCF of 72, 108
1239197 has the same last digit as 9197. 36 × 2 = 72
For 9197, since power of 9 is odd its last digit is 9. 36 × 3 = 108
Hence, 1239197 has last digit of 9. HCF = 36.
25661027 has the same last digit as 61027, a i.e., 6, Hence, the correct option is (A).
since 6 raised to any power will always have a last
digit of 6. 45. Time to toll together again = LCM of 5, 6, 10, 12, 15
Hence, last digit of 1567143 × 1239197 × 25661027   = 60 seconds.
will be the last digit of 3 × 9 × 6 = 162 Hence, the correct option is (C).
i.e., 2 46. Let the numbers be 3x, 4x and 5x.
Hence, the correct option is (A). LCM (3x, 4x, 5x) = x LCM (3, 4, 5) = 60x
39. This is of the form (43)a – (21)a, where a = 5n. This is Given 60x = 480
always divisible by 43 – 21 = 22. x = 8
Hence it is also divisible by 11. sum 3x + 4x + 5x =12x = 96
Hence, the correct option is (A). Hence, the correct option is (A).

Unit I_Chapter 3.indd 42 27/04/2017 14:18:47


Chapter 3  •  Numbers  |  1.43

47. (121)8 = (1 × 82 + 2 × 8 + 1 × 80)10 = (81)10 53 = 125


2 81 We observe that for five, every time the unit’s digit is
2 40 – 1 repeated.
2 20 – 0 Similarly for 7, it is for every 4 times and for 3, it is for
every 4 times.
2 10 – 0
\ 57 × 712 × 39 = 5 × 74 x3 × 34 x2+1 which has unit’s digit
2 5 – 0
   = 5 × 1 × 3 = 5
2 2 – 1
\ Unit’s digit is five.
2 1 – 0
Hence, the correct option is (B).
\ (121)8 = (81)10 = (1010001)2
50. When, 31 ÷ 6, remainder = 3
Hence, the correct option is (C).
When, 32 ÷ 6, remainder = 3
48. (ACD)16 = (A × 162 + C × 16 + D × 160)
When, 33 ÷ 6, remainder = 3
= (10 × 256 + 12 × 16 + 13 × 1)10
We observe that, 3 power any number when divided
= (2765)10
by 6, leaves a remainder of 3.
Hence, the correct option is (A).
\ When 386 is divided by 6, the remainder 3.
49. 51 = 5
Hence, the correct option is (B).
52 = 25

Practice Problems 2
Solutions for questions 1 to 50: 2. What should be subtracted from 546789 so that it
1. If N = 1223334444 … and is a 100-digit number, find becomes a multiple of 7?
the remainder when N is divided by 16. (A) 5 (B) 6 (C) 7 (D) 8
(A) 15 (B) 13 (C) 11 (D) 9 Sol. When 546789 is divided by 7, the remainder is 5. Hence
Sol. The number n, the number of times it occur in N and 5 should be subtracted from 546789 so that it becomes
the number of digits it contributes and the total number a multiple of 7.
of digits are tabulated below. Hence, the correct option is (A).
3. A, B, and C are digits and 64A3B6C is divisible by
n No. of No. of digits Total number
occurrences of digits
360. How many values can (A, B) take?
(A) 8 (B) 10 (C) 9 (D) 5
1 1 1 1
Sol. The number is divisible by 10.
2 2 2 3
\ c = 0
3 3 3 6 The number is divisible by 8
– – – – \ B = 1, 3, 5, 7 or 9.
9 9 9 45 The number is divisible by 9. The sum of all the known
10 10 20 65 digits of the number leaves a remainder of 1.
11 11 22 87 \ A + B = 8 or 17.
12 6 12 99 \ (A, B) can be (7, 1) (5, 3) (3, 5) (7, 1) or (8, 9).
1 (of 12) Half 1 100 It can take 5 values.
Hence, the correct option is (D).
\ The last 4 digits of N are 2121
4. If n is a positive integer greater than 1, n5 – n is always
The first 2 comes from the 5th 12 divisible by
The 12 comes from the 6th 12 (A) 5 (B) 7 (C) 11 (D) 13
The 1 is part of the 7th 12. Sol. If n = 2, 25 – 2 = 32 – 2 = 30
N 2121 If n = 3, 35 – 3 = 243 – 3 = 240
\ Rem = Rem =9
16 16 If n = 4, 45 – 4 = 1024 – 4 = 1020
Hence, the correct option is (D). In all the cases n5 – n is divisible by both 3 and 5.

Unit I_Chapter 3.indd 43 27/04/2017 14:18:47


1.44  |  Unit 1  •  Quantitative Aptitude

Only 5 is there in the options. The number of round is equal to the number of primes
Hence, the correct option is (A). less than 100, which is 25.
Hence, the correct option is (A).
General proof: Any value of n where n is a positive integer
can be expressed in the form 5k or 5k – 1 or 5k – 2 or 5k – 3 7. The product of seven integers, which are not necessarily
or 5k – 4 where k is an integer. distinct, between 5 and 19 both exclusive is 16081065.
If n = 5k, n is always divisible by 5 Find the sum of these integers.
(A) 81 (B) 83 (C) 85 (D) 79
If n = 5k – 1, n + 1 = 5k is always divisible by 5
Sol. 16081065 = 5 (3216213) = 5(9) 357357
If n = 5k – 2, n2 + 1 = 5k ′ is always divisible by 5 = 5(9)(357)(1001)
If n = 5k – 3, n2 + 1 = 5k ′ is always divisible by 5 = 5(9)3(7)(17)7(11)(13)
If n = 5k – 4, n – 1 = 5k – 4 – 1 = 5k – 5 = 5(k – 1) and is = 7(7)(9)15(11)(13)(17)
always divisible by 5. This is the only way to express the number as the prod-
Hence in general, for any n, uct of 7 numbers between 5 and 19. The sum of these 7
factors is 79.
n(n – 1) (n + 1) (n2 + 1) = n5 – n is always divisible by 5.
Hence, the correct option is (D).
5. The difference of any 40-digit number and its reverse is
always divisible by _____. 8. The middle digit of a three-digit number is equal to the
(A) 9 but not always by 11 sum of the other two digits. Find the number of factors
(B) 11 but not always by 9 of the greatest such odd number.
(C) 99 (A) 10 (B) 12 (C) 8 (D) 14
(D) 198 Sol. Let the three-digit number be abc.
Sol. Consider an n-digit number. b = a + c
Consider n = 3 and 4 The greatest odd number must have the greatest pos-
Let P = abc = 100a + 10b + c sible value of a and an odd value of c. (Also b ≤ 9)
The reverse Q = cba = 100c + 10b + a \ a = 8, c = 1 and b = 9
\ P – Q = 99(a – c) or abc = 891 = 111(34)
Let P = abcd = 1000a + 100b + 10c + d Number of factors of 891 is (1 + 1) (4 + 1) = 10
The reverse Q = dcba = 1000d + 100c + 10b + a Note: b = a + c means the number is divisible by 11.
\ P – Q = 999(a – d) + 90(b – c) Hence, the correct option is (A).
For n = 3, P – Q is divisible by both 9 and 11
9. The sum of all the factors of 11111111 is _____.
For n = 4, P – Q is divisible by 9 but not necessarily by (A) 11599268 (B) 13549728
11. In general when n is odd, the difference is divisible (C) 15479848 (D) 12499488
by 9 and 11. When n is even, the difference is divisible
by 9 but need not be divisible by 11. In the given prob- Sol. 11111111 = 11(1010101) = 11(101) (10001)
lem, N is 40, an even number. = 11(101) (11025 -1024) = 11(101) (1052 - 322)
\ The difference must be divisible by 9 but not neces- = 11(101) (137) (73)
sarily by 11. 112 − 1 1012 −1 1372 −1
Sum of all the factors = . . .
Hence, the correct option is (A). 11 −1 101 −1 137 − 1
6. An organization has 99 employees, who were all 732 −1
= 12 (102) (138) (74) = 12499488
assigned numerical codes from 2 to 100. A certain 73 −1

number of rounds of an emergency exit drill took place. Hence, the correct option is (D).
In the first round, all the employees whose codes were
10. If 1 ≤ R ≤ 50, how many values of R are such that
divisible by 2, made an exit. In the second round, all the
(R – 1)! is not divisible by R?
remaining employees whose codes were divisible by 3
(A) 20 (B) 16 (C) 15 (D) 22
made an exit and so on, until all the employees exit. In
each round, at least one employee made an exit. How Sol. If (R – 1)! is not divisible by R, R is a prime number.
many rounds of the drill took place? Since 1 ≤ R ≤ 50, the objective is to find the number of
(A) 25 (B) 24 (C) 49 (D) 99 prime numbers between 1 and 50. On calculating we
Sol. In the first round, employees 2, 4, …, 180 made an exit. find there are 15 primes between 1 and 50. Hence there
In the second round, 3, 9, 15 made an exit. are 15 values of R such that (R – 1)! is not divisible by
R. Also for R = 4, (R – 1)! i.e., 3! is not divisible by 4.
In the third round, 5, 25, 35, 55, 65, 85, 95 made an exit.

Unit I_Chapter 3.indd 44 27/04/2017 14:18:48


Chapter 3  •  Numbers  |  1.45

Hence, the total number of values is (15 + 1) = 16 Substituting the smallest value 3 for k the multiple of 7
Hence, the correct option is (B). is obtained.
11. Let S be the set of positive integers such that the \The smallest number = 9k + 6 = 9 x 3 + 6 = 33
­following conditions are satisfied. So, the general form of the number satisfying the con-
  I.  The elements of S range from 2000 to 2400. dition is 33 + 63k
II.  Each element of S has only even digits. Dividing 999, the largest 3-digit number that is ­possible,
How many elements of S are divisible by 6? by 63, the quotient is 15. Hence, k = 15 and 63k + 33 =
(A) 18 (B) 15 (C) 13 (D) 17 978; which is the largest 3-digit number
Sol. The numbers between 2000 and 2400 (both inclusive), Hence, the correct option is (D).
which have only even digits and which are multiplies
of 3 are listed below. (The sum of the digits has to be a 14. Find the sum of all the possible distinct remainders
multiple of 6) which are obtained when cubes of prime numbers are
divided by 6.
2004 2202 2400
(A) 9 (B) 11 (C) 13 (D) 15
2022 2208
Sol. Prime numbers less than 5 are 2 and 3.
2028 2220
23 and 33 leave respective remainders of 2 and 3 when
2040 2226 divided by 6.
2046 2244 Prime numbers greater than or equal to 5 are of the
2064 2262 form 6k ± 1 where k is a natural number.
2082 2268 (6k + 1)3 leaves a remainder of 1 when divided by 6.
2088 2280 (6k - 1)3 leaves a remainder of 5 when divided by 6.
2286 \ Sum of all the distinct possible remainders is 11.
There are 18 such numbers. Hence, the correct option is (B).
Hence, the correct option is (A). 15. From a book, in which the pages are numbered in the
usual way with odd numbers appearing on the right
⎛ 1⎞ ⎛ 1⎞ ⎛ 1⎞ ⎛ 1⎞ ⎛ 1 ⎞
12. ⎜1 − ⎟ ⎜1 − ⎟ ⎜1 − ⎟ ⎜1 − ⎟ …. ⎜1 − = pages and even numbers on the left pages, 31 consecu-
⎝ 4 ⎠ ⎝ 9 ⎠ ⎝ 16 ⎠ ⎝ 25 ⎠ ⎝ 900 ⎟⎠
tive leaves were torn off. Which of the following could
29 31 29 31 be the sum of the 62 page numbers on these leaves?
(A) (B) (C) (D)
60 60 30 30 (A) 1955 (B) 2201
⎛ 1⎞ ⎛ 1⎞ ⎛ 1⎞ ⎛ 1 ⎞ ⎛ 1 ⎞ (C) 2079 (D) None of these
Sol. ⎜1 − ⎟ ⎜1 − 9 ⎟ ⎜1 − 16 ⎟ ⎜1 − 25 ⎟ …… ⎜1 − 900 ⎟
⎝ 4⎠ ⎝ ⎠ ⎝ ⎠ ⎝ ⎠ ⎝ ⎠ Sol. Let S be the sum of the 62 page numbers. The sum
of the first 62 numbers is 31(63) = 1953.(i.e., in this
⎛ 1 ⎞⎛ 1⎞⎛ 1 ⎞⎛ 1 ⎞ ⎛ 1 ⎞ case S = 1953). If first leaf is left intact and instead leaf
= ⎜1 − 2 ⎟ ⎜1 − 2 ⎟ ⎜1 − 2 ⎟ ⎜1 − 2 ⎟ ….. ⎜1 − 2 ⎟
⎝ 2 ⎠⎝ 3 ⎠⎝ 4 ⎠⎝ 5 ⎠ ⎝ 30 ⎠ 32 (comprising pages 63, 64) is torn off, S would be
1953 – (1 + 2) + (63 + 64) or 1953 + 124. If the leaves
⎛ 1 ⎞ ⎡⎛ 1 ⎞ ⎛ 1 ⎞ ⎤ ⎡⎛ 1 ⎞ ⎛ 1 ⎞ ⎤ torn off are 3 to 33, S would be 1953 + 2(124). In gen-
= ⎜ 1 − ⎟ ⎢⎜ 1 + ⎟ ⎜ 1 − ⎟ ⎥ ⎢⎜ 1 + ⎟ ⎜ 1 − ⎟ ⎥
⎝ 2 ⎠ ⎣⎝ 2 ⎠ ⎝ 3 ⎠ ⎦ ⎣⎝ 3 ⎠ ⎝ 4 ⎠ ⎦ eral, S would be of the form 1953 + 124n, i.e, 1953,
2077, 2201, …. Among the choices, only 2201 is of this
⎛ 1⎞ ⎛ 1 ⎞ ⎛ 1 ⎞
⎜1 + 4 ⎟ …. ⎜1 − 30 ⎟ ⎜1 + 30 ⎟ form.
⎝ ⎠ ⎝ ⎠ ⎝ ⎠ Hence, the correct option is (B).
1 31 31 16. K(N) denotes the number of ways in which N can be
= × 1 × 1 × ……. 1 × = .
2 30 60 expressed as a difference of two perfect squares. Which
Hence, the correct option is (B). of the following is maximum?
13. What is the largest three-digit number which when (A) K(110) (B) K(105)
divided by 9 leaves a remainder 6 and when divided (C) K(216) (D) K(384)
by 7 leaves a remainder 5? Sol. Let N = a2 - b2 = (a - b) (a + b)
(A) 988 (B) 989 (C) 992 (D) 978 If a - b and a + b are of opposite parity, a and b will not
Sol. The number is of the form 9k + 6 be natural numbers.
This number when divided by 7 leaves a remainder 5. \ For a and b to be natural numbers, a - b and a + b
So, 9k + 6 – 5 is divisible by 7 must be positive.

Unit I_Chapter 3.indd 45 27/04/2017 14:18:51


1.46  |  Unit 1  •  Quantitative Aptitude

\ K(N) represents the number of ways of expressing N 18. In how many ways can 152100 be expressed as a
in the form (a - b) (a + b), where a - b and a + b are ­product of two different factors?
positive. (A) 24 (B) 36 (C) 39 (D) 40
The numbers in the 5 choices (N ), their prime factors, Sol. Expressing 152100 as product of prime factors, we
and k(N) are tabulated below. have 152100 = 22 × 32 × 52 × 132
N Prime factors K(N) Number of ways in which 152100 can be expressed as
110 2(5) (11) 0
a product of two different factors
105 (3)(5)(7) 4 = 1/2 [(2 + 1)(2 + 1)(2 + 1)(2 + 1) – 1]
216 23 33 4 = 1/2 [81 – 1] = 1/2 [80] = 40
384 2 3
7
6
Hence, the correct option is (D).
450 213252 0
19. If a, b, c, and d are natural numbers such that ad + bd = cd,
If there is only one 2 in N, K(N) = 0 which of the following is true?
If there is no. 2, K(N) = number of ways of expressing (A) The minimum of a, b, and c is at least d.
N as a product of two factors. (B) The maximum of a, b, and c is at most d.
If there is more than one 2, (say if these are 2m or 2m + 1) (C)  d lies between the minimum of a, b, and c and the
2’s), the 2’s can be split in m ways. If there are n of maximum of a, b, and c.
some other prime factors, those factors can be split it (D) d=1
(n + 1) ways. Sol. Suppose, a = 3, b = 4, c = 5 and d = 2
\ For N = 216 = 2333, the 2’s can be split as 2, 22, i.e., Then they satisfy ad + bd = cd
in 1 way. The 3 threes can be split as 0.3; 1.2; 2.1 or 3.0,
In this case, the minimum of a, b and c is at least d.
i.e., in 4 ways.
Hence, the correct option is (A).
\ K(N) = 4.
20. Let w, x, y, and z be four natural numbers such that their
For N = 384 = 27(3), the 2’s can be split as 2, 26; 22, 25
sum is 8m + 10 where m is a natural number. Given m,
or 23, 24 . For each of these split, the one 3 can go with
which of the following is necessarily true?
either part.
(A) The maximum possible value of
\ k(N) = 3 (N) = 6. This is the maximum. w2 + x2 + y2 + z2 is 6m2 + 40m + 26.
Hence, the correct option is (D). (B) The maximum possible value of
17. N is a natural number obtained by adding 16 to the w2 + x2 + y2 + z2 is 16m2 + 40m + 28.
product of four consecutive even natural numbers. How (C) The minimum possible value of
many of the following statements are always true? w2 + x2 + y2 + z2 is 16m2 + 40m + 28.
(1) N is divisible by 32 (D) The minimum possible value of
(2) N is divisible by 16 w2 + x2 + y2 + z2 is 16m2 + 40m + 26.
(3) N is divisible by 64 Sol. Given w + x + y + z = 8 m + 10.
(4) N is a perfect square
In m = 1, x + y + z = 6 m + 10 = 16
(A) 0 (B) 1 (C) 2 (D) 3
Sol. Let the even natural numbers be 2k, 2k + 2, 2k + 4 and When the sum of three natural numbers is constant, the
2k + 6. sum of their squares is minimum when the numbers are
as close as possible.
N = 16 + (2k) (2k + 2) (2k + 4) (2k + 6)
So the four numbers must be 2m + 2, 2m + 2, 2m + 3
= 16(1 + k(k + 1) (k + 2) (k + 3)) and 2m + 3.
= 16(1 + k(k + 3) (k + 1) (k + 2)) \ the minimum value of w2 + x2 + y2 + z2 = (2m + 2)2 +
= 16(1 + (k + 3k) (k + 3k + 2)
2 2 (2m + 2)2 + (2m + 3)2 + (2m + 3)2 = 16m2 + 40m + 26
= 16(1 + (k2 + 3k)2 + 2(k2 + 3k)) Hence, the correct option is (D).
= 16(k2 + 3k + 1)2 21. N is a natural number greater than 6, find the remainder
obtained when N 7 – N is divided by 6.
k2 + 3k + 1 is odd for any positive integral value of k.
(A) 0 (B) 1 (C) 2 (D) 3
⇒ (k2 + 3k + 1)2 is also odd.
ol. N7 - N = N (N6 - 1) = N((N3)2 - 1) = N(N3 - 1) (N3 + 1)
S
\ 16 (k2 + 3k + 1)2 is a perfect square divisible by 16.
Hence only (B) and (D) are true. N(N - 1) (N2 + N + 1) (N + 1) (N2 - N + 1)
Hence, the correct option is (C). = N(N - 1) (N + 1) (N2 + N + 1) (N2 - N + 1)

Unit I_Chapter 3.indd 46 27/04/2017 14:18:51


Chapter 3  •  Numbers  |  1.47

N - 1, N and N + 1 are consecutive integers. The p­ roduct (A) 1 (B) 2


of any three consecutive integers is divisible by 6. (C) 3 (D) 4
\ N 7 - N is divisible by 6. ol. Let the smaller number be n and the larger number
S
\ the remainder is 0. be N.
Hence, the correct option is (A). N ² + n3 = 593
22. Find the number of ways in which 24700 can be ⇒ N < 25 and N + 55 = n², ⇒ n² < 55 + 25 ⇒ n < 9
expressed as a product of two co-prime factors. for n = 8, n3 = 512 and N ² = 593 – 512 = 81 ⇒ N = 9
(A) 4 (B) 16 (C) 8 (D) 32 \N–n=1
Sol. 24700 = (247) (100) = (19) (13) (12 ) (5 )
2 2
Hence, the correct option is (A).
Number of ways in which a number having n prime 25. Ramu was given a problem of adding a certain num-
factors can be expressed as a product a two co-prime ber of consecutive natural numbers starting from 1. By
factors = 2n - 1 mistake, he added a number twice. He obtained the sum
\ required number of ways = 24 - 1 = 8 as 860. Find the number he added twice.
Hence, the correct option is (C). (A) 15 (B) 40 (C) 25 (D) 30
1 Sol. Let the number added twice be x.
23. Find the value of 2 + The correct sum = 860 – x.
2
2+ N ( N + 1)
2 This must be in the form =
2+
2 2
2+ where N is a natural number.
1
2+
2 N ( N + 1)
When N = 39, = 780
(A) 103/52 (B) 113/52 2
(C) 123/52 (D) 133/52 N ( N + 1)
When N = 40, = 820
1 1 2
Sol. 2 + = 2+
2 2 N ( N + 1)
2+ 2+ When N = 41, = 861
2 2 2
2+ 2+ 860 – x < 860
2 2
2+ 2+ \ N ≤ 40.
1 5
2+
2 2 If N = 40, x = 40.
1 1 If N < 40, x > N. This is not possible.
= 2 + = 2+
2 2 Hence, the correct option is (B).
2+ 2+
2 2 26. How many integers when squared would exceed a
2+ 2+
4 14 ­perfect square by 113?
2+
5 5 (A) 5 (B) 4 (C) 3 (D) 2
1 1 1 Sol. Let each integer satisfying the given condition be
= 2 + = 2+ = 2+ denoted by N.
2 2 2
2+ 2+ 2+ N 2 exceeds a perfect square by 113.
10 28 10 38
2+ + Let us denote the perfect square by x2
14 14 14 14
N 2 - x2 = 113.
1 1 1 38
= 2 + = 2+ = 2+ = 2+ (N - x) (N + x) = 113.
28 76 28 104 104
2+ + 113 is prime.
38 38 38 38
\ (N - x, N + x) = (-1, -113) or (1, 113) or (113, 1) or
208 38 246 (-113, -1).
= + = = 123/52
104 104 104 \ (N, x) = (-57, - 56) or (57, 56) or (57, -56) or
Hence, the correct option is (C). (-57, 56)
N has two possible values, –57 or 57.
24. When the square of a number and the cube of a smaller
number are added the result is 593. If the square of the Hence, the correct option is (D).
smaller number exceeds the bigger number by 55, find 27. If x, y are positive integers and x2 – y2 = 255, how many
the difference of the two numbers. ordered pairs (x – y, x + y) are there?

Unit I_Chapter 3.indd 47 27/04/2017 14:18:53


1.48  |  Unit 1  •  Quantitative Aptitude

(A) 4 (B) 8 have 9999 – 297 = 9702 which is exactly divisible by


(C) 6 (D) 12 693 and hence by 7, 9 and 11. The largest 4-digit num-
ol. x - y = 255
S 2 2 ber which when divided by 7, 9 and 11 leaves a remain-
der of 5 in each case = 9702 + 5 = 9707
(x - y) (x + y) = 255.
Hence, the correct option is (C).
Both x - y and x + y must be positive.
32. N1 and N2 are natural numbers not more than 100.
Also x - y < x + y.
A ⊕ B is defined as the remainder of A divided by B.
\ (x - y, x + y) = (1, 255); (3, 85); (5, 51) or (15, 17) A # B is defined as the product of A and B.
Hence, the correct option is (A). (N1 ⊕ 8) # (N2 ⊕ 7) = 21. How many possible values
28. If N = 24 × 32 × 73 × k is a perfect square as well as does (N1, N2) have?
­perfect cube, find the total number of factors of the (A) 144 (B) 180 (C) 168 (D) 192
least value of k, given k is a natural number. Sol. (N1 ⊕ 8) # (N2 ⊕ 7) = 21. In other words, the product of
(A) 40 (B) 120 (C) 80 (D) 60 the remainders of N1 divided by 8 and N2 divided by 7
Sol. N = (2 ) (3 ) (7 ) k is a perfect square and a perfect cube.
4 3 3
is 21.
Hence the index of each of its prime factors must be \ The only possible remainders when N1 and N2 are
divisible by 6 divided by 8 and 7 are 7 and 3 respectively.
\ Least value of k is 22 (34) (73) (Q Any remainder must be less than the divisor).
\ Total number of factors of k is (3) (5) (4) = 60 N1 and N2 are natural numbers not more than 100.
Hence, the correct option is (D). N1 can be 7, 15, 23, …, 95
29. If the least positive integer divisible by 22 ⋅ 3 ⋅ 5, 3 ⋅ 52 ⋅ 7 N2 can be 3, 10, 17, …, 94
and 5 ⋅ 7 ⋅ 112 has x distinct prime factors, then find x. N1 has 12 possible values and N2 has 14 possible values.
(A) 3 (B) 5 (C) 6 (D) 7
\ (N1, N2) has 168 possible values.
Sol. Least positive integer divisible by 22.3.5,3.52.7 and
Hence, the correct option is (C).
5.7.112 is their LCM. = 22. 3.52. 7. 112.
33. There are a certain number of soldiers in a field. If the
\ Its distinct prime factors are 2, 3, 5, 7 and 11.
soldiers are arranged in rows of 8 or 15 or 20, one sol-
Number of distinct prime factor = 5. dier is left out. If the soldiers are arranged in rows of
Hence, the correct option is (B). 9 or 13, four soldiers only are left out. Find the number
30. Find the smallest four-digit number which when of soldiers in the field.
divided by 9 leaves the reminder 5 and when divided by (A) 109 (B) 113 (C) 117 (D) 121
11 leaves the remainder 7. Sol. If the soldiers are arranged in rows of 8 or 15 or 20, one
(A) 1058 (B) 1041 (C) 1089 (D) 1085 soldier is left to stand alone in the last row. Hence if the
Sol. LCM of 9 and 11 is 99. When the smallest four-digit total number of soldiers is divided by 8 or 15 or 20, the
number 1000 is divided by 99, we have the remainder remainder will be 1. Similarly, if the total number of
as 10. Hence, 1000 – 10 = 990 is divisible by 99 soldiers is divided by 9 or 13, the remainder will be 4.
An option satisfying both these conditions is only.
Thus the smallest four-digit number which is divisible
by 9 and 11 is 990 + 99 = 1089 Alternate method:
The smallest four-digit number which when divided by Number of soldiers on the field = LCM (8, 15, 20)c + 1
9 leaves a remainder 5 and when divided by 11 leaves a = (120c + 1), where c is a constant
remainder 7 = 1089 – C, where C is the common differ- Number of soldiers on the field = LCM (9, 13)k + 4
ence between the divisor and the remainder in both the
= 117k + 4 where k is a constant.
cases. C = 9 – 5 = 4 or C = 11 – 7 = 4
Hence the smallest four-digit number required Hence 120c + 1
= 1089 – 4 = 1085 = 117k + 4.
Hence, the correct option is (D). The above equation is satisfied when k = 1 for c = 1
31. Find the largest four-digit number which when divided Thus number of soldiers in the field = 1(120) + 1 = 121
by 7, 9, and 11 leaves a remainder of 5 in each case. Hence, the correct option is (D).
(A) 9236 (B) 9467 (C) 9707 (D) 9763 34. Rohan has a certain number of (less than 10000) sweets
Sol. LCM of 7, 9 and 11 = 7 × 9 × 11 = 63 × 11 = 693. with him. He would be left with 1 sweet, if he distrib-
Dividing the largest 4-digit number 9999 by 693 we utes them equally among 12 or 16 or 18 children. If he
get a remainder of 297. Subtracting 297 form 9999, we distributes them equally among 17 children, he would

Unit I_Chapter 3.indd 48 27/04/2017 14:18:53


Chapter 3  •  Numbers  |  1.49

be left with no sweets. Find the number of possibilities Since y can’t be negative, y = 4
for the sweets he has. x = 28/y = 28/4 = 7
(A) 2 (B) 3 (C) 4 (D) 5
Hence the larger of the two numbers is 7x = 7 × 7 = 49
Sol. Let the number of sweets with Rohan we N.
Alternate method:
N = k1 LCM(12, 16, 18) + 1 = 17 k2, where k1 and k2 are
Going by the options, option (A) says the larger number
natural numbers.
is 28. The smaller number would then be 28 – 21 = 7.
144k1 + 1 = 17k2 LCM of 28 and 7 is 28. Option (B) says the larger num-
8k + 1 ber is 35. Since 196 is not a multiple of 35, option (B)
8k1 + 1 = k2
17 is ruled out. Option (C) says the larger number is 42;
The least value of k2 (17) is realized when k1 = 2. smaller number would then be 42 – 21 = 21
\ The least value of N = 289. LCM of 42 and 21 is 42. Hence not possible.
N must be of the form 289 + k LCM (144, 17) Option (D) says the larger number is 49. Smaller num-
= 2448k + 289, where k is a whole number. ber would then be 49 – 21 = 28
\ 2448k + 289 < 10000 LCM of 49 and 28 is 196
\ k can be 0, 1, 2 or 3 i.e., it has 4 possible values. Hence, the correct option is (D).
Hence, the correct option is (C). 37. Find the units digit of the sum of the factorials of the
first 100 natural numbers.
35. Four blocks of chocolates of weights (A) 1 (B) 5 (C) 3 (D) 7
1 1 3 15
6 kg, 10 kg, 8 kg, and 3 kg, respectively, were Sol. Units digit of the factorial of any natural number which
8 2 4 16
is 5 or more is 0. Required units digit = units digit of 1!
bought for a birthday party. The blocks were divided
+ 2! + 3! + 4! = 1 + 2 + 6 + 4 = 3.
into pieces such that all the pieces are of the same
weight. What is the least number of pieces that can be Hence, the correct option is (C).
obtained? 38. What is the remainder of (253 + 273 + 293 + 313) divided
(A) 61 (B) 63 (C) 66 (D) 67 by 112?
Sol. Weight of each piece (in kg) (A) 84 (B) 56 (C) 28 (D) 0
⎛ 1 1 3 15 ⎞ Sol. a + b = (a + b) (a + b – ab). Also 25 + 31 = 27 + 29
3 3 2 2

= HCF ⎜ 6 ,10 , 8 , 3 ⎟
⎝ 8 2 4 16 ⎠ = 56 and 112 = 56 (2)
25 + 313 = 56 [252 + 312 – 25 × (31)] = 56 (an odd
3
⎛ 49 21 35 63 ⎞ HCF ( 44, 21, 35, 63) 7 number)
= HCF ⎜ , , , ⎟ = =
⎝ 8 2 4 16 ⎠ LCM (8, 2, 4,16) 16
Similarly 273 + 293 = 56 (an odd number)
49 21 35 63 253 + 313 + 273 + 293 = 56 (an even number)
+ + +
Number of pieces obtained = 8 2 4 16 = 67 = A number divisible by 112.
7
16 \ Remainder is 0.
Hence, the correct option is (D). Hence, the correct option is (D).
36. The LCM of two numbers is 196 and the HCF is 7. If 39. Find the remainder when 2168 is divided by 105.
the difference of the two numbers is 21, find the larger (A) 7 (B) 15 (C) 3 (D) 1
of the two numbers. Sol. 105 = 3 (5) (7)
(A) 28 (B) 35 (C) 42 (D) 49 ⎛ 2168 ⎞ ⎛ (3 − 1)168 ⎞
Soi. Since the HCF of the two numbers is 7, we have 7x and Rem ⎜ ⎟ = Rem ⎜ ⎟
⎝ 3 ⎠ ⎝ 3 ⎠
7y as the two numbers where x and y are co-primes.
7x – 7y = 7(x – y) = 21, x – y = 21/7 = 3, x = y + 3 ⎛ ( −1)168 ⎞
= Rem ⎜ ⎟ = 1 (13)
The LCM of the two numbers is 7xy = 196 ⎝ 3 ⎠
 xy = 196/7 = 28 ⎛ 2168 ⎞ ⎛ 484 ⎞ ⎛ (5 − 1)84 ⎞
⇒  (y + 3)y = 28 Rem ⎜ ⎟ = Rem ⎜ ⎟ = Rem ⎜ ⎟ = 1(14)
⎝ 5 ⎠ ⎝ 5 ⎠ ⎝ 5 ⎠
⇒ y2 + 3y = 28
Remainders of 2N divided by 7 have a cycle of 3.
y + 3y – 28 = 0
2

⎛ 2168 ⎞ ⎛ 2 56 × 3 ⎞ ⎛ 23 ⎞
⇒ (y + 7) (y – 4) = 0 \ Rem ⎜ ⎟ = Rem ⎜⎜ ⎟⎟ = Rem ⎜ ⎟ = 1 (15)
⎝ 7 ⎠ ⎝3 ⎠ ⎝ 7⎠

Unit I_Chapter 3.indd 49 27/04/2017 14:18:55


1.50  |  Unit 1  •  Quantitative Aptitude

From (13), (14), (15), 2168 - 1 is divisible by 3, 5, 7 and Note: 1051 = 251 × 551. Remainders of 2N divided by 7
hence by their L.C.M i.e., 105. have a cycle of 3.
⎛ 2168 ⎞ ⎛ 251 ⎞
\ Rem ⎜ ⎟=1 \ Rem ⎜ ⎟ can be found (16)
⎝ 105 ⎠ ⎝ 7 ⎠
Hence, the correct option is (D).
⎛ 551 ⎞ ⎛ (53 )17 ⎞ ⎛ (18 (7) − 1)17 ⎞
40. N = 1051 – 750 Rem ⎜ ⎟ = Rem ⎜ ⎟ = Rem ⎜ ⎟
Consider the following statements. ⎝ 7 ⎠ ⎝ 7 ⎠ ⎝ 7 ⎠
  I.  The remainder of N when divided by 11 is 8. = (-1)17 = -1(17)
II.  The remainder of N when divided by 7 is 5. ⎛N ⎞
Which of the following can be concluded? From (16) and (17), Rem ⎜ ⎟ can be found.
⎝7⎠
(A) Only I is true
41. Find the remainder when the 300-digit number
(B) Only II is true
112222333333 … is divided by 8.
(C) Both I and II are true
(A) 1 (B) 3 (C) 5 (D) 6
(D) Neither I nor II is true
Sol. We can define the following concepts to describe the
Sol. N = 1051 - 750
300-digit number, say N.
I. The remainder of 10n, where n is any odd number,
when divided by 11 is always 10. Number of
A more general statement is that, if n is odd, the remain- units in the No.of digits Cummulative
Block block in the block no of digits
der of An divided by A + 1 is always A.
⎛N ⎞ 11 2 2
Rem ⎜ ⎟ 2222 4 4 6
⎝ 11 ⎠
⎛ (11k + 10) − 11( 68) + 2 ⎞
= Rem ⎜ ⎟ =8 9….9 18 18 90
⎝ 11 ⎠
I is true. 1010…10 20 40
1111…11 22 44
II. 1051 = (7 + 3)51 = (7 + 3) multiplied 51 times. 1212…12 24 48
(7 + 3) (7 + 3) = M(7), where M(7) denotes an unspeci- 1313…13 26 52 274
fied multiple of 7 + 32 14……14 13 26 300
(7 + 3)3 = (7 + 3) M(7)+ 32) \The last 3 digits of N are 414 and
= A multiple of M(7) + 33.
N 414
It follows in general that (7 + 3)N = M(7) + 3N Rem = Rem =6
8 8
1051 = M(7) + 351.
Hence, the correct option is (D).
⎛ 1051 ⎞ ⎛ 351 ⎞ ⎛ (33 )17 ⎞
Rem ⎜ ⎟ = Rem ⎜ ⎟ = Rem ⎜ ⎟ 42. N = 1613 – 773 – 843, which of the following statements
⎝ 7 ⎠ ⎝ 7 ⎠ ⎝ 7 ⎠ is not true?
⎛ 2717 ⎞ ( 28 − 1)17 (A) N is divisible by 4 and 23.
= Rem ⎜ ⎟ = Rem (B) N is divisible by 23 and 11.
⎝ 7 ⎠ 7
(C) N is divisible by 4 and 7.
⎛ ( −1)17 ⎞ (D) N is divisible by 8 and 11.
= Rem ⎜ ⎟ = -1
⎝ 7 ⎠ Sol. As (a + b)3 - a3 - b3 = 3ab(a + b),
\ 2717 is 1 less than a multiple of 7, (or 6 more than a N = 1613 - 773 - 843 = 3(77)(84)(161)
multiple of 7) while 750 = 7 (107) + 1 = 32(7)3(11)(4)(23)
⎛N ⎞ ⎡ ( M (7) + 6 ) − ( 7(10 7) + 1) ⎤ \ N is divisible by 4, 23, 11, 7 but not by 8. Choice (D)
\ Rem ⎜ ⎟ = Rem ⎢ ⎥
⎝7⎠ 7 is false.
⎣ ⎦
Hence, the correct option is (D).
⎛5⎞
= Rem ⎜ ⎟ = 5. 43. What is the remainder when 98100 + 100100 is divided
⎝7⎠ by 99?
II is true. (A) 2 (B) 0 (C) 1 (D) 98
( 99 − 1)
100
Both I and II are true. 98100
Sol. Rem = Rem
Hence, the correct option is (C). 99 99

Unit I_Chapter 3.indd 50 27/04/2017 14:18:57


Chapter 3  •  Numbers  |  1.51

i.e. a = b = c or both.
99k + ( −1) 100
Hence, the correct option is (D).
= Rem =1
99 4 7. (111)2 + (222)3 + (333)4 + (444)5 + (555)6 + (666)7 =
100100 1100 (A) (999)10 (B) (777)8
Similarly, Rem = Rem =1
99 99 (C) (777)10 (D) (888)9
98100 + 100100 Sol. (111)2 + (222)3 + … + (666)7
∴ Rem =2
99 = (111)2 = 23 - 1, (222)3 = 33 - 1, …
Hence, the correct option is (A). (666)7 = 73 - 1
44. What is the remainder when 2123 is divided by 61? \ The given sum = 23 - 1 + 33 - 1 + 43 - 1 + … + 73 - 1
(A) 1 (B) 2 13 + 23 + 33 + - - - - 73 - 1 - 1 - 1 + … - 1 for 7 times
(C) 8 (D) 60 ⎛ 7 ( 7 + 1)2 ⎞
2123 ⎜ ⎟ - 7 = 784 - 7 = 777 = (777)10
Sol. We need Rem . As the index is close to a multi- ⎜ 2 ⎟
61 ⎝ ⎠
ple of the divisor which is prime, we think of Fermat’s Hence, the correct option is (C).
Little theorem
48. The square root of the hexa-decimal number 310 is
260 2120 (A) (3C)16 (B) (2C)16 (C) (1C)16 (D) (C1)16
Rem = 1 ⇒ Rem =1
61 61 Sol. (310)16 = 0 + 1 × 16 + 3 × 162
= 0 + 16 + 768 = 784
2123 23
\ Rem = Rem = 8. The square root of 784 is 28.
61 61
16 28
Hence, the correct option is (C).
1 – 12
45. A three-digit number N leaves the same remainder
28 = (1C)16
upon dividing 68488 and 67516. How many possible
values does N have? \ The square root of (310)16 = (1C)16
(A) 8 (B) 6 (C) 5 (D) 4 Hence, the correct option is (C).
Sol. Let the number be N. 49. The numbers (11)7, (55)7, and (404)7 are in
68488 = N.k1 + R and (A) arithmetic progression
67516 = Nk2 + R where k1 and k2 are natural numbers (B) geometric progression
and R is the remainder 68488 - 67515 = N(k1 - k2). (C) harmonic progression
(D) arithmetic geometric progression
972 = N(k1 - k2)
Sol. (11)7 = 1 + 7 = 8
N must be a factor of 972.
(55)7 = 5 + 35 = 40
972 = 1 × 972 = 2 × 480 = 3 × 324
(404)7 = 4 + 0 × 7 + 4 × 72 = 200
= 4 × 243 = 6 × 162 = 9 × 108
\ 8, 40 and 200 are in G.P
\ N has 6 possibilities.
Hence, the correct option is (B).
Hence, the correct option is (B).
46. If a3 + b3 + c3 = 3abc, then 50. If the geometric mean of the numbers (24)6 and (34)7 is
(A) a+b+c=0 (24)n, then n =
(B) a=b=c (A) 6 (B) 7 (C) 10 (D) 8
(C) Both (A) and (B) Sol. Given
(D) At least one of (A) and (B) (24)6 = 4 + 2 × 6 = 16
Sol. a3 + b3 + c3 = 3abc (34)7 = 4 + 3 × 7 = 25
a3 + b3 + c3 – 3abc = 0 The geometric mean of 16 and 25 is 16 × 25 = 20
(a + b + c) (a2 + b2 + c2 – ab – bc – ca) = 0 Given, geometric mean = (24)n

(a + b + c)
(( a − b ) 2
+ (b − c) + (a − c)
2 2
)=0 (20)10 = (24)n
20 = 4 + 2n ⇒ 2n = 16 ⇒ n = 8
2
Hence, the correct option is (D).
a + b + c = 0 or a – b = b – c = c – a = 0

Unit I_Chapter 3.indd 51 27/04/2017 14:18:59


Chapter 4 Percentage, Profit and Loss
Hints/Solutions
Practice Problems I 5
s = 35
1. Let the value of the property be `x 4
s = 28.
The value of the part that Ganesh owns (in `)
Hence, the correct option is (D).
1
83 6. Let the price in 2003 be 100.
= 3 x×= 5x
100 6 Price in 2004 = 110
3⎛5 ⎞ With respect to the price in 2004, its price is less by
Three fourths of this part is worth ` ⎜ x ⎟
4 ⎝6 ⎠ 110 − 100 1
5 × 100 = 9 %.
i.e., ` x 110 11
8
5 Hence, the correct option is (B).
x = 5,  x = 8
8 7. Let the initial cost of house hold items be x
Hence, the correct option is (B). 100 + 20
Present cost = x = 1.2x.
2. As the salaries of the two persons are equal and since 100
the percentage increase in the salary for one person Let the initial salary of family be y.
equals the percentage decrease in the salary for the 100 + 10
Present salary of family = y = 1 × 1y
other, the increase in the salary and the decrease in the 100
salary must be equal. Initially 0 × 2y = x
\ the total salary of the 2 persons does not change. ⇒ 1 × 2x = 0 × 24y
Hence, the correct option is (C). Percentage of present salary being spent on household
3. Let the original price be `x 0 ⋅ 24 y 240 9
Decrease in the price = `0 × 2x items = × 100 = = 21 %.
1 ⋅1 y 11 11
Price after the decrease = `0 × 8x Hence, the correct option is (D).
0.2 x
% increase = (100) = 25% 8. Let the length of the rectangle be ‘l’ and its breadth
0.8 x be ‘b’
Alternate method: Area = lb
Any quantity which decreases by x% must be increased New Area = A1 = 1 × 32A = 1 × 32lb
100 x New breadth = 1 × 1b
by % to become its original value
100 − x
1 ⋅ 32lb
100 x ⇒ New length = = 1 × 2l.
As x = 20, = 25 1 ⋅1b
100 − x Perimeter = 2(l + b)
Hence, the correct option is (A).
New perimeter = 2(1 × 2l + 1 × lb)
5 2
4. Ratio = : l and b are unknown
6 3
Hence answer is cannot be determined.
Let the first number be 5x,
Hence, the correct option is (D).
Second number is 4x
9. Let B’s salary be x.
5x − 4 x
% less = × 100 = 20%
5x 100 − 20
A’s salary = × x = 0 × 8x
Hence, the correct option is (A). 100
5. Let the marks of Mohan and Sohan be m and s C’s salary = 10000 and it is also 25% more than B’s
respectively. 100 + 25
salary ⇒ × x = 10000.
⎛ 25 ⎞ 5 100
m = s ⎜1 + ⎟ = s.
⎝ 100 ⎠ 4 x = `8000.
m = minimum mark required to pass A’s salary = 0 × 8x = `6400
i.e. pass mark = 35. Hence, the correct option is (D).

Unit I_Chapter 4.indd 52 27/04/2017 14:19:09


Chapter 4  •  Percentage, Profit and Loss  |  1.53

10. Let the numbers of students in school B = x 16. Let the cost price of the item be `x
Number of students in school A = 1 × 3x Profit = `0 × 2x
Given 1.3 x – x = 0 × 3x = 120 x + 0 × 2x = 60 ⇒ x = 50
⇒ x = 400 To gain 30% profit must be `0 × 3x
Total number of students in both the schools \ selling price must be `1 × 3x = `65
= (1 + 1 × 3)x = (2 × 3) × 400 = 920. Hence, the correct option is (B).
Hence, the correct option is (D). 17. Let the cost price of each in be `x
11. Let the initial price be `100 Profit made on selling 5 m = cost price of 2 m = `2x
⎛ 2 ⎞ 4 ⎛ 2 ⎞ Cost price of 5 m = `5x
Final price = ⎜100 + (100) ⎟ + ⎜ 100 + (100) ⎟
⎝ 100 ⎠ 100 ⎝ 100 ⎠ 2
Profit percentage = (100)% = 40%
= `106.08 5
Hence, the correct option is (C).
\ Percentage increase = 6.08%
60 − 50
Hence, the correct option is (B). 18. Required percentage = (100 ) % = 20%
50
12. Let the pass mark be x.
Hence, the correct option is (C).
25
Rahul’s mark = x + x =150 19. Total cost price (80) (12 × 50) = `1000
100
Total profit = (80) (4 × 50) = `360
x = 120 Total selling price = `1360
165.120
Required percent = (100) = 37.5% Total selling price of 20 articles = `360
120
Hence, the correct option is (B). Selling price of each of the remaining articles
30 1360 − 360 2
13. Number of girls = (1000) = 300 = = ` 16
100 60 3
Hence, the correct option is (B).
Number of boys = 1000 - 300 = 700
20. Let his C.P. = `100
Number of UGs = 1000 - 600 = 400
Let the number of female UGs be f. Number of male ⎛ 40 ⎞
M.P. (in `) = 100 ⎜1 + ⎟ = 140
UGs = 400 - f ⎝ 100 ⎠
⇒ 400 - f - f = 120
12
⇒ f = 140 profit in (`) = (100) = 12
140 100
Required percent = (100) = 20% S.P. = `112
700
discount = `28.
Hence, the correct option is (A). discount (∵ Discount = M.P. – S.P.)
14. It can be seen that it depreciates by (18 - 1.5(n - 1)) in 28
the nth year.   = (100) % = 20%
140
\ In the seventh year it depreciates by 9%. Hence, the correct option is (A).
\ required amount = 9% (4,00,000) = 36,000 21. Let the M.P. be `100
Hence, the correct option is (C). 30
First discount (in `) = (100) = 30
15. Let the monthly income of Ram be `100 initially. 100
Price after this discount = `70
Initial expenditure of Ram = `70.
10
Initial Saving of Ram = `30 Second discount (in `) = (70) = 7.
100
Final monthly income of Ram = `126 Price after this discount = `63
Final expenditure of Ram = `84 Total discount = `37
Final savings of Ram = `42 Equivalent single discount percentage = 37%
Percentage increase his savings = 40% Hence, the correct option is (C).
Hence, the correct option is (A). 22. Ratio of profits of P and Q = Ratio of the investments
of P and Q = 1 : 2

Unit I_Chapter 4.indd 53 27/04/2017 14:19:11


1.54  |  Unit 1  •  Quantitative Aptitude

2 Now, undiscounted price of 25 VCDs = price marked


Q’s profit = (9600) =` 6400
3 by Samuel for 25 VCDs = 2500
Hence, the correct option is (B). So, profit on 100 VCDs = 2500
23. Ratio of profits of Ramesh and Suresh 2500
Percentage of profit = × 100 = 33 × 33%.
= (9000) (10) : (6000) (12) = 5 : 4 7500
4 Hence, the correct option is (D).
Suresh’s share = ( 4500 ) =` 2000
9 28. Let his profit / loss percentage be 3x%.
Hence, the correct option is (D). His discount percentage = 2x%
24. Number of articles sold = 20% of 50 = 10 ⎛ 3x ⎞ ⎛ 2x ⎞
His selling price = 200 ⎜1 + ⎟ = 300 ⎜1 − 100 ⎟
Cost of 5 articles = `1200 ⎝ 100 ⎠ ⎝ ⎠
Cost of 1 article = `240
⎛ 3x ⎞ ⎛ 2x ⎞
Cost of 10 articles = `2400 If we made a profit, 200 ⎜1 + ⎟ = 300 ⎜1 − ⎟
⎝ 100 ⎠ ⎝ 100 ⎠
Selling price of 10 articles = 2400 + 1200 = `3600 25
x =
Selling price of 1 article = `360 3
Number of articles remaining = 40. Profit percentage = 25
Total value of the remaining articles at selling price ⎛ 3x ⎞ ⎛ 2x ⎞
= 40 × 360 = 14400. If he made a loss 200 ⎜1 − ⎟ = 300 ⎜1 − ⎟
⎝ 100 ⎠ ⎝ 100 ⎠
Hence, the correct option is (A). 200 - 6x = 300 - 6x which is not possible.
25. Let the marked price be `x. \ He made 25% profit.
Let the cost prices be `100
Hence, the correct option is (B).
Profit = `20
29. Let the cost price to Feroze be `100
Selling Price = `120
Feroze marks up the price to `130
Discount = `0.2x
He sells it at 20% discount; at `104
Given, x - 0.2x = 120 ⇒ x = 150
\ The marked price is 50% alone the cost price. Sohail marks up the price to 20% more than `100,
equal to `120
Hence, the correct option is (B).
The least sale price = 104
26. Let the cost of a car be 100x.
After a discount of 100000, the profit = 15% To sell it at `104, percentage discount to be offered
⇒ Selling price of the car = 115x, and 16 40 1
= × 100 = = 13 %.
Marked price of the car = 115x + 100000 120 3 3
Given that the M.P of the car is 25% more than the C.P Hence, the correct option is (D).
of the car ⇒ 125x = 115x + 100000 30. Ratio of the profit shares of Gopal, Hari and Karthik
⇒ 10x = 100000, x = 10000 = (8000) (112) : (12000) (x) : (16000) (x)
Marked price of the car = 115x + 100000 = 1250000. = 96 : 12x : 16x
Hence, the correct option is (D). 16x > 96 > 12x
27. For Samuel \ x > 6 and x < 8
Let’s assume the marked price of each VCD is `100 As x is an integer, x = 7
For Simon \ Required ratio = 24 : 21 : 28
Cost price of each VCD = 0 × 75 × 100 = 75 Hence, the correct option is (B).
Cost price of 100 VCDs = 7500

Practice Problems 2
2
Solutions for questions 1 to 15: (A) 16 % (B) 20%
3
1. Ravi’s salary before he got an increment was 20% of
the total income of his family. His increment was one- 1
(C) 33 % (D) 25%
fourth of his salary after the increment. What percent- 3
age of the total income of his family is his new salary?

Unit I_Chapter 4.indd 54 27/04/2017 14:19:12


Chapter 4  •  Percentage, Profit and Loss  |  1.55

Sol. Let Ravi’s salary after the increment be 4x x = 2,50,000


\ Before the increment it was 3x and the total fam- Number of votes polled = 5.4x - 3,50,000 = 10,00,000
ily income was 15x. After Ravi’s increment, the family Hence, the correct option is (B).
income is 16x.
5. The publisher of a novel, published the novel in six vol-
Required percentage umes numbered I, II, III, … VI. The number of pages in
Ravi’s new salary each volume is 25% more than that in the previous vol-
= (100%) ume. If the number of pages in volume V of the book is
Total family income
500 more than the number of pages in volume IV, how
4x many pages were there in volume II of the book?
= (100%) = 25%.
16 x (A) 480 (B) 960 (C) 640 (D) 1280
Hence, the correct option is (D). Sol. The volume number and the corresponding number of
2. The price of fan A is twice that of another fan B. The pages are tabulated below.
price of A is increased by 10% and that of B is decreased
Volume I II III IV V VI
by 20%. Find the percentage decrease in the sum of the
prices of the fans. Pages 4x 44(5)x 43(52)x 42(53)x 4(54)x 55x
(A) 10% (B) 0% We see that the number of pages in V is 500x more than
(C) 15% (D) 5% in VI 500x = 500 ⇒ x = 1 and the number of pages in II
Sol. Let the price of fan B initially be `x is 1280.
Price of fan A initially = `2x Hence, the correct option is (D).
10 6. In 2000, the market shares of the toilet soaps Margo,
Increase in the price of A = (2x ) = ` 0 ⋅ 2x Palmolive, and Dove were 40%, 30%, and 30%, respec-
100
tively. Starting from the next year, a new soap enters the
20 market each year and gets 10% of total market share.
Decrease in the price of B = ( x ) = 0.2 x The existing soaps share the remaining market share in
100
As the increase equals the decrease, the sum of the the same ratio as they did in the previous year. What per-
prices does not change. cent of the total market share will Margo have in 2002?
(A) 32% (B) 32.4% (C) 28.8% (D) 34%
\ required percentage change = 0%
Sol. Ratio of market shares of Margo, Palmolive, Dove and
Hence, the correct option is (B).
the new soap in 2001 = 0.9(40) : 0.9(30) : 0.9(30) : 10
3. The length of a rectangle is increased by 20% and its
= 36 : 27 : 27 : 10
breadth is increased by 10%. Which of the following
36
is a possible value of the percentage increase in its Market share of Margo in 2002 = (90) = 32.4%
perimeter? 100
(A) 13% (B) 14% Hence, the correct option is (B).
(C) 12% (D) 16% 7. In 2003, Brijesh paid a tax of 20% of his salary. In
Sol. Let the length and the breadth be  and b respectively. 3
2004, his salary increased by 93 % and the tax
 ≥ b. 4
scheme changed. Under the new tax scheme, he had to
If  = b, percentage increase in the perimeter is 15%. pay a fixed sum of `1000 and an additional 20% on the
Also if  > b, percentage increase is more than 15%. amount above `10000. His salary in 2004 was more
\ Percentage increase can be 16%. than `10,000 and he paid a tax of `500 more than what
he paid in 2003. What is his salary in 2004?
Hence, the correct option is (D).
(A) `15500 (B) `17000
4. In an election among three contestants P, Q, and R, P (C) `18200 (D) `20000
gets 120% more votes than Q. P beats R by 3,50,000
Sol. Let the salary in 2003 be x.
votes. Q beats R by 5% of the total votes. Find the total
number of votes polled (in lakhs). The tax paid by him in 2003 is 0 × 2x
(A) 12 (B) 10 (C) 9 (D) 11 ⎛ 93 ⋅ 75 ⎞
Salary in 2004 = x ⎜1 + = 1 × 9375x
Sol. Let the number of votes Q gets be x ⎝ 100 ⎟⎠
Number of votes P gets = 2.2x This salary > 10000
Number of votes R gets = 2.2x - 3,50,000 Given tax difference = 500
5 (1 × 9375x - 10000) 0 × 2 + 1000 - 0 × 2x = 500
x - 2.2x - 3,50,000 = ( x + 2.2 x + 2.2 x − 3, 50, 000)
100 (1 × 9375) (0 × 2)x - 1000 - 0 × 2x = 500.

Unit I_Chapter 4.indd 55 27/04/2017 14:19:13


1.56  |  Unit 1  •  Quantitative Aptitude

0 × 1875x = 1500 The above implies that for some value of sales amount
x = 8000. between `100000 and `150000, the incomes from both
the schemes is the same.
Salary in 2004 = 1 × 9375x = `15500.
Let the commission for a sale amount of ` (100000 + x)
Hence, the correct option is (A).
be equal.
8. The production of rice in the year 2001 was 1000
⇒ commission on `(100000 + x) under scheme 1
tonnes, which was 25% of the total food grain pro-
duction in that year. In the next year if the production = commission on `(100000 + x) under scheme 2
of rice decreased by 4% and production of rice as a ⇒ 4700 + 2% of x = 3500 + 5% of x
percentage of total food grain production increased by ⇒ 1200 = 3% of x,
5 percentage points, what is the total food grain pro-
⇒ 1% of x = 400 or x = 40000
duction in 2002?
(A) 4020 tonnes (B) 3200 tonnes Hence, 100000 + x = 140000.
(C) 3800 tonnes (D) 3540 tonnes Hence, the correct option is (A).
Sol. Total food grain production in 10. At the beginning of a year, the owner of a jewel shop
100 raised the prices of all the jewels in his shop by x% and
2001= × 1000 = 4000 the lowered then by x%. The price of one jewel after
25
this up–down cycle reduced by `100. The owner car-
⎛ 100 − 4 ⎞ ried out the same procedure after a month. After this
Production of rice in 2002 = ⎜ ⎟ × 1000 = 960t
⎝ 100 ⎠ second up–down cycle, the price of that jewel was
Rice production as a percentage of food grain produc- `2304. Find the original price of that jewel (in `).
tion, in 2002 = 25 + 5 = 30%. (A) 2600 (B) 2550 (C) 2650 (D) 2500
\ Total food grain production in 2004 Sol. Let the original price of that jewel be `P.
100 ⎛ x ⎞ ⎛ x ⎞
= × 960 = 3200. P ⎜1 +
30 ⎟ ⎜1 − 100 ⎟ = P −100
⎝ 100 ⎠ ⎝ ⎠
Hence, the correct option is (B).
⎛ x ⎞⎛ x ⎞ P −100
9. A salesman has to choose between two schemes of ⇒ ⎜1 + ⎟ ⎜ 1− ⎟= P
remuneration. The first scheme has a fixed salary of ⎝ 100 ⎠ ⎝ 100 ⎠
`3700 and a commission of 2% on sales above `50000. ⎛ x ⎞⎛ x ⎞
The second scheme has no salary but offers commis- ⇒ ( P −100 ) ⎜1 + ⎟ ⎜ 1− ⎟ = 2304
⎝ 100 ⎠ ⎝ 100 ⎠
sion only. The commission, starting from 3% of sales
for the first `50000 or part there of, increases at the rate ⎛ P −100 ⎞
⇒ ( P −100 ) ⎜ ⎟ = 2304
of 1 percentage point for every increase of `50000 or ⎝ p ⎠
part there of sales, upto a maximum of 20% of sales.
⇒ P2 - 2504P + 10000 = 0
What is the minimum value of the sales above which
he can prefer the second scheme? (P - 2500) (P - 4) = 0
(A) `140000 (B) `90000 P must be greater than 2304 since each up down cycle
(C) `40000 (D) `240000 reduces the price effectively.
Sol. The commissions received under the two schemes, for var- \ P = 2500
ious levels of sales amounts can be tabulated as follows: ⎛ x ⎞⎛ x ⎞
Note: ( P −100 ) ⎜1 + ⎟ ⎜ 1− ⎟
Sales Commission Commission ⎝ 100 ⎠ ⎝ 100 ⎠
Amount (`) under scheme under scheme
( −100 ) ⎜⎛1 + ⎞⎟ ⎜⎛1 − ⎞⎟ −100 ⎜⎛1 + ⎞⎟ ⎜⎛1 − ⎞⎟
x x x x
1 (`) 2 (`) P
⎝ 100 ⎠ ⎝ 100 ⎠ ⎝ 100 ⎠ ⎝ 100 ⎠
1. First 50000 3700 3% of 50000
= 1500 = P - 100 - (some number which is less than 100)
2. Second 50000 2% of 50000 4% of 50000 = Some number which is more than P - 200)
= 1000 = 2000 \ 2304 > P - 200
3. Fist 100000 3700 + 1000 1500 + 2000 2504 > P
= 4700 = 3500
Only Choice (D) satisfies this condition.
4. Third 50000 2% of 50000 5% of 50000 Hence, the correct option is (D).
= 1000 = 2500
11. In a public sector unit (PSU), there are 45600 employ-
5. First 150000 4700 + 1000 3500 + 2500
= 5700 = 6000
ees. When the PSU offered a voluntary retirement
scheme (VRS), 40% of the employees applied for the

Unit I_Chapter 4.indd 56 27/04/2017 14:19:14


Chapter 4  •  Percentage, Profit and Loss  |  1.57

VRS. After scrutinizing, the PSU has rejected 15% of 50


Area after the second cut = 0.6 x − x (0.6 x ) = 0.3 x
the applications. But only 9120 employees took the 100
retirement through the scheme. What percentage of the Given, 0.6x - 0.3x = 30 ⇒ x = 100
total number of employees did not take retirement even
Hence, the correct option is (B).
though their applications are not rejected?
(A) 25% (B) 14% 14. A new coach was appointed in the middle of a season
(C) 24% (D) 12.75% for a football team. After he took over, the team won
Sol. From the given information, the percentage of employ- 80% of the 60 matches it played. But the overall per-
ees who took retirement through the scheme formed of the team was only 60%. Find the minimum
number of matches the team must have played that sea-
9120
= × 100 = 20%. son before the new coach took over.
45600 (A) 10 (B) 15
Now by taking 100 as the base, we can calculate the (C) 20 (D) 25
answer easily. Sol. Let the minimum number of matches be x. Let us say,
Out of 100 employees, 40 applied for VRS. of these x matches y matches were won. Number of
But PSU allowed only 85% of 40 i.e. 34 employees to 80
matches won after the coach took over = (60) = 48 .
take VRS. 100
\ 34% – 20% = 14%
48 + y
i.e., 14% of the total employees did not take retirement, Given, (100) = 60 ⇒ 48 + y = 0.6(60 + x)
although their applications are not rejected. 60 + x
Hence, the correct option is (B). ⇒ x would be minimum when y = 0.
12. One month Mrs. and Mr. Rai take home `20000 each. In this case, x = 20
These amount represent an increase of 25% over Hence, the correct option is (C).
Mrs. Rai’s take home in the previous month and a
decrease of 33.33% over Mr. Rai’s take home in the 15. A motorist used 10% of his fuel to cover 20% of his
previous month, respectively. What was their total total journey. He covered another 40% of his total jour-
income in the previous month? ney under similar conditions. For the rest of journey,
(A) `40000 (B) `50000 the conditions were different. Find the maximum per-
(C) `46000 (D) `41666 centage by which his fuel efficiency (distance covered
per unit quantity of fuel) can drop, so that he can still
Sol. Present salary of Mrs. Rai = 20000 cover the remaining journey without a refill.
This is a gain of 25% from last month’s earnings. 3 6
(A) 71 % (B) 72 %
⇒ Last month’s salary of Mrs. Rai. 7 7
100 1 2
= × 20.000 = 16000 (C)
27 % (D)
74 %
125 7 7
Mr. Rai’s salary has come down to `20000. This repre-
sents a 33 × 33% drop. Sol. Let the total distance be 100 km. Let the total fuel
100 be 10 litres. As 10% of the fuel was used to cover
Mr. Rai’s salary last month = × 20000
66 ⋅ 66 20% of his total journey driving conditions, a litre is
= 30000 required for driving every 20 km (under given driving
conditions).
Total income of Mr. Rai and Mrs. Rai last month
\ Total fuel required for conditions which were
= 30000 + 16000 = 46000.
­different = Fuel required for the 60% of the journey
Hence, the correct option is (C).
60
13. Anand cut a rectangular piece of paper twice. With the (100)
100 = 3 litres.
first cut, he decreased its area by 40%. With the second =
20
cut, he decreased its area by 50%. Its final area was
30 sq cm less than its area after the first cut. Find its \ 7 litres is required for driving the balance 40 km.
original area (in sq cm). 40
20 −
(A) 120 (B) 100 (C) 90 (D) 80 7 = 71 3 %
Required percent =
Sol. Let its original area be x sq cm. 20 7
40 Hence, the correct option is (A).
Area after the first cut = x - x = 0.6 x
100

Unit I_Chapter 4.indd 57 27/04/2017 14:19:16


1.58  |  Unit 1  •  Quantitative Aptitude

Solutions for questions 16 and 17: 19. A man bought 100 mangoes at a certain price, with the
In an exam, every question correctly answered fetches intention of selling each at a profit of 25%. But 20 man-
2 marks. Every question wrongly answered loses 1 mark. goes got spoilt. If he sold the rest at the intended price,
Unanswered questions have no marks associated with them. what was his profit or loss percentage?
Ram and Shyam wrote this exam. Ram attempted a certain (A) 0% (B) 6.66% profit
number of questions and 30% of them went wrong. Shyam (C) 6.25% loss (D) 12.5% profit
attempted a certain number of questions and 40% of them Sol. Let the initial selling price of each mango be `x.
went wrong. Ram got 40 marks more than the pass mark.
\ New selling price = `1.25x.
Shyam got 25 marks more than the pass mark. The two of
them together attempted a total of 100 questions. Overall selling price now = 1.25x (80) = `100x.
16. Find the pass mark in the exam. \ As there was 25% overall profit on selling,
(A) 15 (B) 20 (C) 25 (D) 30 (100 x ) (100)
Overall cost price = = `80x.
Sol. Let the number of questions attempted by Ram be x. 125
Let the pass mark be p. If the remaining 80 apples are sold at `x, then the sell-
⎛ 70 x ⎞ 30 ing price = `80x.
Ram’s mark = 2 ⎜ ⎟ - 100 x =1.1x
⎝ 100 ⎠ \ Overall C.P. = Overall S.P., hence no profit or no loss.
⎛ 60 ⎞ Alternate method:
Shyam’s mark = 2 ⎜ ⎟
⎝ 100 ⎠ Let the cost price of 1 mango = `1
40 Cost Price of 100 mangoes = `100(5)
(100 - x) - (100 − x ) Profit percentage expected = 25%
100
⇒ 0.8(100 - x) 1.1x = p + 40 (3) \Proposed sale price per mango = `1.25(6)
⇒ 0.8(100 - x) = p + 25  (4) 20 mangoes are rotten; 80 mangoes are available for
Solving (3) and (4), x = 50 and p = 15 sale.
Hence, the correct option is (A). If these mangoes are sold at the original sale price of
17. If there are 80 questions in the exam, find the percent- `1.25 per mango, the money received from the sale of
age of marks secured by Ram. 80 mangoes
(A) 40.625% (B) 45.75% = 80 (1.25) = `100(7)
(C) 50.75% (D) 34.375% As (5) and (7) are equal, there is neither profit nor loss.
Sol. Percentage secured by Ram Hence, the correct option is (A).
(1.1 ) (50) 1.1(50) Note: Information that sale price is increased by 25%
= (100)% = (100)% = 34.375%
80( 2) 80( 2) is redundant because, no mangoes are sold of this price.
Hence, the correct option is (D). 20. A trader sells an article at a profit of `25. If the cost
Solutions for questions 18 to 30: price is reduced by `25 and consequently the selling
price is reduced by 25% he would make a profit of
18. The cost of production of a motorbike which is sold at 25%. What is his initial cost price?
20% profit went up by 40%. What should be the percent- (A) `100 (B) `50 (C) `75 (D) `40
age increase in the selling price to maintain the profit
percentage the same even at the new cost of production? Sol. Let the initial cost price be `x.
(A) 40% (B) 20% (C) 68% (D) 60% Initial S.P. = x + 25
Sol. Let the initial cost be `100. Now, the cost price is reduced by `25
It went up by 40% i.e., new cost price = `140 New C.P. = (x – 25)
Initial Selling Price = `120. Selling price is reduced by 25%.
(as the profit is 20%) 3
\ New S.P. = (x + 25) × .
Present Selling Price = 140(1 × 2) = `168 4
Percentage increase in selling price Profit = 25%
168 − 120 5 3
= × 100 \ (x - 25) × = (x + 25) ×
120 4 4
5x - 125 = 3x + 75
48
= = 40% 2x = 200 ⇒ x = 100.
1⋅ 2
Hence, the correct option is (A). Hence, the correct option is (A).

Unit I_Chapter 4.indd 58 27/04/2017 14:19:17


Chapter 4  •  Percentage, Profit and Loss  |  1.59

21. A dealer bought 50 television sets at `10000 each. For (A) 2% decrease (B) 2% increase
every set purchased from him, he gave one set free. The (C) 4% decrease (D) 4% increase
loss made by him is equal to the selling price of 15 sets. Sol. Let Mr. Singh’s initial income per hour be `100
What is the selling price of each set, that is bought?
Present income per hour = `120
(A) `10000 (B) `15000
(C) `12500 (D) `20000 Let the number of hours per month be 100, initially
Sol. Let the selling price of one television be x. Present number of hours = 80
1 Earlier monthly income = 100 × 100 = 10000
Total money made = (50x) = 25x
2 Present monthly income = 120 × 80 = 9600
(since one TV is given free with each one bought by
Percentage decrease in his income
customer.)
400
Given total cost = 10,000 × 50 = 500000 = × 100 = 4%.
10000
Loss = 15(x) = 15x Alternate method:
Selling price + Loss = Cost price Mr. Singh’s income (I ) is the product of his hourly rate
⇒ 25x + 15x = 500000 (R) and the number of hours he works (n).
40x = 500000 i.e., I = rn
6R
x = 12500 R increases by 20% to and
5
Selling price of each television = `12500.
4n
Hence, the correct option is (C). n decreases by 20% to .
5
22. A car manufacturing company sold a car to a show
room at 25% profit. The show room sold it to a cus- 6 R 4n
\ The income changes to = ×
tomer at the same profit as that at which the company 5 5
sold it. The customer sold it to another customer at a 24
loss percentage whose value equals that of the profit = I , i.e., it decreases by 4%.
25
percentage made by the showroom. The first customer
Hence, the correct option is (C).
sold it for `288000. Find the cost of manufacturing the
car (in `lakhs). 24. In a certain year, the wholesale price index fluctuated
(A) 1 (B) 1.2 (C) 2.4 (D) 0.72 as given below:
Sol. Let the cost of manufacturing be `x. Period Percentage of increase or decrease
x over the preceding period
Profit made by the company = ` . (a) 1st April to 30th April Increased by x%
4
x (b) 1 May to 31 May
st st
Decreased by x%
Profit made by the show room = ` .
4 (c) 1st June to 30th June Increased by x%

5x (d) 1st July to 31st July Decreased by x%


Cost price to the show room = ` .
4
Profit % to the show room = 20% If the decrease for the period 1st April to 31st May was
160 points and that for the period from 1st June to 31st
Loss% to the first customer = 20% July was 134.4 points, what was the price index on 1st
3x April? (in points)
Cost price of the first customer = `
2 (A) 800 (B) 900 (C) 1000 (D) 1200
3x 4 Sol. Let the whole sale price index on 1 April be points. As
st
Selling price of the first customer × i.e., per data,
2 5
12 x ⎛ x ⎞⎛ x ⎞
=` = 28800. \x = 240000 p - p ⎜1 + ⎟ ⎜ 1− ⎟ = 160
10 ⎝ 100 ⎠ ⎝ 100 ⎠
Hence, the correct option is (C). ⎛ x2 ⎞ ⎛ x2 ⎞
p - p ⎜1 − ⎟ = 160 p ⎜ ⎟ = 160  (8)
23. Mr. Singh is paid as per the number of hours he puts in ⎝ 10000 ⎠ ⎝ 10000 ⎠
per month. The rate of pay was increased by 20% per The second equation:
hour, but the number of hours put in by him went down
⎛ x2 ⎞ ⎛ x2 ⎞ ⎛ x2 ⎞
by 20%. What is the percentage increase/decrease in p ⎜1 − ⎟ − p ⎜1 − ⎟ ⎜1 − ⎟ = 134 × 4
his income? ⎝ 10, 000 ⎠ ⎝ 10, 000 ⎠ ⎝ 10, 000 ⎠

Unit I_Chapter 4.indd 59 27/04/2017 14:19:19


1.60  |  Unit 1  •  Quantitative Aptitude

⎛ x2 ⎞ x2 Sol. Let the cost of 10000 hard disks be `10000


p ⎜1 − ⎟⋅ = 134 × 4  (9) After a discount of 10%, cost = `9000
⎝ 10000 ⎠ 10000
20% of the hard disks are damaged.
⎛ x2 ⎞ ⇒ Number of hard disks to be sold = 8000
from (1), 160 ⎜1 − ⎟ = 134 × 4
⎝ 10000 ⎠ Selling price = 20% more than the price at which he
x2 bought
1− = 0×84 x = 40% Total sales = `(8000 × 1 × 2 × 0 × 9) = `8640
10000
\ from (8), p = 1000. 9000 − 8640
Loss percentage = ×100
Hence, the correct option is (C). 9000
25. Ravi manufactures watches. Each day he manufactures
360
as many watches as the cost price per watch (in `).  = × 100 = 4%.
Each day he sells all his watches at a profit of `60 per 9000
watch and at the end of the day his profit percentage is Alternate method:
10%. Find his daily profit (in `).
Ratio of CP of undamaged disks and damaged disks
(A) 36000 (B) 20000
= 80% : 20% = 4 : 1
(C) 40000 (D) 50000
Undamaged disks are sold at a price which is 20% more
Sol. Let his cost price be `x per watch
than the C.P.; ⇒ profit percentage on these = (+)20%
10 Damaged disks are to be taken as unsaleable;
Given, x = 60 ⇒ x = 600
100 ⇒ loss percentage on these = (–)100%
10 Overall percentage of profit/loss
⇒ Profit = [(600) (600)] = `36000
100 = [4 × (+20) + 1(–100)]/(4 + 1) = (–20/5) = –4
Hence, the correct option is (A). \ 4% loss
26. Ajay sold an article at 20% profit to Balu. Balu sold it Hence, the correct option is (D).
at 30% profit to Chetan. Dinesh sold a similar article at
Note:
20% loss to David. David sold it at 30% loss to Edward.
The sum of the price that Chetan and Edward paid for 1. Information about number of disks is redundant.
their respective articles is `28000 more than what Ajay 2. The statement about the discount (10%) received at
paid. If Ajay and Dinesh bought the article for the same the time of purchase, is also redundant.
price, Find the sum of the prices paid by Ajay and 28. P and Q started a business. They made an annual profit
Dinesh (in rupees) for their respective articles. of `50000. Q being a working partner received 20% of
(A) 50000 (B) 40000 the annual profit as his salary. If the entire profits were
(C) 45000 (D) 52000 divided in the ratio of their investments, P would have
Sol. Let the cost prices for Ajay and Dinesh be `a and `d received `8000 more as his profit share than what he
respectively. actually got. Find P’s actual profit share (in `).
Cost price of Balu = `1.560. (A) 33600 (B) 32400
(C) 32000 (D) 31600
\ Ajay’s article’s value increases by `0.56a.
Sol. Let the investments of P and Q be `p and `q respectively.
Cost price of David = `0.8d.
Salary of Q = `10000
Selling price of David = `0.56d
0.56a + 0.56d = 28000 p p
(50000) = ( 40000) + 8000
a + d = 50000 p+q p+q
Hence, the correct option is (A). p:q=4:1
4
27. Mr. Londa imported 10000 hard disks from Korea at Profit share of p = (40000) = `32000
5
a discount of 10% on the marked price. Out of these,
20% of the hard disks were damaged in transit. If the Hence, the correct option is (C).
selling price of an undamaged disk is 20% more than 29. A, B, C, and D started a business with investments in
its cost price, what is the profit/loss percentage on total the ratio 3 : 4 : 5 : 6. As B and C were working partners
sales? they were paid equal salaries. The ratio of B’s and C’s
(A) 6.66% profit (B) 10% profit total annual income is 9 : 10. If the total annual profit is
(C) 6.66% loss (D) 4% loss `84000, find B’s salary (in `).

Unit I_Chapter 4.indd 60 27/04/2017 14:19:20


Chapter 4  •  Percentage, Profit and Loss  |  1.61

(A) 9000 (B) 12000 Month Satish Sanjay Sunil


(C) 15000 (D) 18000
1 st
20 20 20
ol. Let the investments of A, B, C and D be `3x, `4x, `5x
S
and `6x respectively. Let the salary of B be `S. 2 nd
21 22 20
3rd 22 22 23
4x + S 9
Given = 4th 23 24 23
5 x + S 10
5 th
24 24 23
⇒ S = 5x 6 th
25 26 26
Total amount profit = 18x + 2S 7th 26 26 26
= 28x = 84000 ⇒ x = 3000 8 th
27 28 26
\ B’s salary is 5x i.e., 15000 9 th
28 28 29
Hence, the correct option is (C). 10th 29 30 29
30. Satish, Sanjay, and Sunil started a business with an 11 th
30 30 29
investment of `20000 each. At the start of each month
12 th
31 32 32
starting from the second, Satish adds `1000. At the
start of each even month starting from the second,
Sanjay adds `2000. At the start of the 3rd, 6th, 9th, and \ the ratio of their profit shares is
12th months, Sunil adds `3000. Who gets the greatest
⎛ 20 + 31 ⎞ ⎛ 22 + 30 ⎞
share out of the annual profit? ⎜ 2 ⎟12 : 2 ⎜ 2 ⎟
(A) Satish ⎝ ⎠ ⎝ ⎠
(B) Sanjay (5) + 20 + 32 : 20 + 20 + 32 + 3(3)( 26)
(C) Both Satish and Sunil
= 51(6) : 52(6) : 72 + 9(26) = 153 : 156 : 153
(D) Sunil
We see that Sanjay gets the greatest share.
Sol. The data is tabulated below. All the amounts are in
thousands of rupees. Hence, the correct option is (B).

Unit I_Chapter 4.indd 61 27/04/2017 14:19:21


Chapter 5 Simple Interest and Compound Interest
Hints/Solutions
Practice Problems I ⎡⎛ 10 ⎞
2

Interest earned = 5000 ⎢⎜1 + ⎟ − 1⎥ = `1050
Solutions for questions 1 to 20: ⎢⎣⎝ 100 ⎠ ⎥⎦
24000 ×18 × 5 Hence, the correct option is (D).
1. Simple Interest = = `21,600
100 8. Let the sum be `P and the rate of interest be R% p.a.
Amount = Principal + Simple Interest 2
⎛ R ⎞
= 24000 + 21600 = `45,600 P ⎜1 + ⎟ = 6000 (9)
⎝ 100 ⎠
Hence, the correct option is (C). 3
⎛ R ⎞
2. The interest on a sum remains the same each year under P ⎜1 + ⎟ = 7200 (10)
⎝ 100 ⎠
simple interest.
Dividing (10) by (9),
\ Total interest = 3(1000) = `3000
R
Hence, the correct option is (B). 1 + = 1.2
100
3. Let the sum be `100.
R = 20
Sixteen times of sum = `1,600
Hence, the correct option is (B).
Interest = `1,500
9. Let the sum be `P.
100 × 30 × x
1500 = Let the rate of interest be R% p.a.
100
⇒ x = 50 i.e. 50 years. ⎡⎛ R ⎞
3
⎛ R ⎞ ⎤
2

P ⎢⎜1 + ⎟ − ⎜⎝ 1 + ⎟ ⎥ = 2420
Hence, the correct option is (D). ⎢⎣⎝ 100 ⎠ 100 ⎠ ⎦⎥
4. Let the sum be `x.
⎡⎛ R ⎞
4
⎛ R ⎞ ⎤
3
Then Amount = `10x. P ⎢⎜1 + ⎟ − ⎜⎝ 1 + ⎟ ⎥ = 2662
Interest = `9x. ⎢⎣⎝ 100 ⎠ 100 ⎠ ⎥⎦

Let the time period be r years and rate of interest be r%. 100t R
x×r×r P  = 2420 (11)
⇒ 9x = 3 100
100
⇒ r = 30% ⎛ R ⎞
3
R
P  ⎜1 + ⎟ = 2662 (12)
Hence, the correct option is (C). ⎝ 100 ⎠ 100
5. Let its present value be `P. Dividing (12) by (11),
⎛ 6 (10) ⎞ R
P ⎜1 + = 2400
⎝ 100 ⎟⎠ 1 +
100
= 1.1 ⇒ R = 10
P = 1500 Alternate method:
Hence, the correct option is (D). (100) The interest on a sum for the kth year under com-
6. Let the rate of interest be R% p.a. pound interest, interest being compounded annually is
Amount at the end of 2 years = `630 1+ R
(Interest for the (k – 1)th year)
⎡⎛ R ⎞ ⎤
2
100
3630 = 3000 ⎢⎜1 + ⎟ ⎥
⎢⎣⎝ 100 ⎠ ⎥⎦ ⎛ R ⎞
2420 ⎜1 + = 2662
2 ⎝ 100 ⎟⎠
⎛ R ⎞
(1.1)2 = ⎜1 + ⎟ \ R = 10
⎝ 100 ⎠
Hence, the correct option is (A).
R
1.1 = 1 + 10 = R 10. Let the sum be `100
100
Hence, the correct option is (A). Amount after eight years = `400
7. Interest earned on a sum of `P invested for T years at Amount after sixteen years = `1,600
R% p.a. under compound interest i.e., it takes 8 + 8 = 16 years for the sum to become
⎡⎛ R ⎞
T
⎤ sixteen times of itself.
= `P ⎢⎜1 + ⎟ − 1⎥
⎢⎣⎝ 100 ⎠ ⎥⎦

Unit I_Chapter 5.indd 62 27/04/2017 14:19:29


Chapter 5  •  Simple Interest and Compound Interest  |  1.63

100     400     1600 P = 20000
Hence, the correct option is (A).
8 8 16. Let the rate of interest be R% p.a.
8 + 8 = 16 years ⎛ R ⎞
Extra interest = (10000) ⎜ (2) = 4000
Hence, the correct option is (D). ⎝ 100 ⎟⎠
11. Let the sum be `x. R = 20
Then, the amount after three years = `2.197 × x Hence, the correct option is (C).
⎛ 100 + r ⎞
3 17. Let the sum be `P.
⇒ 2.197 × x = x ⎜ ⎟ Let the rate of interest be R% p.a.
⎝ 100 ⎠
⎡⎛ 2
R ⎞ ⎛ R ⎞⎤
⎛ 100 + r ⎞
3
P ⎢⎜1 + ⎟ − ⎜1 + ⎟ ⎥ = 14400 (13)
⇒ (1.3)3 = ⎜ ⎟ ⎢⎣⎝ 100 ⎠ ⎝ 100 ⎠ ⎥⎦
⎝ 100 ⎠
⎡⎛ 3
R ⎞ ⎛ R ⎞ ⎤
2

100 + r P ⎢⎜1 + ⎟ ⎜ − 1 + ⎟ ⎥ = 17280 (14)


⇒ 1.3 =
100 ⎢⎣⎝ 100 ⎠ ⎝ 100 ⎠ ⎥⎦
S
⇒ 130 = 100 + r ⇒ r = 30% Dividing (14) by (13), ⇒ R = 20
x
Hence, the correct option is (A).
Hence, the correct option is (C).
20
12. Rate of interest = % = 10% per half year. 18. If a sum is doubled in the same time at simple interest
2
as well as compound interest, the rate of interest under
⎡⎛ 10 ⎞
2
⎤ simple interest is more than rate under compound
Interest = 200 ⎢⎜1 + ⎟ − 1⎥ = `42
⎢⎣⎝ 100 ⎠ ⎥⎦ interest.
Hence, the correct option is (A).
Hence, the correct option is (B).
19. Let the sum be `P.
13. Rate of interest = 10% per quarter. A sum of `P under
compound interest when invested at R% p.a. would Let the rate of interest be R% p.a
⎛ R ⎞
N
⎛ R ⎞
0.4P = (P) ⎜ (2)
become `P ⎜1 +
⎝ ⎟
100 ⎠ ⎝ 100 ⎟⎠
A sum of `100 would become 20 = R
2
⎡⎛ 10 ⎞ ⎤ ⎛1 + x ⎞ P = P ⎛1 + 20 ⎞
100 ⎢⎜1 + ⎟ ⎥ = 146.41 ⎜⎝ ⎟ ⎜⎝ ⎟
100 ⎠ 100 ⎠
⎣⎝ 100 ⎠ ⎦
⇒ x = 44
\ This is the effective rate of interest is 46.41% p.a.
Hence, the correct option is (D).
Hence, the correct option is (D).
20. Let the sum be `2P.
14. Let the sum Ashok borrowed from the bank lending at
10% p.a. be `x `P lent at simple interest fetches `0.2P as interest each
year. `P lent at compound interest fetches `0.24P as
Sum he borrowed from the other bank = `(84000 – x)
interest in the second year and `0.3456P as interest in
10 20 the fourth year
x+ (84000 – x) = 13200
100 100 0.24P – 0.2P = 400
x = 36000  P = 10000
difference = 84000 – 2x = 12000 Required difference = 0.3456P – 0.2P
Hence, the correct option is (D). = `1456
15. Let the sum be `P Hence, the correct option is (C).
⎛ 10 ⎞
Extra interest = P ⎜ ⎟ (2) = 4000
⎝ 100 ⎠

Unit I_Chapter 5.indd 63 27/04/2017 14:19:31


1.64  |  Unit 1  •  Quantitative Aptitude

Practice Problems 2 amount that `10000 becomes under simple interest


Solutions for questions 1 to 20: ⎡ ⎛ 20 ⎞ ⎤
= (10000) ⎢1 + 3 ⎜ ⎟ ⎥ = 16000
1. What is the principal, if after five years at 11% p.a. ⎣ ⎝ 100 ⎠ ⎦
­simple interest, it amounts to `18600? interest realized is `1160 less
(A) `12000 (B) `8000 Hence, the correct option is (D).
(C) `15000 (D) `11000
4. Krishna takes a loan of `8000 at simple interest. After
Pnr four years, he takes an additional loan of `14440. From
Sol. Given P + = 18600
100 that point, compound interest at 10% per annum is cal-
⎛ nr ⎞ culated on the total amount repayable on the first loan
P ⎜1 + = 18600 n = 5, r = 11
⎝ 100 ⎟⎠ as well as the second loan. He repays a total of `30250

after two more years to clear the entire loan amount.
18600 What is the annual rate of simple interest?
⇒ P= = 12000
5 × 11 (A) 8% (B) 9%
1+
100 (C) 7% (D) 6%
Alternate method: Sol. Let the amount that `8000 becomes after 4 years be x.
Then (x + 14440) (1.1)2 = 30,250
A = P (1 + nr%), ⇒ 18600 = P(1 + 55%) = 1.55 × P
Or x + 14440 = 30250/1.21
⇒ 18600 × 100 = 155P, ⇒ P = 12000
x + 14440 = 25000; 10560 = x
Hence, the correct option is (A).
Now, let the rate of simple interest for the first 4 years
2. Given that a principal amounts to `10080 at 10% p.a be r% Then 8000(1 + 4r/100) = 10,560; 320r = 2560,
simple interest after two years, what is the compound r=8
interest for two years on this principal at the same rate?
(A) `10164 (B) `1764 Hence, the correct option is (A).
(C) `1640 (D) `3764 5. Prakash invested a certain amount in a six-year fixed
⎛ nr ⎞ deposit scheme, interest being compounded annually.
ol. For Simple Interest P ⎜1 +
S = 10080
⎝ 100 ⎟⎠ The interests accrued on this deposit for the fourth
n = 2, r = 10 and fifth years, respectively, are `1331 are `1464.10.
If George deposited `12000 in the same scheme, how
P = 10080/1.2 = 8400 much interest would be accrued on this deposit for the
n = 2, r = 10, P = 8400 first two years?
⎛ 10 ⎞
2 (A) `1320 (B) `1452
A = 8400 ⎜1 + ⎟ = 10164 (C) `2520 (D) `1440
⎝ 100 ⎠
Sol. When compounded annually, the interest accrued for
\ C.I. = 10164 - 8400 = `1764 the nth year is r% more than the interest accrued for the
Alternate method: (n – 1)th year, where r is the annual rate of interest.
A = P(1 + nr%) ⇒ 10080 ⇒ P(1 + 20%); ⎛ r ⎞
\ I5 = ⎜1 + ⎟ I4
10080 ⎝ 100 ⎠
Under C.I., A = P(1 + R%)n = (1.1)2 = 10,164
1.2 ⎛ r ⎞ r
Hence, C.I. = 10,164 – P = 10,164 – 8400 = 1764. ⇒ 1464.1= ⎜1 + ⎟ (1331) ⇒ 1 + 100 = 1.1
⎝ 100 ⎠
Hence, the correct option is (B).
⇒ r = 10%
3. `10000 was lent at compound interest, interest being
Interest accrued for the first two years when `12000 is
compounded annually for 3 years. The annual rates of
invested in the same scheme
interest for the first, second, and third years were 10%,
20%, and 30%, respectively. If it was instead, lent at ⎡⎛ 10 ⎞ ⎤
= 1200 ⎢⎜1 + ⎟ − 1⎥ = 12000[(1.1) –1]
2

20% p.a. simple interest for the same time, how much ⎣⎝ 100 ⎠ ⎦
more/less interest would be realized? = 12000 × 0.21 = 2520
(A) ` 530 more (B) `530 less
(C) `1160 more (D) `1160 less Hence, the correct option is (C).
6. The difference between the amounts to be repaid by a
Sol. Amount that `10000 becomes under compound interest
man at the end of two years, at 20% per annum com-
⎛ 10 ⎞ ⎛ 20 ⎞ ⎛ 30 ⎞ pounded annually and half yearly, is `1084.50. What is
=1000 ⎜1 + 1+ 1+ = 17160
⎝ 100 ⎟⎠ ⎜⎝ 100 ⎟⎠ ⎜⎝ 100 ⎟⎠ the principal?

Unit I_Chapter 5.indd 64 27/04/2017 14:19:32


Chapter 5  •  Simple Interest and Compound Interest  |  1.65

(A) `45000 (B) `50000 (A) `320 (B) `450


(C) `51500 (D) `52500 (C) `250 (D) `400
Sol. If compounding is done annually r = 20%, n = 2, Sol. Let the annual instalment be `x.
Let P = 100, A = 100(1.2)2 = 144 Let (FV)i denote the value of the ith instalment at the
If compounding is done half-yearly end of the fifth year.
r = 10%, n = 4, A = 146.41 Difference = 2.41 Sum of future values of all the instalments = `1450
If P = 100 then, difference = 2.41 ( x ) (8) ( 4)
(FV)1 = x +
1084.5 100
If difference is 1084.5, P = ×100 = `45000
2.41 ( x ) (8) (3)
(FV)2 = x +
Hence, the correct option is (A). 100
7. Venkat lends a sum P at r% compound interest, com- ( x ) (8) ( 2)
pounded every moment for ten years. It becomes ‘a’ (FV)3 = x +
100
times P after ten years. What is the annual interest rate?
(Assume that a = e2) ( x ) (8) (1)
(FV)4 = x +
(A) 100% (B) 20% 100
(C) 40% (D) 200% (FV)5 = x
Sol. Sum = P, C.I = r%, n = 10 years x
\ 5x + (8) (4 + 3 + 2 + 1) = 1450
⎛ nr ⎞ 100
⎜ ⎟
Given, Pe ⎝ 100 ⎠ = a . P ⇒ 5x + 0.8x = 1450
r 1450
10 x
e 100 = a ⇒ x= = 250
5.8
Given e = a2 hence er/10 = e2 ⇒ r = 20 Alternate method:
1 th
Hence, the correct option is (B). Each instalment must be less than `( of 1450) =
5
8. Sourabh borrows `2500000 at 12% compound interest `(290). From among the choices given, only Choice (C)
from a bank and invests in shares. The investment gives satisfies the condition.
him a return of 20% per annum and he repays `500000
Hence, the correct option is (C).
at the end of first year. How much does he make for
himself after paying all the outstanding amount at the 10. A man took a loan of `100000 at 8% per annum com-
end of the second year? pound interest. He repays `10000 per annum. What is
(A) `424000 (B) `356241.50 the amount due from him at the beginning of the third
(C) `525000 (D) `484241.80 year?
(A) `91917 (B) `81917
Sol. He borrows `2500000
(C) `93970 (D) `95840
At the end of the first year it becomes
Sol. Principal = `100000
2500000 × (1 + 12/100)1 = `2800000
At the end of first year it amounts to
He repays `500000(15)
100000(1 + 8/100)1 = 108000
`2300000 is the principal for the 2nd year.
Out of this he repays `10000
This becomes 230000 (1 + 12/100)1 = `2576000
So, amount due at the end of 1st year
Which is repaid by the person, at the end of 2nd year.
= 108000 – 10000 = `98000.
In the first year, (on his earnings side), he earns 20% on
`2500000 = `500000. At the end of the second year this amounts to
Out of this, (15) is repaid. Investment remains as 98000 [1 + 8/100]1 = 105840.
`250000. Out of this he repays `10000
This earns 20% during the second year. So, it becomes \ At the beginning of the third year amount due from him
`3000000. Out of this `2576000 is repaid. So, he = 105840 – 10000 = `95840
makes, 3000000 – 2576000 Hence, the correct option is (D).
= `424000 11. Srikanth buys a car worth `525000. He pays `125000
Hence, the correct option is (A). as down payment and agrees to pay the remaining
9. What annual instalment will discharge a debt of `1450 amount in instalments. What is the approximate yearly
due after five years at 8% p.a simple interest? instalment amount to be paid by him, if at 12% p.a.

Unit I_Chapter 5.indd 65 27/04/2017 14:19:34


1.66  |  Unit 1  •  Quantitative Aptitude

compounded annually, he repays the remaining amount (A) 10 % p.a (B) 12% p.a
in three more years? Given that (1/1.12)3 = 0.71. (C) 15% p.a (D) 18% p.a
(A) `160000 (B) `165000 Sol. Suhaas borrows at 28% compound interest and pays
(C) `180000 (D) `183000 back in 1 year (let him borrow P1)
Sol. Cost of the car = `525000 1
⎛ 28 ⎞
Cash payment = `125000 P1 ⎜1 + = 38400
⎝ 100 ⎟⎠

\ Principal of loan to be discharged in instalments
   P1 (1.28) = 38400
= `400000
  P1 = 30000
Pr
Each instalment X = Let Bhanu borrow P2
⎡ ⎛ 100 ⎞ n ⎤
100 ⎢1 − ⎜ ⎟ ⎥ Given  P1 + P2 = 54000;
⎢⎣ ⎝ 100 + r ⎠ ⎥⎦ P2 = 24000
400000 × 12 ⎛ 4×r⎞
 = 24000 ⎜1 + = 38400
⎡ ⎛ 100 ⎞3 ⎤
100 ⎢1 − ⎜ ⎝ 100 ⎟⎠
⎟ ⎥
⎢⎣ ⎝ 112 ⎠ ⎥⎦ 4r/100 = 1.6 – 1= 0.6
4000 ×12 4000 ×12 ⇒ r = 15%
= =
(1 − 0.71) 0.29 Hence, the correct option is (C).
4000 ×12 4000 × 12 14. Akshay deposits a sum P in a bank at r% compound
@ = = 160000. interest. The amount becomes 27P after three years by
0.29 0.3
compounding annually. Instead, if the bank had com-
As the actual denominator is slightly less than 0.3 the pounded half yearly, what is the additional amount
actual answer should be slightly more than `160000. Akshay would have received in terms of P?
Actual answer is `165000 as per the options. (A) 54P (B) 37P (C) 18P (D) 5P
Hence, the correct option is (B). Sol. Sum = P; C.I. = r%; In 3 years, amount = 27P
12. A sum was lent for a year, another sum was lent for ⎛ r ⎞
3

P ⎜1 + = 27 P
2 years and another sum was lent for 3 years. Each ⎝ 100 ⎟⎠
sum was lent at 5% p.a compound interest. If each
sum amounted to the same value, the ratio of the first, ⎛ r ⎞
3

⇒ ⎜1 + = 27
­second and third sums is _____. ⎝ 100 ⎟⎠
(A) 400 : 420 : 441 (B) 20 : 21 : 22
(C) 22 : 21 : 20 (D) 441 : 420 : 400 r
⇒ 1+ = 3 r = 200.
Sol. Let the first, second and third sums be `f, `s and `t 100
respectively. Compounding half yearly, the amount will be
2 3
⎛ 5 ⎞ ⎛ 5 ⎞ ⎛ 5⎞ ⎛ 100 ⎞
6
f ⎜1 + ⎟ = s ⎜1 + ⎟ = t ⎜1 + ⎟ = A (say) P ⎜1 + = P × 26 = 64P
⎝ 100 ⎠ ⎝ 100 ⎠ ⎝ 10 ⎠ ⎝ 100 ⎟⎠

A A A Additional Amount received = 64P – 27P = 37P
f :s:t = : :
21 441 9261
Hence, the correct option is (B).
  20 400 8000
15. A sum was split into four parts (P1, P2, P3, P4) where
20 400 8000 P1 : P2 : P3 : P4 = 1 : 4 : 5 : 2. Each part was lent at simple
= : :
21 441 9261 interest. P1 was lent at 10% p.a. for a year. P2 was lent at
= 8820 : 8400 : 8000 = 441 : 420 : 400 20% p.a. for 5 years. P3 was lent at 2% p.a. for 4 years.
P4 was lent at 6% p.a. for 10 years. The greatest of the
Hence, the correct option is (D).
interests on the parts exceeds the least of the interests
13. Suhaas borrowed a certain amount at 28% p.a. com- on the parts by `7800. Find the total interest fetched by
pound interest and repays it in one year. Bhanu borrows the parts (in `).
a certain amount at a certain interest rate under simple (A) 5700 (B) 8550
interest and returns it after four years. If the amounts (C) 11400 (D) 17100
repaid by Suhaas and Bhanu are the same and that is
Sol. Let P1 = `x
equal to `38400 and the sum of their principals bor-
rowed is `54000, at what rate Bhanu paid the interest? P2 = `4x.

Unit I_Chapter 5.indd 66 27/04/2017 14:19:35


Chapter 5  •  Simple Interest and Compound Interest  |  1.67


P3 = `5x (A) `28124.24 (B) `29324.20
P4 = `2x (C) `31349.75 (D) `34702.50
⎛ 10 ⎞ Sol. `20000 deposited at the beginning of the first year
Simple interests on P1, P2, P3 and P4 are (x) ⎜ ⎟ (1)
⎝ 100 ⎠ becomes, in 2 years, an amount equal to
⎛ 20 ⎞ ⎛ 2 ⎞ 20000 × (1.05)2 = `22050
(4x) ⎜ (5), (5 x ) ⎜ ( 4)
⎝ 100 ⎟⎠ ⎝ 100 ⎟⎠ (i.e., at the beginning of the third year)
`20000 deposited at the beginning of the second year
⎛ 6 ⎞
and (2x) ⎜ (10) i.e., becomes in 1 year (20000) × (1.05) = `21000
⎝ 100 ⎟⎠
\ Amount at the beginning of the third year
`01x, `4x, `0.4x and `1.2x respectively
= 22050 + 21000 = `43050
4x – 0.1x = 7800 x = 2000
Now `10000 is withdrawn
The total simple interest = 5.7x = `11400
\There is `33050 in the account on which 5% interest
Hence, the correct option is (C). for one more year is earned.
16. A man invested `40000 in a bond which gives 10% p.a. Final amount = 33050 × 1.05 = `34702.5
interest, compounded half yearly. If the annual rate of
interest is increased by 20 percentage points at the end Hence, the correct option is (D).
of every half year, what will be the interest for the first 19. Anwar borrowed `72000 at 20% p.a. compound inter-
one and half years? est, interest being compounded annually. He repaid `x
(A) `15875 (B) `16750 at the end of the first year. He repaid `57600 at the end
(C) `20375 (D) `19875 of the second year and thereby cleared the loan. Find x.
Sol. As the rate of interest is increasing by 20 percent- (A) 40000 (B) 36000
age points annually, the rate of interest per half year (C) 38400 (D) 32000
increases by 10 percentage points. Sol. Value of `72000 at the end of the first year = `86400.
Since `x was repaid at the end of the first year, in order
Interest of to clear the loan at the end of the second year, the
Half Interest for the end of
year Principal the half year half year Amount
amount to be repaid must be
20
1 40000 2000 2000 42000 (86400 – x) + (86400 – x)
100
2 42000 6300 8300 48300
1.2 (86400 – x) = 57600
3 48300 12075 20375 60375
x = 38400
\ The required interest will be `20375. Hence, the correct option is (C).
Hence, the correct option is (C). 20. In the above question, if all the interests considered had
17. A loan has to be repaid after three years under com- been compound interest, interest being compounded
pound interest, interest being compounded annually at annually, how much more/less interest would be realized?
30% p.a. Find the amount borrowed if the amount to be (A) 1632 more (B) 1632 less
repaid is `87880 (in `). (C) 2014.40 more (D) None of these
(A) 36000 (B) 40000 Sol. The amount that `1000 becomes under compound
2
(C) 48000 (D) 45000 ⎛ 10 ⎞
interest = (1000) ⎜1 + ⎟ = `1210
Sol. Let the present value be `x. ⎝ 100 ⎠
87880 The amount that `3000 becomes under compound
x = 3
= 40000
⎛ 30 ⎞ ⎛ 30 ⎞
2
⎜⎝1 + ⎟ interest = (3000) ⎜1 + ⎟ = `5070
100 ⎠
⎝ 100 ⎠
Hence, the correct option is (B).
The amount that `4000 becomes under compound
18. Srikar saves `20000 at the beginning of each year in a 2
savings bank account that pays 5% p.a, interest being ⎛ 20 ⎞
interest = (4000) ⎜1 + ⎟ = `5760
compounded annually. If, at the beginning of the third ⎝ 100 ⎠
year, instead of depositing `20000, he withdraws The interest realized is `520 less
`10000, how much would be the total savings of the
man at the end of three years? Hence, the correct option is (D).

Unit I_Chapter 5.indd 67 27/04/2017 14:19:36


Chapter 6 Averages, Mixtures and Alligations
Hints/Solutions
Practice Problems 1 7. Quantity of milk in 20 litres = (0.6) (20) = 12 litres.
Upon addition of 5 litres of pure milk, the new solution
1. The two digit numbers divisible by 10 are 10, 20, 30,
of 25 litres would contain 17 litres as milk.
40, 50, 60, 70, 80 and 90.
17
Sum of the two digit numbers divisible by 10 \ Percentage of milk = (100) = 68%
25
= 10 + 20 + 30 + 40 + 50 + 60 + 70 + 80 + 90 = 450
450 Hence, the correct option is (C).
\required average = = 50
9 8.
Hence, the correct option is (B). 80 108
2. There are 25 odd numbers less than 50 96
1 + 49 12 16
Average = = 25
2
Hence, the correct option is (D). Ratio = 3 : 4
3. The last multiple of 12 less than 100 is (12) Hence, the correct option is (C).
Average of all the multiples of 12 75
9. Quantity of milk in A = (20) =15 litres.
100
(8) (9)
12 (1 + 2 + 3 + .... + 8 ) 12
2 60
 = = = 54 Quantity of milk in B = x litres.
8 8 100
Hence, the correct option is (B). 60 66
15 + x= (20 + x)
4. Average salary per month 100 100
4 (6000) + 4 (8000) + 4 (13000) 15 + 0.6x = 13.2 + 0.66x ⇒ x = 30
=
12 Hence, the correct option is (B).
6000 + 8000 + 13000 10. Let the cost price of the milk be `x/litres.
= = `9000
3 Let the quantity of water to be added to the milk be y
Hence, the correct option is (B). litres.
5. Let the numbers of male employees and female employ- Selling price of the mixture = ` x/litre.
ees be m and f respectively. x (100) 5
180 m + 170 f Cost price of the mixture = = ` x/litre.
Average height of the employees = 100 + 60 8
m+ f
Cost of water = `0/litre
m Method 1:
180 + 170
f
= . 5
m \ 15x + 0 = x (15 + y) 9 = y
+1 8
f
As m/f is unknown, the average height cannot be found. Method 2:
Hence, the correct option is (D). 5
x − x
6. Let the number of girls in the class be g. y
= 8 =3
15 5 5
150 3 x−0
Number of boys in the class = g = g. 8
100 2
y=9
⎛ 3 ⎞
Total of the ages of the boys = 10 ⎜ g ⎟ . = 15 g years. Hence, the correct option is (A).
⎝ 2 ⎠ 2
Total of the ages of the girls = 8 g years. ⎛ 90 − 9 ⎞ 81
11. Ratio of milk and present mixture = ⎜ ⎟ =
Total of the ages of the students = 23 g years. ⎝ 90 ⎠ 100
\ Ratio of milk and water = 81 : 19
23 g
Average age of the class = = 9.2 years. Hence, the correct option is (C).
3
g+ g 12. Let the total number of matches that Sachin Tendulkar
2
Hence, the correct option is (C). played be x.

Unit I_Chapter 6.indd 68 27/04/2017 14:19:44


Chapter 6  •  Averages, Mixtures and Alligations  |  1.69

2100 2201 15. Cost price of each litre of diluted milk


+1 =
x −1 x 100 16
= (4) = `
Among the choices, choices (A) and (C) satisfy the 100 + 25 5
equation above. By the principle of allegation,
Hence, the correct option is (D). Amount of water used for mixing
13. Let the average of classes I and II be A kg and B kg. Amount of milk used for mixing

Let the number of students in the classes I and II be a 16 4
and b respectively. 4−
5 1
= = 5 =
Aa + Bb A + B 16 16 4
= −0
a+b 2 5 5
2Aa + 2Bb = Aa + Bb + Ab + Ba Hence 0.25 litres of water will be mixed with each litre
(a – b) (A – B) = 0 of milk.
As a ≠ b, A = B(3) Hence, the correct option is (A).
and 2A + B = 60  (4) 16. Let the required proportion be x : y : z
solving (3) and (4), we get A = B = 20. 10 x + 12 y + 18 z
= 14
Hence, the correct option is (C). x+ y+z
14. Cost price of a mixture of two varieties of wheat Going by the options, we have only Choice (D) satisfy-
ing the above equation.
100
= x 28 = ` 20 Alternate method:
100 + 40
Prices of 3 varieties of sugar are `10, `12 and `18
Let the cost price of the first and the second varieties of per kg.
wheat be `y per kg and `(y + 3) per kg respectively. Let them be mixed in the ratio x : y : z
Let the quantities be 3x kg and 4x kg respectively. Hence, price of the mixture is
y ( 3 x ) + ( y + 3)( 4 x ) (10x + 12y + 18z)/(x + y + z)
Cost price of the mixture = But this value is given on `14.
3x + 4 x
7 xy + 12 x 12 Hence, (10x + 12y + 18z)/(x + y + z) = 14
= = y + = 20 ⇒ 10x + 12y + 18z = 14x + 14y + 14z
7x 7
⇒ 4x + 2y – 4z = 0;
Cost price of the first variety of wheat ⇒ 2x + y – 2z = 0
12 128
= 20 − = ` /kg This equation has infinite solution sets. Hence, from
7 7
among the given options, the one which satisfies the
Alternate method:
equation will be the solution.
A mixture of two varieties of wheat is sold at `28 per
Option (D), i.e., 3 : 4 : 5 satisfies the equation.
kg, at a profit of 40%.
Hence, the correct option is (D).
Hence, the cost price of the mixture = (28)/(1.4) = `20
Let x and (x + 3) be the prices in rupees of the first and 3 4
17. In the first, second and third mixtures, th, th and
the second varieties respectively. th 5 5
7
The quantities are mixed in the ratio 3 : 4 (given) of the contents respectively is alcohol. Let x ml of
10
By alligation equation, each of these be drawn and mixed. The resulting mix-
q1 p − p ⎛3 4 7 ⎞
= 2 ; ture of 3x ml will have ⎜ x + x + x ⎟ ml of alcohol
q2 p − p1 ⎝5 5 10 ⎠

21
3 ( x + 3) − 20 i.e. x ml of alcohol \ It will have 70% alcohol.
⇒ = 10
4 20 − x
Hence, the correct option is (B).
3 x − 17 18. Applying the alligation equation, Quantity of milk/
⇒ = ;
4 20 − x 16 − 0
quantity of water = = 8 :1
⇒ x = 128/7 18 − 16

Hence, the correct option is (A). Hence, the correct option is (A).

Unit I_Chapter 6.indd 69 27/04/2017 14:19:46


1.70  |  Unit 1  •  Quantitative Aptitude

19. Let A’s, B’s, C’s, and D’s weights be a kg, b kg, c kg and x 12 − 10.5 3
d kg respectively. 1800 − x = 10.5 − 10 = 1
b+c+d Hence x = 3/4 (1800) = 1350 and y = 1/4 (1800) = 450
a = = 3a = b + c + d (5)
3 Interest earned by the business man from the part lent
a+c+d 1350 × 1 × 10
b = = 3b = a + c + d (6) at 10% = = ` 135
3 100
Adding (5) and (6), Hence, the correct option is (A).
3 (a + b) = a + b + 2 (c + d) 24. Let the capacity of the vessel be x ml.
a + b = c + d Amount of milk originally in the vessel = 9x/100 ml
c+d a+b Amount of milk in the vessel after replacement by water
= 30 \ = 30.
2 2 = 6x/100 ml.
Hence, the correct option is (B). Amount of milk in the 9 litres withdrawn
20. Let the fifth number be x. Then, from the given data, = 9(9)/100 = 81/100 ml
second number = 7x Hence 9x/100 – 6x/100 = 81/100;
7x + 3 ⇒ 3x/100 = 81/100
first number =
3 ⇒ x = 27 litres
7x + 3 Alternate method:
fourth number = −2
3 In the case of replacement, the relation between the
7x + 3 7x + 3 ⎛ 7x + 3 ⎞ ­initial and the final concentration is:
+ 7x + +⎜ − 2 ⎟ + x = 5 x 16.2 = 81
3 9 ⎝ 3 ⎠ ⎛v − x⎞
C1 ⎜ ⎟ = C2
Solving, we get x = 6 ⎝ v ⎠
Hence, the correct option is (D). C1, C2 are the initial and the final concentrations respec-
21. As the tens digits of the 9 numbers are different, the tively; v is the total volume, x is the volume replaced.
tens digits must be 1, 2, 3, 4, 5, 6, 7, 8 and 9. The units ⎛v − 9⎞
digits of the numbers must be 0, 1, 2, 3, 4, 5, 6, 7 and 8. Hence, 9% ⎜ ⎟ = 6%; ⇒ v = 27
⎝ v ⎠
36
Their average = =4 Hence, the correct option is (A).
9
25. Let the quantities of A, B and C used for mixing be
Hence, the correct option is (B).
a kg, b kg and c kg respectively.
22. Average value per coin = 775 paise/100 coins 7.75
paise/coin. By the application of the alligation equation. 15(100)
Cost price of the mixture = = `9/kg
2
The number of 5 paise coins and the number of 10 100 + 66
3
10 − 7.75 2.25 9
paise coins = = = Total cost price of the mixture = 6a + 9b + 12c = 9
7.75 − 5 2.75 11
(a + b + c) ⇒ a = c
Hence the number of 5 paise coins = (9/20) × (100) = 45
As b = 50 and a + b + c = 100, a = 25 and b = 25.
Hence, the correct option is (D).
\ The quantity of variety A is 25 kg.
Note: The problem can also be solved using simultane-
ous equation. Hence, the correct option is (B).
23. Let the part lent at 10% be `x and that lent at 12% be
(1800 – x).

Practice Problems 2 (A) 24 years (B) 40 years


(C) 52 years (D) None of these
1. The average age of a board of 10 advisors of a company
is the same as it was 3 years ago, on account of the Sol. Let the present average age of ten advisors as well as
replacement of one of the older advisors by a younger their average 3 years ago, be A. Also let the age of the
man. How much younger is the new man than the direc- younger man be Y and that of the director replaced by
tor whom he replaced? him be D.

Unit I_Chapter 6.indd 70 27/04/2017 14:19:47


Chapter 6  •  Averages, Mixtures and Alligations  |  1.71

10 A + 30 − D + Y TA + TB + TC
= A , as the present average is same
10 Hence p =
nA + nB + nC
as the earlier average.
Hence D – Y = 30 years 73.5 ( nA + nB + nC ) 1.5nA + 4 nC
= +
Hence, the correct option is (D). nA + nB + nC nA + nB + nC

2. The average of n numbers is a. If one of the numbers, y, ⇒ p > 73.5 (9)
is replaced by x, the average becomes b. Find the rela- p can also be written as,
tion between n, a, x, y, and b. T + TB + TC
1 x− y 1 a−b p= A
(A) = (B) = nA + nB + nC
n a−b n y−x
1 x+ y 1 a+b 77.5( nA + nB + nC ) 2.5nA + 4.nB
(C) = (D) = = −
n a+b n x+ y nA + nB + nC nA + nB + nC
Sol. an = y + k (7) ⇒ p < 77.5 (10)
bn = x + k (8) Hence 73.5 < p < 77.5
where k is the sum of the (x – 1) numbers which are Hence, the correct option is (D).
other than y and x.
5. All the members of a club meet for lunch every Monday.
Subtracting equation (8) from equation (7) Last week, just before the bill was presented, two of the
1 a−b club members were called for an official meeting and
n(a – b) = y – x ⇒ =
n y−x left. The remaining members were presented with a bill
of `1440. It was customary to divide the bill equally. To
Hence, the correct option is (B).
cover the share of the two who left, each member had
3. The average weight of N boys in a group is 36 kg. If to pay an extra amount of `24. How many people were
20 other boys whose average weight is 30 kg. join present for the lunch?
the group, the average weight of the group would be (A) 10 (B) 12 (C) 15 (D) 14
the same as what it would be if 5 boys whose average
weight is 40 kg leave the group. Find N. Sol. Let the number of club members who meet for lunch
(A) 10 (B) 15 (C) 20 (D) 25 1440 1440 1440 1440
be N. + 24 = ; − = 24
N N −2 N −2 N
36 N + ( 20)(30) 36 N − (5)( 40)
Sol. = Solving, N = 12
N + 20 N −5
⇒ N = 10 Hence, the correct option is (B).
Hence, the correct option is (A). 6. The average of n numbers is 16. If 5/8th of the num-
bers are doubled and 3/8th of the numbers become
4. The students of three classes, A, B, and C take a test. 10/3 times their original values, by what percentage
The average per student marks of the classes A and B does the current average exceed the original average?
put together is 71. The average marks per student of (A) 20%
the classes B and C put together is 76. The average per (B) 30%
student marks of the class A and C put together is 79. (C) 25%
Find the range of the average marks (p) of all the three (D) Cannot be determined
classes put together.
(A) 72 < p < 76 (B) 73 < p < 75 Sol. Total of all numbers = 16n
(C) 71 < p < 77 (D) 73.5 < p < 77.5 Once 5/8th of the numbers are doubled and 3/8th of the
Sol. Let the number of students in the three classes be nA, numbers are increased by a factor of 10/3,
nB and nC and the total scores of students in the three
( 2 A) + ⎜⎛ ⎟⎞ + 16n,
5n 3n 10 B
classes be TA, TB and TC total of all numbers =
8 8 ⎝ 3 ⎠
TA + TB = 71(nA + nB)
where A and B are the averages of values of the num-
TB + TC = 76 (nB + nC) bers which are respectively doubled and increased by a
TC + TA = 79 (nC + nA) factor of 10/3.
--------------------------------- 5n 5n
2TA + 2TB + 2TC = 150nA + 147nB + 155nC A + B + 16 n
New average = 4 4 .
⇒ TA + TB + TC = 75nA + 73.5nB + 77.5nC n

Unit I_Chapter 6.indd 71 27/04/2017 14:19:49


1.72  |  Unit 1  •  Quantitative Aptitude

As A and B are not known, the new average cannot be 10. The average weight of all the students in a class equals
determined. Hence the percentage increase in the aver- the average of the average weight of the boys and that
age cannot be determined. of the girls, which of the following holds true?
Hence, the correct option is (D). (A) The numbers of boys and girls in the class are equal.
(B) The average weights of the boys and the girls are
7. Ajay attempted to add ten two-digit numbers. One of
equal.
them, a, was the reverse of one of the others. If a was
(C) At least one of (A) and (B)
replaced by another two-digit number b and the reverse
(D) Neither (A) nor (B)
of a was replaced by the reverse of b and the average
was found, it would be 2.2 more. The sum of the digits Sol. Let the numbers of boys and girls in the class be b and
in b exceeds the sum of the digits in a by g respectively. Let the average weights of the boys and
(A) 1 (B) 2 (C) 3 (D) 4 the girls be B and G respectively.
Sol. Let a = 10 p + q and b = 10 r + s Bb + Gg B + G
=
Let the sum of all the numbers excluding a and b be x. b+g 2

Let the average of the numbers be A. 2Bb + 2Gg = Bb + Bg + Gb + Gg
x + 10 p + q + 10 q + p = 10 A (11) B (b – g) + G (g – b) = 0
x + 10 r + s + 10 s + r = 10 (A + 2.2)  (12) (B – G) (b – g) = 0
Subtracting (11) from (12), B = G or b = g or both.
11 [(r + s) – (p + q)] = 22 Hence, the correct option is (C).
⇒ (r + s) – (p + q) = 2 11. Two alloys A and B contain copper and zinc in the ratio
Hence, the correct option is (B). 4 : 9 and 5 : 6, respectively. If equal weights of the
8. There are N students in a class. Their class teacher gave two are melted together to form a third alloy, find the
them a task of finding the average of the first N natural ratio of the weights of copper and zinc in the third alloy
numbers and each of them left out a different number named C.
and found the average of the remaining numbers. The (A) 109 : 167 (B) 113 : 164
average of the averages obtained by all the students (C) 109 : 177 (D) 107 : 158
was 21. Find N. Sol. Alloy A Alloy B
(A) 39 (B) 40 (C) 41 (D) 38 Copper 4x 5y
Sol. As each student missed a different number, the sum of Zinc 9x 6y
the all the numbers added by the students = (N – 1) ------- ------
( N − 1) N ( N + 1) Total wt. 13x 11y
(sum of the first N natural numbers) = ------- ------
2
As the quantity to be drawn from the Alloys A and B
The number of numbers added by the students in total
to form another alloy C must be a multiple of 13 and
= (N - 1) (N) 11, let us choose the quantity drawn as LCM of 13 and
( N − 1) N ( N + 1) 11; which is 143. Hence 13x = 11y = 143 ⇒ x = 11
\ Average = = 21 and y = 13.
2( N − 1)( N )
Amount of copper in Alloy C = 4x + 5y
⇒ N = 41
= 4(11) + 5(13) = 109 kg.
Hence, the correct option is (C).
Amount of zinc in alloy C = 9x + 6y
9. In a class, there are 50 students. The average weight of
all the girls is 30 kg. The average weight of 30 of the = 9(11) + 6(13) = 99 + 78 = 177 kg.
boys is 40 kg. Which of the following can be a possible Hence the ratio of copper and zinc = 109 : 177
value of the average weight of the entire class (in kg)? Hence, the correct option is (C).
(A) 31 (B) 33 (C) 35 (D) 37
12. A vessel has 10 ml of a solution of milk and water con-
Sol. There are at least 30 boys in the class. taining 20% milk. x ml of milk was added to the vessel
\ There are more boys than girls in the class. to reverse this ratio. y ml of water was then added to the
If the average weight of the entire class is w, 30 < w < vessel to reverse the ratio once again. Find x + y.
40 and there are more boys than girls w is closer to 40 (A) 140 (B) 150
than 30, i.e., w > 35. (C) 160 (D) 170
Only Choice (D) satisfies this condition. Sol. Initial quantity of milk in the vessel = 0.2 (10) = 2 ml
Hence, the correct option is (D). Initial quantity of water in the vessel = 0.8 (10) = 8 ml

Unit I_Chapter 6.indd 72 27/04/2017 14:19:50


Chapter 6  •  Averages, Mixtures and Alligations  |  1.73

After x ml of milk was added, the ratio of milk and 2


(i.e less than or equal to 66 %), the milk quantity to
water would become 4 : 1 3
2
x = 4(8) – 2 = 30 be at most 66 litres. After the second replacement the
3
After y ml of water was added, ratio of milk and water th
729
would again become 1 : 4. vessel would have of its contents as milk. After
1000 6561
th
y = 4 (4(8)) – 8 = 120 the third replacement, the vessel would have
\ x + y = 150 10000
of its contents as milk i.e. 65.61 litres of milk (which is
Hence, the correct option is (B).
2
13. There are two containers having mixtures of hydro- less than 66 litres)
3
chloric acid and water. In container 1, the ratio of
The least number of times the procedure has to be
hydrochloric acid and water is 1 : 2 and in container 2
­carried out is 3.
the ratio of hydrochloric acid and water is 4 : 1. Find
the amount of the mixture that should be taken from Hence, the correct option is (C).
container 1 in order to make 28 litres of a mixture con- 15. Alloy A has 80% copper and 20% tin. A certain pro-
taining equal amount of water and hydrochloric acid. cess when applied repeatedly to the alloy decreases
(A) 15 litres (B) 14 litres the copper quantity in the alloy by one-fifth and the tin
(C) 20 litres (D) 18 litres quantity in it by one-tenth, each time it is applied. The
Sol. The data can be tabulated as: minimum number of times the process must be applied
so that the concentration of copper in the alloy is less
Alcohol Water Concentration
than 70% is ____.
Mixture 1 1 part 2 parts 1/3 (A) 4 (B) 5 (C) 3 (D) 2
Mixture 2 4 parts 1 part 4/5 Sol. Let the number of times the process is applied be N.
Combined Each application results in the copper quantity drop-
Mixture 1 part 1 part 1/2 ping by 20% and the tin quantity dropping by 10%. Let
If q1 and q2 are the quantities of mixture 1 and mixture the weight of the alloy be 100 kg. This has 80 kg of
2 that are mixed, then by alligation equation, copper and 20 kg of tin.
q1 ( 4 / 5) − (1 / 2) ( 3 / 10 ) 9 The process is applied N times, the quantity of copper
= = =
q2 (1 / 2) − (1 / 3) (1 / 6 ) 5 in the alloy will be (0.8)N 80 and that of tin in it will be
(0.9)N (20).
It is given that, q1 + q2 = 28;
( 0.8) (80 ) (100)
N
9 < 70
Hence q1 = × 28 = 18 litres
( 0.8) (80 ) + ( 0.9 ) ( 20 )
N N
14

Hence, the correct option is (D).
14. A vessel contains a mixture of 100 litres of milk and (0.8) N (80)
⇒ <7
water. The concentration of milk is 90%. 10% of the (0.8) N 8 + (0.9) N 2
contents of the vessel are withdrawn and replaced ⇒ (0.8)N (24) < (0.9)N (14)
with an equal amount of water. The minimum number
N N
of times that this procedure must be carried out such 12 ⎛ 9 ⎞ ⎛9⎞ 5
that the concentration of milk in the vessel is less than < ⎜ ⎟ i.e. ⎜ ⎟ >1 ≈ 1.7
2 7 ⎝8⎠ ⎝8⎠ 7
N
66 % is _____. ⎛9⎞
3 The values of N and the approximate values of ⎜ ⎟
(A) 1 (B) 2 (C) 3 (D) 4 are tabulated below. ⎝8⎠
Sol. The original mixture had 90 litres of milk and 10 litres
N
9 ⎛9⎞
of water \It had th of milk N ⎜ ⎟
10 ⎝8⎠
After the first withdrawal of 10 litres, 9 litres of milk
and 1 litre of water is lost. \ The resulting mixture 1 1.125
would have 81 litres of milk. On then adding water the 2 1.27
81 3 1.42
vessel would have th of its contents as milk
100 2 4 1.60
We want the milk concentration to be at most 66 %
3 5 1.80

Unit I_Chapter 6.indd 73 27/04/2017 14:19:52


1.74  |  Unit 1  •  Quantitative Aptitude

N must be at least 5 in order that the alloy may have less weight is x kg left A and joined B. As a result, the aver-
than 70% copper. age weight of A, as well as that of B decreased. Which
Hence, the correct option is (B). of the following must be true?
(A) 35 < x ≤ 40 (B) 40 < x < 60
16. A basket ball player played nine matches. The aver-
(C) 60 ≤ x < 65 (D) 30 < x ≤ 35
age number of points he scored per match was 16.
His points in the ith match were two less than that in Sol. As the average weight of A decreased after the student
the (i – 1)th match. Find the average number of points left, his weight must be more than the average weight
scored in the second and the eighth matches. of A. As the average weight of B decreased after the
(A) 8 (B) 12 (C) 16 (D) 18 student joined, his weight must be less than the average
weight of B.
Sol. Let the number of goals in the first match be a. The
number of goals in successive matches are a, a + 2, His weight must be between 40 kg and 60 kg
a + 4, a + 6, a + 8, a + 10, a + 12, a + 14, and a + 16 Hence, the correct option is (B).
9a + 72 20. In an office, the average salary of the employees is
The average number of goals = = a + 8 = 16
9 `8000. If the average salary of all the 50 clerks is `3000
The average number of goals in the second and eighth and the average salary of all the managers is `10000,
2a + 16 find the difference in the number of clerks and the num-
matches = = a + 8 = 16
2 ber of managers.
Hence, the correct option is (C). (A) 75 (B) 55 (C) 60 (D) 125
17. The average score of a class of 32 students is 70. If Sol. By using the alligation equation,
the top score and the least score, which differ by 70, Number of clerks 10000 − 8000 2
are excluded, the average score of the class drops by 1. = =
Number of managers 8000 − 3000 5
Find the top score.
(A) 110 (B) 120 (C) 130 (D) 100 Number of clerks = 50;
Sol. Total score = 2240 ⇒ Number of managers = 125
Total score, excluding the top and the least scores = Difference in the number of clerks and managers
(69) (30) = 2070 = 125 – 50 = 75
Let the top and the least scores be t and l respectively. Hence, the correct option is (A).
t + l = 2240 – 2070 = 170 (13) 21. Four classes P, Q, R, and S take up a test. The ratio of
t – l = 70  (14) the number of students in P, Q, R, and S is 2 : 3 : 6 :
4. The ratio of the average marks of P, Q, R and S is 5
Solving (13) and (14), T = 120
: 4 : 3 : 2. If the average marks of all combinations of
Hence, the correct option is (B). three sections are considered, the greatest is 52. Find
18. The average age of a group of men is 20 years. Two men the least.
whose ages are 23 years and 27 years joined the group. (A) 34 (B) 36.2 (C) 38 (D) 41.8
The average age of the group increased by a prime Sol. Let the number of students in P be 2x
number. The number of men in the original group is
Let the average marks of P be 5y
(A) 3 (B) 4 (C) 5 (D) 6
Numbers of students in Q, R and S are 3x, 6x and 4x
Sol. Let the number of men in the original group be N.
respectively.
The total age of the original group = 20 N years
Average marks of Q, R and S are 4y, 3y and 2y
The total age of the new group = (20 N + 50) years respectively.
Let the increase in the average be x years. Total marks of P, Q, R and S are 10xy, 12xy, 18xy and
20 N + 50 = (N + 2) (20 + x) 8xy respectively. Greatest weighted average
⇒ 10 = x (N + 2). Therefore, N + 2 and x must be f­ actors ⎛ 10 xy + 12 xy + 18 xy 10 xy + 12 xy + 8 xy ⎞
of 10. As x is prime, x can be only 2 or 5. ⎜ 2 x + 3x + 6 x , 2 x + 3x + 4 x , ⎟
= maximum ⎜ ⎟
If x = 2, N = 3. If x = 5, N = 0, which is not possible. ⎜ 12 xy + 18 xy + 8 xy , 10 xy + 18 xy + 8 xy ⎟
⎜ ⎟
\N=3 ⎝ 3x + 6 x + 4 x 2x + 6x + 4x ⎠
Hence, the correct option is (A). ⎛ 7 1 12 ⎞ 7
max ⎜ 3 y, 3 y, 2 y, 3 y ⎟ = 3 y
19. There are two classes A and B. The average weight of ⎝ 11 3 13 ⎠ 11
the students in class A is 40 kg. The average weight 7
of the students in class B is 60 kg. A student, whose ⇒ 3 y = 52
11

Unit I_Chapter 6.indd 74 27/04/2017 14:19:53


Chapter 6  •  Averages, Mixtures and Alligations  |  1.75

(52)(11) is again withdrawn and then replaced with an equal


⇒ y = amount of water. After these changes, the vessel con-
40
tains 17.1 litres of milk less than it did initially.
38
Least weighted average = y = 41.8 (i)  Find the capacity of the vessel.
13 (A) 120  (B) 150  (C) 90  (D) 75
Hence, the correct option is (D). (ii)  What is the least number of such additional
22. Two vessels contain alcohol and water in the ratio 3 : 4 replacements required, so that the vessel contains
and 65 : 79. In what ratio should the solution in the first less than 75% milk?
vessel be mixed with the solution in the second, so as to (A) 1  (B) 2  (C) 3  (D) 4
get a solution with alcohol and water in the ratio 4 : 5? Sol. Let the capacity of the vessel be x litres.
(A) 7 : 16 (B) 16 : 7
(i) quantity of milk in the vessel finally
(C) 36 : 85 (D) 85 : 36 2
Sol. Concentration of alcohol in the 1st mixture = 3/(3 + 4) ⎛ x−9⎞
=x ⎜ ⎟ = x – 17.1
= 3/7 ⎝ x ⎠
Concentration of alcohol in the 2nd mixture x2 – 18 x + 81 = x2 – 17.1x
= (65)/65 + 79) = (65/144) x = 90
Concentration of alcohol in the combined mixture Hence, the correct option is (C).
= 4/(4 + 5) = 4/9 (ii) let the number of further replacements be n.
If q1 and q2 are the quantities of mixture 1 and mixture ⎛ x−9⎞
n+ 2
75 x
2 that are combined, then, by the equation of alligation, ⎜ ⎟ ≤ 100
⎝ x ⎠
q1 ( 65 / 144 ) − ( 4 / 9 ) 1 / 144 63 7 x = 90
= = = =

q2 ( 4 / 9) − (3 / 7) 1 / 63 144 16 (0.9)n+2 ≤ 0.75  (17)
Hence, the correct option is (A). least value of n satisfying (17) is 1
23. There are eleven people in a group. The average age of Hence, the correct option is (A).
the oldest and the youngest is 11 years. If any one person 25. A local grocer mixed three qualities of coffee T1, T2,
leaves the group, the maximum and the minimum aver- and T3 at `56 per kg, `64 per kg, and `80 per kg in the
age of the remaining are 12 years and 11 years, respec- ratio 1 : 2 : 4. To 4 kg of this mixture, he added p kg of
tively. Find the average age of the entire group, in years. T1 and 4p kg of T3. The final mixture so formed is sold
(A) 9 (B) 10 for `87.60 per kg at 20% profit. Find p.
7 5 (A) 1/3 (B) 5/9 (C) 2/3 (D) 4/11
(C) 11 (D) 11
11 11
Sol. Let x kg be the quantity of T1 in the mixture.
Sol. Let the average of the ages of the 11 men be A years. Cost of the mixture of T1, T2 and T3
Let the ages of the oldest and the youngest be x years
and y years respectively. 56( x ) + 64( 2 x ) + 80( 4 x )
= = `72 per kg
If any one person leaves the group the maximum and 7x
minimum possible average age of the remaining occur Let y kg of T1 be added to this mixture. The cost of the
if the person aged y years and the person aged x years, final mixture
respectively, leave (72)( 4) + 56 y + 80( 4 y )
=
11A – y = (10) (11) = 110  (15) 4 + 5y
11A – x = (10) (12) = 120  (16) 87.60(100)
This also equals = 73
Adding (15) and (16), 100 + 20
22A – (x + y) = 230 288 + 376 y
x+ y 5 ⇒ = 73
= 11A = 11 4 + 5y
2 11 4
Hence, the correct option is (D). ⇒ =y
11
24. A vessel is filled to its capacity with pure milk. Nine
Hence, the correct option is (D).
litres are withdrawn from the vessel and replaced with
an equal amount of water. Nine litres of the mixture

Unit I_Chapter 6.indd 75 27/04/2017 14:19:55


Chapter 7 Time and Work
Hints/Solutions
Practice Problems 1 1 men → 1/150 (one day’s work)
10 days work = 10/150 = 1/15
1. x men → 120 days
i.e. 15 men for one work men needed = 15 × 5 = 75.
x + 10 men → 100 days
Hence, the correct option is (B).
x (120) = (x + 10) 100
8. Part of the tank filled in a hour by both taps
6x = 5x + 50 ⇒ x = 50.
1 1 5
Hence, the correct option is (B). = + =
10 15 6
2. Job = (15) (9) = 135 mandays = (135) (C) = women \ The tank will be filled in 6 hours by them. In this
days. Time taken by 15 women to complete the job =
6 3
405 time X can fill = th of the tank.
or 27 days. 10 5
15 Hence, the correct option is (D).
Hence, the correct option is (C).
9. Part of the tank filled in a hour by both pipes
3. Let the job be 1 unit.
1 1 1
Let the times taken by A, B and C the complete it be 3x = − =
9 18 18
hours, 4x hours and 6x hours respectively.
\ The tank will be filled in 18 hours by them.
Parts of the job completed by A, B and (in a hour are
Hence, the correct option is (C).
1 1 1
units, units and units respectively. 1
3x 4x 6x 10. In one day Raj can build th of the wall.
18
1 1 1
Required ratio = : : = 4 : 3 : 2. 1
3x 4x 6x Kiran can build th of the wall.
30
Hence, the correct option is (B).
4. Work done by Amar one day = 1/12 1
To complete the work Raj takes 9 days.
Work done by Bharat in 1 day = 1/24 2
Work done by Charu in 1 day = 1/24 1
Remaining work = .
1 1 1 1 2
Work done by 3 of them in one day = + + =
12 24 24 6 In one day both can together complete
\ They take 6 days to finish the work.
1 1 48 4
Hence, the correct option is (B). + = =
18 30 18 × 30 45
5. Per day
Adam = 1/25 1 1 ⎛ 45 ⎞ 45
To complete the work, it takes them ⎜ ⎟ =
2 2⎝ 4 ⎠ 8
Adam + Chris = 8/75 days.
8 1 1 45 5
\ Chris = − = Total number of days = + 9 =14 days.
75 25 15 8 8
\ Chris takes 15 days.
Hence, the correct option is (B).
Hence, the correct option is (D).
11. Rate of work and time taken to do a work are in inverse
6. P and Q take less time to complete the job compared to proportion.
Q and R. \ R is slower than P. We should new compare
Hence, Kaushik is one and a half times more efficient
R and Q (not P and Q).
than Ravi implies, Ravi takes one and a half time more
P and Q take less time to complete the job compared to time than Kaushik, to do the same work.
P and R. \ R is slower than Q.
Kaushik takes 20 days.
\ R is the slowest of the 3 workers.
Hence Ravi takes 20(1 + 1.5) = 50 days to do the same
Hence, the correct option is (C). work.
7. In 10 days, amount of work completed by both together
Men Work Days
10 10 70 7
10 1 15 = + = = th of the work.
20 50 100 10
X 5 10
Hence, the correct option is (D).

Unit I_Chapter 7.indd 76 27/04/2017 14:20:04


Chapter 7  •  Time and Work  |  1.77

12. Initially let there be x men who take y days. 17. Let x boys be required to work, then
xy = 56 (15) ⎛1⎞ 1 1
(x - 1) (y + 1) = xy(16) x ⎜ ⎟ + + = 1
⎝8⎠ 2 4
\x-y=1 (17)
From (15) and (17), as 56 = 8 × 7, x +6
= 1, x = 2.
x = 8 and y = 7. 8
Hence, the correct option is (B). Hence, the correct option is (C).
13. Let Peter and Pan take 8 and 12 days respectively to do 18. Number of bolts produced by type A in 10 days
1 unit work. Let the job mentioned in the problem be 10
x units of work. So, they take 8x days and 12x days to = (1500) = 500; Remaining = 1000
30
do x units of work respectively. Both together can com- Type B produces 1000 bolts in 10 days.
(8 x ) × (12 x )
plete x units of work in = 4.8x days ⇒ 3000 bolts are produced in 30 days by type B.
20 x
Together they can produce (3000 + 1500)
Given that 4.8x = 36 ⇒ x = 7.5
= 4500 bolts in 30 days.
\ To do x units of work, Pan working alone, takes (12)
(7.5) = 90 days. Hence, the correct option is (B).
Hence, the correct option is (D). 19. Let the required cost be `x
14. The total work completed is 10 cows = 20 sheep
⎛1⎞ ⎛ 1 1 1⎞ \ 10 cows + 40 sheep = 60 sheep
28 ⎜ ⎟ + ⎜ + + ⎟ 4 = 1 ⇒ z = 40.
⎝ z ⎠ ⎝ 36 45 z ⎠ 20 cows + 10 sheep = 50 sheep
Hence, the correct option is (A). Let the cost of grass for each sheep be `y/day.
15. Parts of the job done by P and Q, Q and R and P and R (60y) (20) = 900
1 1 1 3
in a day are , and respectively y=
24 20 30 4

1  x = (50y) (18) = 675.
\P+Q= (18)
24 Hence, the correct option is (B).
1 20. The cost of grass for 20 cows and 30 sheep for 30 days
Q+R= (19)
20 = `720.
1 Given 30 sheep eat double the grass eaten by 20 cows.
P+R= (20)
30 ⇒ Cost of grass eaten by 30 sheep in 30 days
Adding (18) and (19) and subtracting (20) from the 2
= × 720 = `480
7 3
sum, 2Q = The cost of grass eaten by 1 sheep in one day
120
480 8
7 = = .
Q= 30 × 30 15
240
1 The cost of grass eaten by 20 sheep in 15 days
From (18), P =
80 8 × 20 ×15
= = `160.
1 15
From (19), R = Hence, the correct option is (B).
48
240
\ P, Q and R take 80 days, days and 48 days to 21. Money received for one day’s work
7
complete the job respectively. 13500
= = 300
Hence, the correct option is (C). 45
16. Each jump followed by a slip would have enabled it 3
Amount of money received by Gagan = × 300
to cover 0 × 4 m. In 198 jumps, it would have covered 4
= 225 per one day.
(0 × 4) (198) = 79 × 2 m. In the next jump, it would have
come out of the well. For 10 days, Gagan receives 225 × 10 = `2,250.
Hence, the correct option is (D). Hence, the correct option is (B).

Unit I_Chapter 7.indd 77 27/04/2017 14:20:06


1.78  |  Unit 1  •  Quantitative Aptitude

22. Pipe A can fill the tank in 20 hours. Pipe B in 30 hours. Note: The time taken to complete the job would be the
Total time for both A and B together to fill the tank same if Ramesh starts the job. Whenever the part of
20 × (30) the job completed by two workers working on alternate
= = 12 hours. days in the first two days
30 + 20
1
But it took 3 hours more i.e. 15 hours in total. = where n is an integer, the time taken to complete
n
Let the number of hours in which the leak can empty
the tank be x. the job = 2n days.
1 1 1 60 Hence, the correct option is (C).
⇒ = − ⇒x= = 60 hours.
x 12 15 5−4 11
25. Part of the job completed in the first 2 days =
Hence, the correct option is (A). 60
23. Ratio of work done by Gokul, Govardhan and Ganesh 11
After S cycles of 2 days, part of the job completed = .
in equal durations of time 12
1 1 1
= : : respectively i.e., 6 : 3 : 2 1
10 20 30 This is completed in 10 days. Remaining part =
12
3 On the 11th day, Rakesh would work. He would com-
⇒ Govardhan gets (6600) = `1,800.
11 5
plete the remaining part in th of that day.
Hence, the correct option is (A). 6
24. Part of the job completed in the first two days 11 1 1 1 11
\ Total time = = + + = .
1 1 ⎛ 1 ⎞ 2c a b c 20
= =   job = 20 ⎜ ⎟
c 10 ⎝ 20 ⎠ Hence, the correct option is (B).
\ time taken to complete the job = 20 (2) = 40 days

Practice Problems 2 ol. Let x be the number of men initially engaged.


S
Solutions for questions 1 to 25: Let d be the number of days for which they worked.
Hence, x men completed half of the work in d days
1. A man builds one-eighth part of a wall every day. Out
(21)
of the length of the wall built per day, 20% falls off at
the end of the day till the wall is completely built In Because 2x men joined, total number of working
how many days can he complete the construction of the men = x + 2x = 3x.
wall? This new group completes the remaining work 6 days
1 4 earlier; i.e., 3x men complete half of the work in (d – 6)
(A) 8 (B) 10 (C) 9 (D) 9
5 5 days(22)
1 As the product of men and number of days is the same
Sol. Portion of wall built per day = th = 12.5%
8 in both cases, being equal to work done,
20% of this (i.e. 2.5% of the wall) falls off. (x) (d) = 3x (d – 6);
Therefore 10% of the wall is completed every day. ⇒ d = 3 (d – 6), 2d = 18 (23)
To determine whether the whole of the 10th day is But, 2d is the duration required by x men to complete
needed for work, consider work done in 9 days. the work.
In 9 days 90% of the wall is built. On the 10th day, one- Hence, the correct option is (A).
1 / 10 4 3. Raman can do a piece of work in half the time taken
tenth has to be built. So, he takes = = days
1/ 8 5 by Kapil. Sunil can do the same work in one-third of
4
So, he takes 9 days to construct the wall. the time taken by Raman. All three of them work on it
5
for 30 days after which Kapil leaves. Sunil and Raman
Hence, the correct option is (D).
complete the remaining work in 18 more days. How
2. A group of men are building a wall. After half the wall many days would it take for Raman alone to complete
has been built, double the number of men join the origi- the total work?
nal group. The wall gets completed 6 days earlier than (A) 52 (B) 414 (C) 138 (D) 207
scheduled. What is the total number of days the initial
Sol. Let the number of days taken by Kapil be x.
group of men would have taken to complete the wall?
(A) 18 (B) 16 (C) 14 (D) 12 15 3
Work done by Kapil in one day = =8
1 4

Unit I_Chapter 7.indd 78 27/04/2017 14:20:08


Chapter 7  •  Time and Work  |  1.79

2 Only Choice (B) violates this condition.


Work done by Raman in one day =
x Hence, the correct option is (B).
2×3 6 6. Vivek, Rameshwar, and Bhuvan divide a work amongst
Work done by Sunil in one day = =
x x themselves in the ratio of 2 : 3 : 5. Their rates of work
Work done by 3 of them together in 30 days. are in the ratio 1 : 2 : 3. It takes Vivek 12 days to com-
plete his part. What is the amount of work completed
30 2 × 30 6 × 30 270
= + + = by them in 8 days from the start?
x x x x (A) 29/40 (B) 11/45
1 (C) 4/5 (D) 31/45
Work done by Sunil and Raman in 18 days =
24 Sol. From the ratio (2 : 3 : 5),
270 144
Total work = + = 1 ⇒ x = 414 1 3
x x Vivek does = of work in 12 days.
a 10
x
\ Time taken by Raman = i.e. 207 days. 10
2 ⇒ Vivek does full work in 12 × i.e., 60 days.
2
Hence, the correct option is (D). From the ratio 1 : 2 : 3, it can be decided that Rameshwar
4. A group of 50 salesmen plan to achieve their target for does in 30 days. Bhuvan does in 20 days.
the next 30 days by working 12 hours a day. Due to
various reasons, they put in only 10 hours a day for the In 8 days they complete
first 15 days. Now, if 10 men leave and the rest con- 8 8 8 11 1 1 1 11
tinue working for only 10 hours a day, how many days + + = = + + = .
60 30 20 6 a a b c 20
more than the initially estimated time will they require
Hence, the correct option is (C).
to meet their target?
1 3 1 3 7. A certain structure has to be erected. The workers erect-
(A) 11 (B) 12 (C) 13 (D) 13
4 4 4 4 ing the structure have to work a fixed number of hours
Sol. Total work = 30 (12) (50) man hours = 18000 man hours. each day. They would be paid fixed lab our wages for
Amount of work done in 15 days putting 10 hours a day each hour of work. If the labor wage per hour of each
= 15 (10) (50) = 7500 man hours. worker was one-sixth more and the number of hours of
1
Work remaining = 18000 - 7500 = 10,500 man hours. work th less, the total wage of the workers would be
35
Number of men remaining = 40 ______.
Number of hours = 10 per day. 2 1
10500 1 (A) th more (B) th more
Number of days needed = = 26 days. 15 5
40 (10) 4
1 2
1 (C) th more (D) th less
\ They require 11 more days to complete the work. 15 15
4
Hence, the correct option is (A). Sol. Let the number of days that the workers have to work
to complete the erection of the structure be D. Let the
5. P can complete a job in 27 days. He starts it and after
hourly wage be W rupees per worker. Let us say they
three days Q joins him. They work together for six days.
work H hours per day. Let the number of workers be N.
P then leaves and R takes his place. Q and R complete
the job in twelve more days. If Q takes at most 54 days Total wages of the workers presently = D(N W H) = N
to complete the job, then which of the following cannot D W H.
be a possible value of the number of days taken by R to If the changes mentioned take place, the new total
complete it? wages will be
(A) 36 (B) 34 (C) 40 (D) 38 ⎛ 7W ⎞ ⎛ 34 H ⎞ 17
ND ⎜ ⎟ ⎜ 35 ⎟ i.e., 15 N D W H.
Sol. P works for a total of 9 days. Q works for a total of 18 ⎝ 6 ⎠⎝ ⎠ 
days. R works for a total of 12 days. Let the times taken 2
by Q and R to complete the job be q days and r days This is th more than the present total wages of the
respectively. 15
workers.
9 18 12 Hence, the correct option is (A).
+ + =1
27 q r
8. Pradeep can work 2/3 times as fast as Abishek and
q ≤ 54 Antony together. Pradeep and Antony together can
\ r ≥ 36 work twice as fast as Abishek. If Antony alone takes

Unit I_Chapter 7.indd 79 27/04/2017 14:20:11


1.80  |  Unit 1  •  Quantitative Aptitude

45 days to complete a job, how long (in days) would 1 20


Pradeep and Abishek individually take to complete the (A) (B)
4 47
job, respectively?
(A) 30, 36 (B) 36, 30 1
(C) (D) None of these
(C) 45, 60 (D) 60, 45 3
1
Sol. Let the work done by Pradeep, Abishek and Antony in Sol. P’s rate = (Combined rate of P, Q and R)
one day be a, b and c respectively. 4
2 1
Then, a = (b + c)(24) \ P must have done th of the job.
3 4
a + c = 2b (25) Hence, the correct option is (A).
1 11. Machines P, Q, and R can do a piece of work in 20, 30,
c = (26)
45 and 60 days, respectively. Machines P, Q, and R start
2
From (24), 3a - 2b = 2c  or  3a - 2b = the work together. Machine P goes out of order after
1 45
and from (25), 2b - a = 5 days. After three more days machines Q and R also
45 go out of order. Machine P got repaired by then and
3 1 1 it completes the remaining work. What portion of the
Adding, we get \ 2a = a= and b =
45 30 36 total work did machine P do?
So, Pradeep takes 30 days and Abishek takes 36 days. (A) 3/4 (B) 11/20 (C) 7/20 (D) 3/5
Hence, the correct option is (A). 1
Sol. In one day, Machine P does th of the work.
20
9. Jadeja, Bhangar, Balaji, and Dravid can do a piece of
work in 8, 16, 32, and 64 days, respectively. Dravid 1
Machine Q does th of the work
starts the work and Balaji joins him after one-fourth 30
of the work is done, Bhangar joins them after half the 1
work is done and Jadeja joins them after three-fourth Machine R does th of the work
60
work is done. How many days does it take to complete
Together in 5 days they complete
the work?
2 24
(A) 30 (B) 32 (C) 25 (D) 24 5 5 5 (1 + 2 + 3) 1
35 35 + + = ( 5) = portion of the work.
20 30 60 60 2
Sol. Dravid starts the work and completes 1/4th work in 3 3 3+ 6 9
16 days. Balaji joins and together they complete 1/4th In 3 days Q and R complete + = =
work more. For them to complete 1/4th it takes 30 60 60 60
1/ 4 16 30 9 21
= days. Remaining = 1 - − =
1 / 64 + (1 / 32) 3 60 60 60
5 21 36 3
Then Bhangar joins and Dravid, Balaji and Bhangar Portion of work done by P = + = = .
complete another 1/4th of the work. For them to com- 20 60 60 5
plete 1/4th it takes Hence, the correct option is (D).
1/ 4 16 12. A man started a job. Starting from the second day, each
= .
1 / 64 + 1 / 32 + 1 / 16 7 day a new man joined with which the capacity of each
man doubled. The job was completed in 6 days. On
Then Jadeja joins and all four of them complete the
which day will the job be completed if the joining of a
remaining (1/4)th work.
new man on a day results in each man working at thrice
For them to complete 1/4th, it takes the rate as he did on the previous day?
1/ 4 16 (A) 6th (B) 5th (C) 4th (D) 3rd
= days.
1 / 64 + 1 / 32 + 1 / 16 + 1 / 8 15 Sol. Let the work done by Rohit on the first day be 1 unit
Total number of days Job = 1 + 2 (2) + 3 (D) + 4 (8) + 5 (16) + 6 (32) = 312
⎛ 1 1 1⎞ 24 units.
= 16 ⎜1 + + + ⎟ = 24 days. If the men worked at twice the rate, part of the job com-
⎝ 3 7 15 ⎠ 35
pleted in the first 4 days = 1 + 2 (C) + 3 (9) + 4 (27) =
Hence, the correct option is (D). 142 units. On the 5th day, 5 (81)
10. The ratio of the rates of doing work of P, Q, and R is = 405 units can be done.
3 : 4 : 5. If they completed a job working together, what \ The job will be completed on the 5th day.
part of it did P complete? Hence, the correct option is (B).

Unit I_Chapter 7.indd 80 27/04/2017 14:20:13


Chapter 7  •  Time and Work  |  1.81

13. Anil can complete a piece of work in 6 days and Mukesh fraction of the time taken by a single large pump, will
can complete it in 8 days each working alone. Consider they take to fill the tank?
two cases in which they work on alternate days. The (A) 1/6 (B) 1/7 (C) 1/8 (D) 1/9
first case when Anil starts and the second when Mukesh Sol. Let the rate at which each large pump fills the tank be
starts. What is the difference of the number of days l litres/hr.
taken in the two cases?
Rate at which each small pump fills the tank
1 1 3
(A) (B) (C) 1 (D) 1 2
2 4 4 = l litres/ hr.
Sol. Case 1: Anil starts 3
1 Rate at which six small pumps and three large pumps
Anil 2 days
6 ⎛2 ⎞
fill the tank = 6 ⎜ l ⎟ + 3l = 71 litres/hr.
1 ⎝3 ⎠
Mukesh
8 th
1
7 \ They will take the time taken by a single large
In a period of 2 days work is done pump to fill it. 7
24
21 Hence, the correct option is (B).
In 3 such time periods i.e. 6 days
24 16. Two pipes P and Q can fill a cistern in 12 and 18 hours
7 1 respectively. Both the pipes were opened at 10:00 a.m.
i.e. work is done. Remaining work is and it is
8 8 and the cistern was full at 6:00 p.m. What could be the
6 3 minimum possible duration for which one of the pipes
done by Anil in = . must have been closed during that interval?
8 4
1
3 (A) 4 hours (B) 2 hours
Hence a total of 6 days. 2
4
4
By a similar calculation, if Mukesh starts, a total of (C) 2 hours (D) hours
1 3
7 days are required. Difference = days. Sol. The time from 10:00 a.m. to 6:00 p.m. = 8 hours.
4
Hence, the correct option is (B). So, one of the pipes worked for 8 hours and the other
pipe worked for x hours.
14. Anwar would have to incur `600, `900, and `1200 as
the expenses if he got a job done by A, B, and C, respec- 8 x
  + = 1 (27)
tively. The daily wages of A, B, and C are `100, `60, 12 18
and `40, respectively. Find the cost to Anwar of getting x 8
the job done by all three of them (in `). or + = 1 (28)
12 18
(A) 600 (B) 800 (C) 750 (D) 900
We get x = 6 from equation (27)
600
Sol. Time in which A can complete the job = = 6 days. So, pipe Q was closed for 2 hours or
100
900 ⎡ 8 ⎤ 20
From Eqn (28), x = 12 ⎢1 − ⎥ =
Time in which B can complete the job = = 15 days ⎣ 18 ⎦ 3
60
1200 ⎡ 20 ⎤ 4
Time in which C can complete the job = 30 days So, pipe P was closed for ⎢8 − = hrs
40 ⎣ 3 ⎥⎦ 3
Part of the job which can be done by A, B and C in a day 1
Minimum possible time = 1 hours.
1 1 1 4 3
= + + = Alternate method:
6 15 30 15
15 Part of the tank filled, in 1 hour, by the two pipes
\ They would take days to complete it. 1 1 5
4 = + = th part.
⎛ 15 ⎞ 12 18 36
Cost to Anwar = ⎜ ⎟ (100 + 60 + 40 ) =` 750 . 36
⎝ 4⎠ Hence, time required to fill = = 7.2 hour.
5
Hence, the correct option is (C).
Actual time taken to fill = 6:00 p.m. – 10:00 a.m.
15. Six small pumps and three large pumps are fitted to a
= 8 hours.
tank. Each small pump works at two-third the rate of
each large pump. If all the pumps work together, what Extra time taken 8 – 7.2 = 0.8 hours

Unit I_Chapter 7.indd 81 27/04/2017 14:20:17


1.82  |  Unit 1  •  Quantitative Aptitude

Extra time was needed because one pipe was closed for 19. A piece of work when done by a man, a woman, and a
some time. child costs `720, `810, and `1080, respectively. Their
Hence, part filled by 2 pipes in 0.8 hours = Part not respective daily wages are `60, `45, and `30. If a family
filled due to closure of one tap consisting of a husband, wife, and their child is engaged
5 1 to complete this work, how much would it cost?
i.e. Part not filled = 0.8 × = th part. (A) `1080 (B) `780 (C) `810 (D) `60
36 9
Minimum time of closure is to be found out; Sol. Number of days required:
720
⇒ The faster pipe was closed. by a man = = 12
60
Rate of filling of the faster pipe = 1/12th/hour.
810 1080
Hence duration of closure = (1/9)/(1/12) = (12/9) by a woman = = 18; and by a child = = 36
45 30
= (4/3) hours.
If 1 man, 1 woman and 1 child work together,
Hence, the correct option is (D).
1 1 1 1
17. A pipe can fill a tank in 4 hours, while a leak which is + + work will be done in 1 day = th
at one-fourth the height of the tank can empty upto that 12 18 36 6
part in 2 hours. If both are operated simultaneously and \ 6 days will be required.
initially the tank is full, then when will it be one-fourth Cost = 6 × (60 + 45 + 30) = `810.
full? Hence, the correct option is (C).
1
(A) 2 hours (B) 2 hours 20. There are 5 milk tanks in a dairy farm—A, B, C, D,
3
1 and  E. Each contains 3300 litres of milk. Milk is
(C) 1 hours (D) 6 hours pumped from one tank to the other as follows.
2
From A to C @ 130 litres/minute
3 From B to E @ 100 litres/minute
Sol. To fill th part, the filling pipe takes 3 hours.
4 From B to A @ 80 litres/minute
3 From E to C @ 210 litres/minute
To empty th part, the emptying pipe takes 2 hours. From D to B @ 30 litres/minute
4
\ When both are operated simultaneously, they will empty From C to D @ 250 litres/minute
3 1 1 1 Which tank would be the first to get emptied? How
th part of the full tank in − = long will it take to get emptied?
4 2 3 6
i.e., 6 hours. (A) D, 15 minutes (B) B, 22 minutes
(C) A, 66 minutes (D) E, 30 minutes
Hence, the correct option is (D).
Sol. Effective out flow from A = -130 + 80 = -50 litres/
18. N taps numbered from 1 to N are fitted to a tank. The minute.
rate at which the nth tap, for n = 1, 2, …, N fills the
tank, equals the sum of the rates of all the taps num- Effective outflow from B = -100 - 80 + 30 = -150
bered below it. If the sixth tap can fill it in 80 ­minutes, litres/minute.
find the time in which the ninth tap can fill it (in Effective outflow from C = -250 + 130 + 210 = +70
minutes). litres/minute. Effective inflow occurs into C).
(A) 10 (B) 5 (C) 20 (D) 40 Effective outflow from D = -30 + 250 = 220 litres/min-
Sol. Let the rate of filling of the first tap be x litres/hr ute (effective inflow occurs into D). Effective out flow
Let the rate of filling of the nth tap be denoted by tn from E = -210 + 100 = - 110 litres / minutes.
t2 = x Effective out flow occurs from A, B and E only.
t3 = 2x Maximum effective outflow occurs from B.
t4 = 4x = 22x \ B would be the first to get emptied time taken to
empty
t5 = 8x = 23x
3000
\ tn = 2n - 2x B = = 22 minutes.
150
t 6 23 x 1 Hence, the correct option is (B).
= =
t 9 26 x 8 21. P and Q are filling pipes which can fill a tank in 15

1
\ Time taken by the ninth tap = (80 ) = 10 minutes . and 20 minutes, respectively. R is an emptying pipe
8 which can empty the full tank in 30 minutes. The three
Hence, the correct option is (A). pipes are operated continuously one after the other in

Unit I_Chapter 7.indd 82 27/04/2017 14:20:18


Chapter 7  •  Time and Work  |  1.83

the order of P, Q, and R, each being kept opened for 23. The efficiency of a man is reduced by half every two
2 ­minutes until the tank is filled. After how much time hours. At maximum efficiency, he could have com-
will the tank be full? pleted the job in 150 hours. How many hours does it
2
(A) 30 minutes (B) 32 minutes take him to complete the job, if his efficiency becomes
3 maximum after every 8 hours, and then reduces as
(C) 36 minutes (D) 34 minutes mentioned above?
Sol. In one cycle of 6 minutes the part of the tank that is (A) 300 (B) 360 (C) 320 (D) 600
2 2 2 10 1 Sol. Given that the man can complete the job in 150 hours
filled = + − = =
15 20 30 60 6 at maximum efficiency. i.e. at maximum efficiency he
In order to decide whether the 6th cycle is needed in 1
full or in part, consider the situation after 5 cycles. can do of the work per hour.
15
1 5 (He works each 2 hours with the same efficiency)
5 × = th of the tank is filled, in the first five cycles.
6 6 \ In 8 hours he can do :
5 × 6 = 30 minutes are over.
⎡ 1 1 1 1 ⎤
1 2 ⎢ + + + ⎥
Part of tank to be filled = th. ⎣150 2(150) 4(150) 8(150) ⎦
6
After next 2 minutes, the part of the tank that is empty 2 × 15 1
= = th of the work.
1 2 1 8 × 150 40
= − = , as pipe P fills (2/15)th part.
6 15 30 Hence, total number of hours = 8(40) = 320.
1 2 Hence, the correct option is (C).
Q can fill this part in 20 × = minutes,
30 3 24. P, Q, and R are three machines. They produce elec-
⇒ 6th cycle is not required in full. tronic gadgets. The ratio of the rates of P, Q, and R is
2 2 3 : 4 : 5. P worked for 6 days, Q worked for 8 days, and
\ The tank is filled in 30 + 2 + = 32 minutes. R worked for 10 days. They manage to produce 400
3 3
gadgets. How many gadgets would they have produced
Hence, the correct option is (B). in the same time if P’s rate doubled and Q’s rate tripled?
22. Two taps can normally fill a cylindrical tank in 16 hours (A) 546 (B) 728 (C) 637 (D) 910
and 48 hours. But a leak which can empty the tank in Sol. Let the efficiencies of P, Q and R be 3x gadgets/day, 4x
24 hours is present at three-fourth of the tank’s height gadgets/day, 5x gadgets/day.
from the base. Find the time taken to fill the tank if the
(3x) (6) + (4x) (8) + (5x) (10) = 400 x = 4
taps are opened simultaneously (in hours).
(A) 12 (B) 15 (C) 18 (D) 21 Required number of gadgets
3 = (6x) (6) + (12x) (8) + (5x) (10) = 728.
Sol. Required time = Time taken to fill the bottom th
1 4 Hence, the correct option is (B).
+ time taken to fill the top th 25. P and Q can complete a job in 12 days. Q and R can
4
The leak will not affect the filling of the bottom complete it in 20 days. R and P can complete it in
10 3 cm 15 days. Find the time taken by P, Q, and R working
together, to complete it.
\ Time taken to fill the bottom part is 10 3 cm (time (A) 10 days (B) 20 days (C) 5 days (D) 8 days
taken by the taps to fill the tank). Time taken to fill the
Sol. Parts of the job, which can be completed, by P and Q,
top part is 5 3 cm (time taken by the taps to fill the
tank along with the leak) 1 1 1
Q and R and R and P in a day are th, th and
Time taken to fill the bottom part th respectively. 12 20 15
3 ⎛ (16 )( 48) ⎞ 1 1 1
= = 9 hours \ 2 P’s, 2 Q’s and 2 R’s can complete + +
4 ⎜⎝ 16 + 48 ⎟⎠ 12 20 15
1
Part of the tank which can be filled by the taps and the = th of the job in a day.
5
leak each hour = 5 3 cm 1
1 1 \ P, Q and R can complete th of the job in a day.
\ Time taken to fill the top th = ( 24) = 6 hours 10
4 4
\ They can complete the job in 10 days.
\ Required time = 15 hours.
Hence, the correct option is (A).
Hence, the correct option is (B).

Unit I_Chapter 7.indd 83 27/04/2017 14:20:21


Chapter 8 Time and Distance
Hints/Solutions
Practice Problems I 9. Relative to the boy the speed of the train is 99 + 9 i.e.,
108 km/hr. Hence, the time it takes to cross him
1. (a) 36 km/hr × 5/18 = 10 m/s
750
Hence, the correct option is (A). = = 25.
5
(b) 12.6 × 5/18 = 3.5 m/s 108 ×
8
Hence, the correct option is (A). Hence, the correct option is (D).
(c) 252/35 × 5/18 = 2 m/s 10. When Eswar finishes the race, Girish would have run
Hence, the correct option is (C). 200 – (10 + 10) = 180 m.
2. 6 m/s × 18/5 = 108/5 km/hr 200 10
\ Ratio of Eswar’s and Girish’s speeds = =
d = s × t ⇒ 108/ 5 × 15/4 = 81 km 180 9
Hence, the correct option is (C).
Hence, the correct option is (A).
11. Ganesh beats Harish by 10 m or 2 seconds.
3. As speed and time are inversely proportional, if speed
decreases to 5/6th, time taken will be increasing to 6/5th \ Harish must have taken 2 seconds to run the final 10 m.
of original time. 10
\ Harish’s speed = = 5 m/sec
6t 2
\ – t = 10 ⇒ t = 50 minutes. Hence, the correct option is (A).
5
Hence, the correct option is (A). 12. Akbar gives Birbal a start of 2 s and Birbal covers 10 m
270 in these 2 s.
4. Time taken by them to meet =
Relative speed \ Birbal’s speed is5 m/sec.
270 \ He takes 18 seconds, more to cover the 90 m.
= = 3 hours
50 + 40 \ Akbar covered 100 m in 18 seconds.
\ The meeting time is 12:00 p.m 100 5
His speed = m/sec = 5 m/sec
Hence, the correct option is (B). 18 9
5. Distance that X would have traveled by 10:00 a.m. Hence, the correct option is (B).
= 30 k.m. Distance between the cars at 10:00 a.m. 13. Let the time taken by R to run the race is t seconds.
⎛ 150 ⎞ Time taken by Q to run the race = (t + 30) seconds.
= 150 k.m. Meeting time = ⎜ ⎟ hours after
⎝ 30 + 20 ⎠ Time taken by P to run the race = (t + 50) seconds.
10:00 a.m. i.e. 1:00 p.m. \ P beats R by 50 seconds.
Hence, the correct option is (C). Hence, the correct option is (C).
Total distance 14. Let the speeds of A, B and C be a m/sec, b m/sec and c
6. Average speed =
Total time m/sec respectively.
XY = (4) (90) or 360 km. a 100 10
= =
Hence, the correct option is (B). b 100 − 10 9

7. Let x km be the distance between Hyderabad and b 100 5
Tirupathi. = =
x+x c 100 − 20 4
Average speed =
x x a ⎛ a ⎞ ⎛ b ⎞ 25
+ = =
60 90
c ⎜⎝ b ⎟⎠ ⎜⎝ c ⎟⎠ 18
2 (60) (90)
= = 72 kmph
60 + 90 25 − 18
\ A beats C by (100) = 28 m.
Hence, the correct option is (A) 25
8. The distance to be travelled by the train to cross a plat- Hence, the correct option is (B).
form of length 750 m is 750 + 650 i.e., 1400 m. At 72 15. Speed of Anand’s boat downstream = 8 kmph.
1400
km/hr. or 20 m/s. the train takes i.e. 70 seconds 40
20 Time taken = = 5 hours
to cross the platform. 8
Hence, the correct option is (C). Hence, the correct option is (A).

Unit I_Chapter 8.indd 84 27/04/2017 14:20:31


Chapter 8  •  Time and Distance  |  1.85

⎛ 600 600 600 ⎞ = 9 kmph i.e., relative speed.


16. Time = LCM ⎜ , , ⎟
⎝ 10 15 20 ⎠ 500
\ Time = = 200 sec
= LCM (60, 40, 30) 5
9x
= 120 seconds 18
But, if the time is required from the start of the cat’s run,
Hence, the correct option is (A).
1 minute has to be added. i.e. 200 + 60 = 260 seconds.
17. The relative speed does not change when the two
Hence, the correct option is (C).
exchange their speeds. \ Both will reach the starting
point simultaneously. 22.
P Q
Hence, the correct option is (C).
vu A
18. By formula ( p + q ) = distance
v−u B
⎛ v x 8 ⎞ 24 When ‘B’ starts from city P towards city Q, the dis-
⇒ ⎜ ⎟ = 16
⎝ v − 8 ⎠ 60 tance which ‘A’ would already have covered = 54 × 3
= 162 km.
8v 2
⇒ x = 16 At 9 a.m., train B is separated from train ‘A’ by a dis-
v −8 5
tance of 162 km.
⇒ v = 5v – 40   ⇒ 40 = 4v  ⇒ v = 10 kmph
Train B overtakes train A after a time of
Alternate method:
162 162
d d = hrs = 9 hrs.
− = difference of times of travel ( 72 − 54 ) 18
s1 s2
In 9 hrs, the distance travelled by B = 72 × 9 = 648 km.
16 16 15 + 9 24 2 Distance from city Q, when they meet
− = = =
8 s2 60 60 5 = (1440 - 648) = 792 km

Solving, s = 10 kmph Hence, the correct option is (B).
Hence, the correct option is (A). 23. Let his forward and return speeds be x kmph and y
84 4 kmph respectively.
19. Time after which they meet = = hours
36 + 27 3 2xy
Average speed =
\The first person will cover 36 × 4/3 = 48 km in this x+ y
time.
2 xy x+ y
So, they will meet 48 km from P. Given, =
x+ y 2
Hence, the correct option is (D).
⇒ (x + y)2 - 4xy = 0
20. Let the speeds of the faster car and the slower car be
x kmph and y kmph respectively. ⇒ (x - y)2 = 0 ⇒ x = y
2xy
80 \ =x
Given, = 1 ⇒ x + y = 80(4) x+ y
x+y
1200
80 \ Forward speed = Average speed = = 100 kmph
= 4 ⇒ x - y = 20 (5) 12
x− y
Hence, the correct option is (A).
Solving (4) and (5), x = 50 24. Let the speeds of the cars leaving P and Q be p kmph
Hence, the correct option is (A). and q kmph respectively.
21. px = qy(6)
D 100 A A pz = qx(7)
The cat runs away from an initial point A for 1 minute Dividing (6) by (7),
at a speed of 24 kmph. x y
=
\ AC = 24 × 60 × 5/18 = 400 m   z x
Now, the dog is separated from the cat by a distance of x = yz .
100 + 400 (DA + AC) = 500 m and this lead has to be
covered at a speed of (33 – 24) Hence, the correct option is (C).

Unit I_Chapter 8.indd 85 27/04/2017 14:20:33


1.86  |  Unit 1  •  Quantitative Aptitude

30 1 1 1 1
25. Speed of the boat upstream = = 6 kmph As speed a , time taken ratio is : :
5 time 5 4 6
24 i.e., 12 : 15 : 10.
Speed of the boat downstream = = 8 kmph Given, 3 parts is representing 15 minutes, hence 2 parts
3
will represent 10 minutes.
⎛ 8+6 ⎞
Speed of the boat in still water = ⎜ ⎟ kmph = 7 km/hr Hence, the correct option is (B).
⎝ 2 ⎠
⎛ 8 −6 ⎞ 28. Distance travelled by Habib/Distance travelled by
Speed of the water current = ⎜ ⎟ =1 kmph Akram = 500/440
⎝ 2 ⎠
Hence, the correct option is (C). Let the length of the pace of Habib be x and that of
Akram be y.
26. Akbar completes 1 revolution in 40/5 = 8 sec
Ratio of speeds = ratio of distances covered in equal
Circumference/Akbar’s speed = Circumference/11
intervals of time = (5x/4y)
\Circumference = 88 m
\ 5x/4y = 500/440
Time taken to meet for the first time is x/y = 10/11
⎧ 88 88 ⎫ ⎧ 88 88 ⎫
LCM of ⎨ , ⎬ i.e., LCM of ⎨ , ⎬ Hence, the correct option is (A).
⎩ 22 − 11 11 − 7 ⎭ ⎩ 11 4 ⎭
i.e., LCM of {8, 22} = 88 29. If two or more runners have their speeds as a multiple
of the speed of a runner N the time taken by these run-
So, they meet for the first time 88 seconds after the start.
ners to meet for the first time at the starting point for
Hence, the correct option is (B). the first time is the time taken by the N to complete one
27. Let his usual time be x minutes. Let the distance be round.
d km As the speeds of Harish and Suresh and Multiples of
d km the speed of Girish, required time = Time taken by
Usual speed =
x minute Girish to complete 1 round = 1 minute.
4 Hence, the correct option is (A).
If Ashok traveled at th of his usual speed, his usual
5 30.
d 5 ←(8:00 a.m.)
speed, his time = = × minutes A
x ↓(9:00 a.m.)
4d 4
5x y
5 150 km
x − x = 15   x = 60
4
6 B
If Ashok traveled at th of his usual speed, his time
5 The distances covered by C and D are x and y respec-
d 5 tively. Since C starts early compared to D, he covers
= = x more distance by noon compared to D. Hence x > y.
6d 6
5x If speed of each car is s. x = 4s and y = 3s
1 x2 + y2 = (150)2 (by Pythagoras theorem)
He would be early by x = 10 minutes.
6 Taking x = 4s, y = 3s and solving for s we have,
The ratio of speeds of Ashok in three instances i.e.,
usual speed, 4/5th speed and 6/5th speeds, is s : 4s/5 : s = 30 kmph.

6s/5 i.e., 5 : 4 : 6. Hence, the correct option is (D).

Practice Problems 2 (A) 30 seconds (B) 120 seconds


(C) 180 seconds (D) 60 seconds
Solutions for questions 1 to 23:
ol. Let the original time taken be ‘t’. Since, his speed
S
1. Ram covers a distance of 100 m in a certain time. If he increases to 3 times his original speed, new speed = (3)
increases his speed to 3 times his original speed, the (original speed) ⇒ New time
time taken reduces by 40 seconds. Find the original Original time
time taken (to cover the same distance of 100 m). =
3

Unit I_Chapter 8.indd 86 27/04/2017 14:20:35


Chapter 8  •  Time and Distance  |  1.87

t Distances travelled by Anand and Ashok before meet-


Given, t - = 40
3 ing are x km and y km respectively. As they reached
their destinations simultaneously,
2t
⇒ = 40 ⇒ t = 60 sec. y x
3 = ⇒ y2 + 2y = x2 – 2x
x−2 y+2
Hence, the correct option is (D).
2 = x - y(9)
2. A person covered 50% of a certain distance at 15 kmph,
60% of the remaining distance at 18 kmph and the Solving (8) and (9), x = 6.
rest at 45 kmph. The total travel time of the person is Hence, the correct option is (D).
9.8 hours. Find the total distance covered. 5. A person travels one-third of a certain distance AB at x
(A) 60 km (B) 120 km kmph, one-fourth of the remaining distance at 2x kmph
(C) 150 km (D) 180 km and the remaining distance at 3x kmph. If his average
Sol. Let the total distance covered be 100 d km speed for the entire journey is (x + 2) kmph, then find
50 d 30 d 20d the total distance he covers.
+ + = 9.8 ⇒ d = 1.8 (A) 60 km
15 18 45
(B) 50 km
and 100 d = 180
(C) 40 km
Hence, the correct option is (D). (D) Cannot be determined
3. P and Q are two stations. Train A started from P towards Sol. From the given data, which is about the speeds of travel
Q at 6:00 a.m. at 90 kmph. At the same time, train B and the average speed, in terms of a variable x, the
started from R, an intermediate station 60 km from P, average speed can be determined. As no information
and travelled towards Q at 60 kmph. Train C started from about the total time is available, distance cannot be cal-
Q towards P at 7:00 a.m. at 120 kmph. All the trains culated. The same can be seen from the calculations
crossed each other simultaneously. Find PQ (in km). shown below.
(A) 300 (B) 360 (C) 330 (D) 390
Let the total distance AB be d.
Sol.
60 km x km ⎛ d ⎞ 1 ⎛ 2d ⎞ d − ⎛ d + 1 ⎛ 2d ⎞ ⎞
P R S Q ⎜ 3⎟ 4⎜ 3 ⎟ ⎜ ⎜ ⎟⎟
⎝ ⎠ ⎝ ⎠ ⎝ 3 4 ⎝ 3 ⎠⎠
Suppose the trains met at S. Let RS = x km Total time = + +
x 2x 3x
PS RS SQ
= = +1 d ⎛1 1 1⎞ d ⎛ 7 ⎞
90 60 120 = ⎜ + + ⎟= ⎜ ⎟
x ⎝ 3 12 6 ⎠ x ⎝ 12 ⎠
60 + x x
=
d
90 60 Average speed for the entire journey =
120 = x d⎛ 7 ⎞
RS SQ x ⎜⎝ 12 ⎟⎠
\ =2= +1 = 12x/7 = x + 2, 5x/7 = 2, x = 14/5 kmph
60 120
SQ = 120 km Total distance the person covers cannot be determined
as total travel time is unknown.
PQ = PR + RS + SQ = 300 km.
Hence, the correct option is (D).
Hence, the correct option is (A).
6. Rahul ran around a square plot ABCD of side 0.48 km
4. P and Q are two points, 10 km apart. Anand started once. He ran the distance AB, BC, CD, and DA at speeds
from P towards Q and at the same time, Ashok started of 4 kmph, 6 kmph, 4 kmph, and 6 kmph, respectively.
from Q towards P. They crossed each other after 1 hour.
After that, Anand reduced his speed by 2 kmph and (i) Find his average speed from A to C (in kmph).
Ashok increased his speed by 2 kmph. They reached (A) 2.4 (B) 3.6 (C) 4.8 (D) 7.2
their destinations simultaneously. Find Anand’s initial (ii) Find his average speed for the entire distance
speed (in kmph). (in kmph).
(A) 5.5 (B) 6.5 (C) 4.0 (D) 6.0 (A) 2.4 (B) 3.6 (C) 4.8 (D) 7.2
Sol. Let the speeds of Anand and Ashok before the meeting (iii) 
Find the time taken for the entire distance (in
be x kmph and y kmph respectively hours).
10 (A) 0.05 (B) 0.53
1= ⇒ x + y = 10 (8) (C) 0.4 (D) 0.27
x+ y

Unit I_Chapter 8.indd 87 27/04/2017 14:20:37


1.88  |  Unit 1  •  Quantitative Aptitude

Sol. AB = BC (A) 10 (B) 11


( 2 )( 4 )( 6 ) (C) 12 (D) 15
(i) Average speed = = 4 × 8 kmph
4+6 Sol. Let the speed of his son be x kmph.
Hence, the correct option is (C).
As Ashwin returned home 20 minutes early, he saved a
(ii) Using similar method as shown in (i), average travel of 10 minutes to school from the meeting point
speed of Rahul from C to A = 4 × 8 kmph i.e., and the return journey of 10 minutes. Distance he saved
same as that from A to C. travelling each way = Distance travelled by his son by
\ Average speed of Rahul for his journey walk. His son would have walked for 50 minutes.
= 4 × 8 kmph. ⎛ 50 ⎞ ⎛ 10 ⎞
( x ) ⎜ ⎟ = ( 55 ) ⎜ ⎟
Hence, the correct option is (C). ⎝ 60 ⎠ ⎝ 60 ⎠
Total distance x = 11 kmph
(iii) Time taken for his journey =
Average speed Hence, the correct option is (B).
( 0 ⋅ 48)( 4 ) 9. Two persons R and S had to meet at a place 36 km
= = 0 ⋅ 4 hours. from where they were at that moment. S, who was
4 ⋅8
Hence, the correct option is (C). given a head start of 2 hours, reached the destination
12 ­minutes earlier than R. If R travelled 18 kmph faster
7. A car had to travel a total distance of 600 km. After
than S, at what speed did R travel?
travelling a part of the distance, it developed an engine
problem. It travelled the remaining distance at 4/5th of (A) 25 kmph (B) 24 kmph
its usual speed. It arrived an hour late at its destination. (C) 30 kmph (D) 16 kmph
Had the engine problem occurred after it had travelled Sol. Since R has given S a start of 2 hours and arrives at the
150 km more, the car would have arrived half an hour destination 12 minutes after S, he takes 1 hr 48 min
earlier at its destination. Find the distance it travelled (9/5 hrs) less than S to reach the meeting point com-
without any problem (in km). pared to S. If the speed of R is x, we have,
(A) 150 (B) 250
36 36 9
(C) 200 (D) 300 − =
x − 18 x 5
Sol. Let the distance it traveled without any problem be
d  km. Distance it traveled at the reduced speed = Solving this equation, we have x = 30 kmph
(600 - d) km. Hence, the correct option is (C).
Let its usual speed by S kmph 10. M and N are two points that are 8 m apart. Anand and
d 600 − d 600 Ajay started simultaneously from M and N, respec-
Given, + = + 1 (10) tively. Anand moved towards N at 3 m/sec. Ajay moved
s 4 s
s towards M at 5 m/sec. After reaching their destinations,
5
both turned back at their original speeds towards their
d + 150 450 − d 600 1 starting points. Find the total distance (in m) that Ajay
⇒ + = +  (11)
S 4 s 2 would have travelled before crossing Anand for the
s
5 ­second time.
Subtracting (10) from (11), (A) 12 (B) 13
(C) 15 (D) 14
150 (150 ) s −1
⇒ − = Sol. Anand and Ajay would meet for the first time during
s 4s 2
their onward journeys. They would meet for the second
⇒ S = 75 time during their return journeys. Time taken by Anand
Substituting S in (10) or (11), 8
to travel from M to N = seconds.
d = 300. 3
Hence, the correct option is (D). 8 40
Distance travelled by Ajay in this time = (5) = m.
3 3
8. Everyday, Ashwin starts at 3.00 pm from his home to
pick up his son from school. They reach their house 40 16
\ He would be −8 = m from N when Anand
at 5.00 p.m. One day, school was over at 3.00 p.m. 3 3
Ashwin, not aware of this, started from home as usual. reached N.
He met his son on the way and they reached home 16 8
20 minutes earlier than usual. If the speed of his car is \ Distance between them would then be 8 − = m.
3 3
55 kmph, find his son‘s speed (in kmph).

Unit I_Chapter 8.indd 88 27/04/2017 14:20:39


Chapter 8  •  Time and Distance  |  1.89

Anand would cross Ajay in another and


⎛ ⎛ 5 ⎞⎞
8 L = 90 ⎜ S − ( 9 ) ⎜ ⎟ ⎟ (15)
3 1 ⎝ ⎝ 18 ⎠ ⎠
= seconds.
Relativespeed 3 Solving (14) and (15), we have
\ Ajay would have travelled a distance of S = 12 × 5
40 1 \ L = 900
+ (5) = 15 m
3 3 Hence, the correct option is (D).
when he crossed Anand for the second time. 14. A train 360 m in length, travelling at a uniform speed
Hence, the correct option is (C). overtook a car, travelling parallel to the tracks at
72 kmph and passed it in 12 sec. Forty eight minutes
11. In a kilometre race, Ram beats Shyam by one minute later, the train starts overtaking a cyclist and passed
and Shyam beats Tarun by 30 seconds. If Ram beats him in 9 sec. How much time after the train overtook
Tarun by 250 m in the same race, find the time taken by the cyclist would the car over take him?
Ram to run the race (in seconds). (A) 2 hrs 24 min 36 sec (B) 3 hrs 49 min 49 sec
(A) 180 (B) 270 (C) 360 (D) 330 (C) 3 hrs 12 min 9 sec (D) 2 hrs 24 min 27 sec
Sol. Let the time taken by Ram to run the race be t seconds. Sol.
Time taken by Shyam to run the race = (t + 60) seconds. T

Time taken by Tarun to run the race C
T
= (t + 90) seconds. Ratio of speeds of Ram and Tarun → T
C → →
C C
1000
1000 t
=
1000 − 250 1000 P Q R S
t + 90 Let the speed of the train be V m/s
4 t + 90 360
= Then = 12 ; V = 50 m/s
3 t V − 20
Similarly let the speed of the cyclist be V1m/s
t = 270.
360
Hence, the correct option is (B) =9
50 −V1

12. A train takes 30 seconds to cross a 200 m long platform V1 = 10 m/s
and 40 seconds to cross a 300 m long platform. Find its
Let P be the point where the train begins to overtake the
length (in m).
car Q be the point where the train completely overtake
(A) 150 (B) 225 (C) 200 (D) 100
the car R be the point where the car is, when the train
Sol. Let the length and the speed of the train be Lm and begins to overtake the cyclist.
Sm/ sec respectively
S be the point where the train begins to overtake the
L + 200 cyclist.
Given, = 30 ⇒ L + 200 = 30 S  (12)
S In 48 min, when it is just about to overtake the cyclist, it
L + 300 will cover (48) (60 (50) m, i.e. QS = 48(60) (50) m i.e.
Given, = 40 ⇒ L + 300 = 40 S  (13) QR = 48(60) (20) m
S
Subtracting (12) from (13), 100 = 10 S ⇒ 10 = S And the car will cover (48) (60) (20) m
From (12) or (13), L = 100 m. At this instant the distance between the car and the
cyclist RS is 360 + (48) (60) (50 – 20)
Hence, the correct option is (D).
= 360 + (48) (60) (30)
13. A train takes 2 minutes to overtake a cyclist traveling
at 18 kmph and one and a half minutes to overtake a This distance has to be covered with a relative speed of
cyclist traveling at 9 kmph. Find its length (in m). (20 – 10) = 10 m/s
(A) 1200 (B) 800 (C) 1500 (D) 900 360 + ( 48) (60) (30)
\Time required =
10
Sol. Let the length and the speed of the train be Lm and
Sm/sec respectively = 36 s + 144 min = 2 hr 24 min 36 s
⎛ As the time required is the time after the train overtook
⎛ 5 ⎞⎞
L = 120 ⎜ S − (18 ) ⎜ ⎟ ⎟ (14) the cyclist, it is 9 seconds less; i.e., 2 hr 24 min 27 s.
⎝ ⎝ 18 ⎠ ⎠
Hence, the correct option is (D).

Unit I_Chapter 8.indd 89 27/04/2017 14:20:40


1.90  |  Unit 1  •  Quantitative Aptitude

15. Tony and Harry begin to run in opposite directions on a the boat in still water is 8 kmph more than the speed
circular path of radius 35 m at 20 m/sec and 11 m/ sec, of the stream, find the time taken for the downstream
respectively, from the same point. What is the time taken journey.
by them to meet for the third time at the starting point? (A) 3 hours (B) 4 hours
(A) 11 min (B) 220 sec (C) 8 hours (D) 6 hours
(C) 8 min 40 sec (D) 325 sec Sol. Let the speed of the stream be y, then the speed of the
Sol. Radius = 35 m. boat in still water will be y + 8
⇒ Circumference = 2 × 22/7 × 35 = 220 m Upstream speed = y + 8 - y = 8 kmph.
Time taken for them to meet for the first time at the Hence, upstream time = (96/2)/8 = 6 hours
starting point is LCM [220/20, 220/11] Time for downstream = 9 - 6 = 3 hours.
= LCM [11, 20] = 220 sec Hence, the correct option is (A).
To meet for the third time at the staring point from the 19. Rahim and Saleem are competing in a 1000 m race.
start, they need 220 × 3 = 660 sec = 11 minutes Rahim gives Saleem a head start of 200 m, but his
Hence, the correct option is (A). speed is twice that of Saleem. When Rahim reaches the
16. P and Q start cycling simultaneously around a circular 600 m mark, Rahim reduces his speed by half but still
track 48000 m long with speeds of 10 m/sec and 30 m/ beats Saleem by 20 s. What is Saleem’s speed?
sec, respectively, in opposite directions. After every (A) 14 m/s (B) 5 m/s (C) 3.5 m/s (D) 7 m/s
crossing, P’s speed increases by 10 m/sec and Q’s speed Sol. During the time Rahim covers 600 metres, Saleem cov-
decreases by 10 m/sec. Find the time taken by them to ers 300 m, as Rahim’s speed is double that of Saleem.
cross for the third time (in seconds). During the time Rahim covers the balance 400 metres,
(A) 3600 (B) 7200 Saleem covers 400 m, as their speeds are equal. Saleem
(C) 4800 (D) 9600 had a head start of 200 m and he covers (300 + 400)
Sol. As P’s speed increases by the same amount as that by = 700 m. Hence, when Rahim is at finishing point,
which Q’s speed decreases, their relative speed remains Saleem is at 200 + 700 = 900 m metres. Saleem takes
unchanged. 20 seconds to cover the balance 100 m. Saleem’s speed
= 5 m/s.
\ Time taken by them to meet for the third time = 3
(Time taken by them to meet for the first time) Hence, the correct option is (B).
20. In a race of 500 m, L beats M by 40 seconds and beats
⎛ 48000 ⎞ N by 125 m. If M and N run a 500 m race, M beats N by
= 3⎜ ⎟ = 3600 sec.
⎝ 10 + 30 ⎠ 40 seconds. Find the time taken (in seconds) by M to
Hence, the correct option is (A). run the race.
17. Akbar takes a certain time to go downstream on a river (A) 160 (B) 240 (C) 280 (D) 320
between two villages. He takes 4 times as much time Sol. Let the time taken by L to run 500 m be tL.
to cover the same distance upstream. What is the speed Time taken by M to run 500 m = tL + 40
of the stream, if the distance between the villages is If M and N run a 500 m race, time taken by N = tL + 40
10 km, which can be covered in 2 hours if the boat trav- + 40 = tL + 80
els in still water?
Since L beats N by 125 m, and as they run for the same
(A) 2 kmph (B) 5 kmph
time; ratio of speeds of L and N will be
(C) 3 kmph (D) 8 kmph
500
Sol. First condition gives 4(b – s) = b + s (16) =  (18)
500 − 125
where, b = speed of the boat in still water (kmph) and
L and N, individually cover the distances of 500 m in tL
s = speed of stream (in kmph)
seconds and (tL + 80) seconds respectively.
Second condition gives 10/b = 2 ⇒ b = 5 (17) t + 80
From (16), (17) Hence, ratio of their speeds = L  (19)
tL
4(5 – s) = 5 + s ⇒ s = 3 From (18) and (19)
\ Speed of the stream = 3 kmph t L + 80 500
=
Hence, the correct option is (C). tL 375

18. A boat travels from point P to point Q upstream and Solving, we have tL = 240 sec.
returns from point Q to point P downstream PQ =
Time taken by M to run the race = 240 + 40 = 280 sec
96 km. If the round trip takes 9 hours and the speed of
Hence, the correct option is (C).

Unit I_Chapter 8.indd 90 27/04/2017 14:20:41


Chapter 8  •  Time and Distance  |  1.91

21. Anant started answering a mathematics test sometime \ At the start, angle made by minutes hand = 6p;
between 2:00 p.m. and 3:00 p.m. and ended at some- hours hand = 60 + p/2.
time between 5:00 p.m. and 6:00 p.m. If it is known
At the end of the test, the angles are:
that the position of the minute hand and the hour hand
at the start interchanged with the position at the end, for minute hand = 6q
how long did Anant take the test? hour hand = 150 + q/2
5 10 Since the positions interchange
(A) 2 hours (B) 2 hours
11 13 6p = 150 + q/2 → (1)
5 13 60 + p/2 = 6q → (2)
(C) 2 hours (D) 2 hours
7 14 ⎛ p−q⎞
Sol. 6 ( p − q ) = 90 − ⎜ ⎟
12 ⎝ 2 ⎠
11
1 13
10 2
( p − q ) = 90
2
S 180
9 C 3 ( p − q ) = → (1)
13
8 4
We know that the duration from 2 : p to 5 : q is
7 E 5 3 hours + (q - p) minutes = 180 + (q – p) minutes
6
Substituting from (1)
The angle of rotation of minute hand = 6 degrees/ = 180 – 180/13 = 180 × 12/13 minutes
minute.
= (180 × 12)/(13 × 60) hours
The angle of rotation of hour hand = (1/2) degree/ 10
minute. = 36/13 hours = 2 hours
13
If the duration of the test is ‘t’ minutes, then the sum of
Hence, the correct option is (B).
the angles of rotation of the two hands is 6t + t/2 = 13t/2
degrees(20) 22. Prakash and Pramod are running along a circular track
having started at the same time from the same point,
The test started between 2 and 3 o’ clock, when the
in the same direction. How much more distance would
hour hand was pointing at S (in the diagram) and the
Prakash have travelled compared to Pramod by the time
minute hand was pointing at E (in the diagram).
they meet for the 11th time given that the radius of the
The test ended between 5 and 6 o’ clock, and the posi- track is 7 m, Prakash and Pramod run at 22 m/s and
tion of the two hands got interchanged. 11 m/s, respectively?
This implies that the duration of the test is less than (A) 440 m (B) 384 m
3 hours; (because 3 hours would have been completed (C) 524 m (D) 484 m
if the minute hand came to the point E.) Sol. Since Pramod runs at half the speed of Prakash, they
During the test period, the minute hand completes meet once for every round Pramod makes or for every
2 rounds of the dial, and falls short of the angle SCE 2 round Prakash makes.
for the 3rd round; but the hour hand rotates through the \ If they meet 11 times, Pramod makes 11 round and
angle SCE. Prakash makes 22 round i.e., Prakash covers 11 times
Hence, sum of the angles of rotation of both the hands the circumference more than Pramod, i.e., 11(22/7) (7)
= angle of rotation for 3 rounds of the dial (2)m = 484 m
= 360 × 3 = 1080 degrees. (21) Hence, the correct option is (D).
From (20) and (21), 23. Peter and Paul are running in opposite directions along
a circular track of 600 m with initial speeds of 3 m/s
13t 2 × 1080 and 6 m/s. They start at the same point and at the same
= 1080; t = minutes
2 13 time. Whenever they meet, they exchange their speeds
and carry on in their respective directions. Find the dis-
2 × 1080 10 tance between Peter and Paul (when measured along
= hours = 2 hours.
13 × 60 13 1
the track) when Peter completes 4 rounds.
Alternate method: 4
(A) 150 m (B) 200 m
Let the time at start be p minutes past 2 and the time at
the end be q minutes past 5. (C) 300 m (D) 120 m

Unit I_Chapter 8.indd 91 27/04/2017 14:20:43


1.92  |  Unit 1  •  Quantitative Aptitude

Sol. Since Peter and Paul start at A and have the ratio of (A) 40.5 m (B) 81 m
their speeds as 1 : 2, first time when they meet say at S, (C) 84.5 m (D) 68.75 m
the distances covered by them respectively were in the Sol. Let the point at which the ends cross each other be x
ratio 1 : 2, i.e., Peter would have covered 200 m and mts from the point of entry of the slower train.
Paul would have covered 400 m
\ The time elapsed before the ends meet for the two
A
100 + x 200 − x + 150
Peter Paul trains will be equal to =
15 25
⇒ 500 + 5x = 1050 – 3x
8x = 550
S x = 68.75 mts
Alternate method:
At the point S, the ratio of speeds will now become 2 : 1
Distance between the rear ends of the two trains, at the
So, Peter will cover 400 m and reach A and Paul will instant of entry = 100 + 200 + 250 = 450 metres.
cover 200 m and reach A. So, the 2nd meeting point will
Time taken for the two ends to meet
be at A, the 3rd meeting point will be at S and the 4th
meeting point will be at A and so on. = (distance)/(relative speed)
So, by the time they meet for the 8th time i.e., at A, Peter = 450/(15 + 25) = 11.25 seconds.
would have covered 4 rounds. Distance covered by the slower train
Now, both are at A. = 11.25 × 15 = 168.75 metres.
When Peter covers 1/4th round, (i.e., 150 m), The required distance = 168.75 – (length of slower train)
Paul will cover 1/2 round, (i.e., 300 m) = 168.75 – 100 = 68.75 metres.
Distance between them is either or 150 + 300 Hence, the correct option is (D).
= 450 m or 600 – (150 + 300) = 150 m. Solutions for questions 26 and 27:
Hence, the correct option is (A).
26. A thief was running along to a median on a road at
Solutions for questions 24 and 25: 6 kmph. He crossed a police jeep moving at 9 kmph in the
opposite direction. The jeep had to continue for 10 more
Train T of length 100 m moving at 54 kmph and train U of
minutes before it could find a gap in the median and start
length 150 m moving at 90 kmph running on parallel tracks
chasing the thief. Find the total time taken by the jeep to
enter a 200 m long tunnel at the same instant from opposite
catch the thief from the time it crossed him (in minutes).
directions.
(A) 45 (B) 50 (C) 55 (D) 60
24. Which train exits the tunnel first and at the moment it
Sol. Distance ran by the thief in 10 minutes = 1 km
exits, what length of the other train is still in the tunnel?
(A) T, 50 m (B) U, 10 m Distance moved by the jeep in 10 minutes = 1 × 5 km
(C) U, 90 m (D) T, 75 m Distance between the jeep and the thief when the jeep
Sol. Time taken by Train U to exit the tunnel found a gap in the median = 2.5 km
Length of the tunnel + Length of the train U Additional time in which the jeep would overtake the
=
Speed of B 2⋅5 5
thief = = hours = 50 minutes
200 + 150 9−6 6
=
25 \ Total time = 60 minutes.

Train U exits the tunnel after 350/25 = 14 sec. Hence, the correct option is (D).
In 14 sec, front end of train T’s engine travels 27. Two runners run in the same direction along a circular
= 14 × 15 = 210 mts track 3 km in length. The faster runner overtakes the
That means the front end of engine is 10 mts out of the slower every 1 hour. Find the speed of the slower run-
tunnel. ner, if the faster one completes one length of the track
\ The length of the train T still in the tunnel 2 minutes sooner than the other.
= 100 – 10 = 90 m (A) 20 kmph (B) 18 kmph
(C) 15 kmph (D) 12 kmph
Hence, the correct option is (C).
25. How far is the point where the rear ends of the two Sol. Let the speeds of the faster and the slower runners be
trains cross each other, from the point of entry of the x kmph and y kmph respectively. Time to meet for the
slower train? first time is:

Unit I_Chapter 8.indd 92 27/04/2017 14:20:43


Chapter 8  •  Time and Distance  |  1.93

3 When Anil picks up Chetan he would be 40 t km behind


hrs, and this is given as 1 hour.
x− y Bala. Also the ratio of the speeds of Anil and Bala is
5 : 1. Anil (with Chetan) would catch up with Bala,
Hence [3/(x – y)] = 1 (22)
when he covers 5x and Bala covers x
Time taken, by the faster and the slower runners respec- \ 5x – x = 40 t.
3 3 The bike and Bala would have covered distances of
tively, for one lap are and hrs. As the difference
x y 50 t km and 10 t km respectively
of the above times is given as 2 minutes, 20 200t
3 3 1 XY = 10 t + t + 50t = km = 40 km
= −  (23) 3 3
x y 30 3
⇒t=
From (22), we get y = x – 3 which, when substituted in 5
(23) and solved for y, we get y = 15 kmph. 28. Find the time after which Anil turned back.
Alternate method: (A) 24 minutes (B) 48 minutes
The faster runner overtakes the slower runner once (C) 36 minutes (D) 30 minutes
in every hour. This implies that the number of rounds 3
(or laps) of the track made by the faster in 1 hour is Sol. Time after which Anil turned back = hours = 36
minutes 5
more than the rounds made by of the slower by 1. Let
the slower complete n rounds in 1 how; then the faster Hence, the correct option is (C).
completes (n + 1) rounds in 1 hour. 29. Find Bala’s average speed for the entire trip.
Time taken by the faster to complete 1 round is less (A) 12 km/hr (B) 24 km/hr
than that of the slower by 2 minutes, i.e., (1/30)th hour. (C) 20 km/hr (D) 25 km/hr
1 1 1 40
⇒ − = ; Sol. Bala’s average speed = = 25 km/hr
n n + 1 30 2
t + t +t
1 1 3
⇒ =
n( n + 1) 30; Hence, the correct option is(D).
⇒ n2 + n – 30 = 0; (n + 6) (n – 5) = 0 Note: The distance covered by Chetan until he was
Ignoring the negative value, n = 5. picked up by Anil is the same as that covered by Bala
from the point he was dropped.
Track length is 3 km; hence for 5 rounds, the distance
covered is 15 km; and the speed is 15 kmph. Solutions for question 30:
Hence, the correct option is (C). 30. Two men, M and N started walking towards each other
simultaneously from two places F and G, respectively,
Solutions for questions 28 and 29: which are 50 km apart. They meet after 5 hours. After
Three friends—Anil, Bala, and Chetan, from X wanted to their meeting, M reduced his speed by 1 kmph and N
travel from town X to town Y which was 40 km. Anil who increased his speed by 1 kmph. They arrived at G and
had a bike started along with Bala while Chetan started F, respectively, at the same time. Find the initial speed
simultaneously on foot. After some time, Anil dropped Bala of M.
on the way and went back to pick up Chetan while Bala pro- (A) 4.5 kmph (B) 5.5 kmph
ceeded to Y on foot. Anil picked up Chetan and reached Y at (C) 6 kmph (D) 4 kmph
the same time as Bala. Anil traveled at 50 km/hr. The speed Sol.
at which Bala and Chetan walked was 10 km /hr. →SM ← SN

Let us say Anil turned back after t hr. He would have cov- K
50 km
ered 50 t km then. Chetan would have covered 10 t km.
In the above diagram, we have SM and SN as the speed of
\ The two would be 40t km apart. They would meet in
M and N and K is the meeting pointing of M and N.
40t 2t
more hr .i.e., hr. As distance = (Relative speed) (time),
10 + 50 3
(SM + SN)5 = 50.
20t
In this time, Chetan would have covered km while SP + SQ = 10  (24)
3
100t After M and N meet, they move towards their destina-
Anil would have covered km. Also Bala would
3 tions which they reach at the same time; i.e., the times
2t
have covered km. of travel are equal.
3

Unit I_Chapter 8.indd 93 27/04/2017 14:20:45


1.94  |  Unit 1  •  Quantitative Aptitude

5S M 5SN Hence t = t12 or t = t1 = t2(26)


Hence =
SN + 1 S M −1 But it is given t = 5
SM2 – SM = SN2 + SN  (25) \ t = t1 = t2 = 5 hours.
Substituting (24) in (25) and solving we have M travelled for 5 hours at x kmph and for another
SM = 5.5 kmph 5 hours at (x - 1) kmph.
Alternate method: \ 5x + 5(x - 1) = 50 ⇒ x = 5.5 kmph
If ‘t’ is the time taken to meet, and t1, t2 are the dura- Hence, the correct option is (B).
tions taken after the meeting to reach the respective
destinations then, t = t1 t 2 . But it is given t1 = t2.

Unit I_Chapter 8.indd 94 27/04/2017 14:20:46


Chapter 9 Indices Surds Logarithms
Hints/Solutions
Practice Problems I 1 1

1

1
ka b = kc d
−2 −1 2
⎡ 35 ⎤ 5 ⎡122 ⎤ 2 ⎡ 8 ⎤ 3 Equating powers of k on both sides, we get
1. ⎢ 5 ⎥ × ⎢ 2 ⎥ × ⎢ ⎥
⎣4 ⎦ ⎣7 ⎦ ⎣ 343 ⎦ 1 1 1 1
− = −
a b c d
16 7 4
× × = 24 × 3-3 × 7-1 Hence, the correct option is (C).
9 12 49
8. 61/2, 71/3, 81/4, 91/5 take the LCM of denominators of the
Hence, the correct option is (C).
powers of the numbers. LCM = 60
x −4 × y 6 x 6 × y 3 x −12 × y 7 Raise the numbers with this LCM
2. × −6 ×
z −8 z z −8 (61/2)60, (71/3)60, (81/4)60, (91/5)60
x −4 + 6 −12 . y 6 + 3+ 7 630, 720, 815, 912
= = x-10. y16. z22
z −8−6 −8 Between 630 and 720
Hence, the correct option is (D). (63)10 and (72)10
⎡ ⎧ 1 ⎫⎤ ⎛ y ⎞ 21610 and 4910
1 − ⎢1 − ⎨1 − ⎬⎥ 1 − ⎜1 − (1 + y ) ⎟ 21610 > 4910 i.e. 630 > 720
3. ⎣ ⎩ 1 + y ⎭⎦ = ⎝ ⎠
(1 − y ) (1 − y ) Between 630 and 815
(62)15 and 815
⎛ 1 ⎞
⎜1 − ⎟ 3615 > 815 i.e. 630 > 815
⎝ 1+ y ⎠ y y
= = = Between 630 and 912
(1 − y ) (1 + y )(1 − y ) 1 − y 2
(65)6 and (92)6
Hence, the correct option is (A).
(7776)6 > 816
4. (a - b) (a2 + ab + b2) = a3 - b3
i.e. 630 > 912
Similarly the other powers of x become b3 - c3 and
\ 61/2 is largest in value
c3 - a3
a −b3 3 3 3 3 3 Hence, the correct option is (A).
x × x b −c × x c −a
3 3 3 3 3 3
= x a − b + b − c + c − a = x0 = 1 5 2
9. 2 −5 + 10 + 1000
2 5
Hence, the correct option is (B).
= 2 5 − 5 2 + 10 + 10 10
5. 3430.12 × 24010.08 × 490.01 × 70.1
= (73)0.12 × (74)0.08 × (72)0.01 × 70.1 = 11 10
= 70.36 × 70.32 × 70.02 × 70.1 Hence, the correct option is (D).
= 70.36+0.32+0.02+0.1 = 70.8 = 78/10 = 74/5 10. p − 4 pq = 4 p ( 4 p−4 q )
Hence, the correct option is (B).
4 pq − q = 4 q( 4 p−4 q)
6. 32 ( 2 x +1) = 33( 5 x − 3)
⇒ 2(2x + 1) = 3(5x – 3)
( )
−4
⎛4 p 4 p−4 q ⎞
⇒ 4x + 2 = 15x – 9 Required value = ⎜ ⎟
⇒ 11x = 1 ⇒ x=1
⎜4q
⎝ ( 4 p−4 q ⎟
⎠ )
Hence, the correct option is (A). −4 ⎛ 1
⎞ −4 ×
1 −4
−1
⎛ p⎞ ⎜ ⎛ p ⎞4 ⎟ ⎛ p⎞ 4 ⎛ p⎞ q
7. Let ⎜ 4 ⎟ = ⎜ ⎟ =⎜ ⎟ =⎜ ⎟ =
⎜ q⎟ ⎜⎜ ⎝ q ⎠ ⎟⎟ ⎝q⎠ q
⎝ ⎠ p
pa = qb = rc = sd = k ⎝ ⎠
⎝ ⎠
⇒ p = k 1/ a
Hence, the correct option is (D).
q = k 1/ b
( )
2
11. Given y =12 + 2 35 = 7+ 5
r = k 1/ c , s = k 1/ d
p r k 1/ a k 1/ c 1 1 7− 5
given = ; 1/ b = 1/ d . \ y = 7 + 5, = =
q s k k y 7+ 5 2

Unit I_Chapter 9.indd 95 27/04/2017 14:20:57


1.96  |  Unit 1  •  Quantitative Aptitude

⎛ 7− 5⎞ Similarly abc = 27 for a = b = c = 3 and a + b + c = 27


7 +3 5
7 + 5 – ⎜⎜ ⎟⎟ = for a = b = c = 9
⎝ 2 ⎠ 2
Hence, the correct option is (B).
13 5+ 7 7 7 7
\ y− =

y 2 18. 1612 = 24 ( ) 12
= 23
Hence, the correct option is (D). 3 3 3

12. a2 = 13 + 2 22
818 = 34 ( ) 8
= 32
2 2 8
b2 = 13 + 2 42
625 3 = 54 ( ) 3
= 53
c2 = 13 + 2 30
7 3
d2 = 13 + 2 40 Let us first compare 2 3 and 3 2
⇒ a2 < c2 < d2 < b2 When we raise both numbers to their sixth powers, we
\a<c<d<b get 214 and 39
Hence, the correct option is (C). 214 = 210 (24)= 1024(16) = 16384
13. Given a = 13 + 11, b = 15 + 9, 39 = 36, 33 = 729(27) which is more than 700(27) i.e.,
18900
c = 18 + 6 and d = 7 + 17 . 3 7
39 > 214 \ 3 2 > 2 3
In the given surds, the sum of the terms of each of the 3 8
surds is the same at 24. Then the surd containing the Let us now compare 3 2 and 5 3
terms as close as possible is the greatest, and the surd When we raise both to their sixth powers, we get 39 and
containing the terms as far as possible is the smallest. 516. 39 < 516, since 3 is a lower base and 9 is a lower
Hence a > b > d > c. index.
3 8
Hence, the correct option is (A)
\ 3 2 < 5 3 . Alternatively, we directly see that
14. Let 5x = a, 2y = b. 3a + 4b = 107; 5a + 8b = 189 3 8
3 8
Solving we get a = 5x = 25 = 52 3 < 5 and < . \ 32 < 53
2 3
\x = 2 7 3 8
b = 2y = 8 = 23. \ y = 3 2 3 < 3 2 < 5 3
Hence, the correct option is (D). Hence, the correct option is (A).
15. (35)3 (25)3/2 = (7 x 5)3 (5)3 19. It can be noticed that 11 + 4 6

( 7 ) (5) = (5 7 ) = (5 7 ) = (2 2 + 3 ) 2 = x 2
6 6 6 5x−4
\ 5x – 4 = 6

x = 10/5 = 2 11 + 4 6 x²
\ = ;
Hence, the correct option is (A). 2 2− 3 2 2− 3
16. Given 5x+3 - 5x - 3 = 78120 Multiplying and dividing 2 2 + 3 , it is equal to
⎡ 1⎤
⇒ 5 x ⎢53 − 3 ⎥ = 78120 2 2+ 3 x2
⎣ 5 ⎦ .= x3/5
⇒ 5x [56 - 1] = 78120 × (53) 2 2 + 3 2 2 − 3

⇒ 5x [15624] = 78120 × (53) Hence, the correct option is (D).
⇒ 5x = 54 \ x = 4 20. (a + b + c) + 2 ac + bc
Hence, the correct option is (A)
17. aa × bb × cc = ab × bc × ca = ( a + b) + c + 2 c ( b + a)
⇒ aa – b × bb – c × cc – a = 1
( )
2

Since a, b, c are positive integers > 1 = a+b + c = a+b + c


⇒ a – b = 0, b – c = 0 and c – a = 0 ⇒ a = b = c. Hence, the correct option is (B).
Choice (A) can be true for a = b = c = 2 21. logy[(x – 1) (x + 1)] = 2
Choice (B) can never be true for any of the possible ⇒ logy(x² – 1) = 2
values of a, b, c, since a + b + c = 3a and 3a ≠ 8 for any ⇒ x² – 1 = y² ⇒ x² – y² = 1
integral value of ‘a’. Hence, the correct option is (C)

Unit I_Chapter 9.indd 96 27/04/2017 14:21:01


Chapter 9  •  Indices Surds Logarithms  |  1.97

22. loga a + loga1/2 a + loga1/3 a + ….+ loga1/20 a By observation of the choices, only Choice (D), i.e.
x = 4 satisfies the above equation.
log a log a log a log a
= + 1
+ 1
+ ..... + 1 Alternate method:
log a
log a 2 log a 3 log a 20 x +5 = 5− x

= 1 + 2 + 3 + ….+ 20
Squaring both sides
20 × 21
= = 210
2 ⇒ x + 5 = 25 − 10 x + x

Hence, the correct option is (B). 20
23. log7(x – 7) + log7(x² + 7x + 49) = 4 x= = 2⇒ x = 4
10
⇒ log7[(x – 7) (x² + 7x + 49)] = 4 Hence, the correct option is (D).
⇒ log7 (x3 – 73) = 4 28. The given equation is :
⇒ (x3 – 73) = 2401 logba × logca + logcb logab + logac × logbc – 3 = 0
⇒ x3 = 2744 = (14)3 ⇒ x = 14
(log a) 2 (log b) 2 (log c) 2
Hence, the correct option is (D). ⇒ + + −3= 0
log b ⋅ log c log c ⋅ log a log a ⋅ log b
log a log b log c
24. Let = = =k [(as logba = (log a)/log (b)]
5 6 7
⇒ log a = 5k ⇒ a = 105k ⇒ (log a)3 + (log b)3 + (log c)3
log b = 6k ⇒ b = 106k – 3 log a × log b × log c = 0
log c = 7k when p3 + q3 + r3 – 3pqr = 0, either p = q = r
⇒ c = 107k or p + q + r = 0
b² = (106k)² = 1012k If p = q = r ; log a = log b = log c; ⇒ a = b = c
= 107k x 105k = ac which is contrary to the data. Hence p = q = r is not
Hence, the correct option is (A). acceptable.
7 7 \ p + q + r = 0; ⇒ log a + log b + log c = 0
128 27 ⎛ 2 ⎞ ⎛1⎞
2 5. 0.0000128 = 7 = 7 = ⎜ ⎟ = ⎜ ⎟ ⇒ log abc = 0; ⇒ abc = 1
10 10 ⎝ 10 ⎠ ⎝ 5 ⎠
⎛1⎞
7 Hence, the correct option is (B).
⎜ ⎟
\ log⎛ 1 ⎞ 0.0000128 = log⎛ 1 ⎞ ⎝5⎠
=7 29. Given a = log6 161 = log6 (23 x 7) = log6 23 + log6 7
⎜ ⎟ ⎜ ⎟
⎝5⎠ ⎝5⎠
a = b + log6 7
Hence, the correct option is (D).
a – b = log6 7
26. tan45° = 1
log7 6 = 1/(a – b)
log (tan45°) = 0
\ Product = 0 Hence, the correct option is (C).
\x=0 30. Let x = y2 = z3 = w4 = u5 = k
Hence, the correct option is (C). x = k, y = k1/2, z = k1/3, w = k1/4 and u = k1/5
27. Since log x log5 ( x+5 + x = 0 ) logxxyzwu = logk k . k1/2 . k1/3 . k1/4 . k1/5

⇒ log5 ( x +5 + x =1)
1 1 1 1 137
=1 + + + + =
2 3 4 5 60
=2
17
60
⇒ x+5 + x = 5 Hence, the correct option is (C).

Practice Problems 2
Solutions for questions 1 to 30: 52 a −5 × 5a × 53a + 9
= = 5° = 1
52 a − 5 × 25a 2 ×125a + 3 53a × 54 a + 4 × 5− a
1. Simplify the following: Hence, the correct option is (C).
( 3125) × 625a + 1 × 5− a
3a 5

(A) 5 (B) 25 (C) 1 (D) 125 2.


(x ) ⋅ (x ) (x )
a 2c b 2a c 2b

a
52 a −5 × (52 ) 2 × (53 ) a + 3 (x ) ⋅ (x ) ⋅ (x )
a+b c b+c a c+a b

Sol. 3a (A) 0 (B) x2a+2b+2c


(55 ) 5 × (54 ) a +1 × 5− a (C) 1 (D) x4ab+4ac+4bc

Unit I_Chapter 9.indd 97 27/04/2017 14:21:03


1.98  |  Unit 1  •  Quantitative Aptitude

x 2 ac . x 2 ab . x 2 bc 6. Arrange the following in ascending order.


Sol. ac + bc
x . x ba + ca . x bc + ab p = 26 − 23, q = 18 − 15 ,

x 2 ac . x 2 ab . x 2 bc r = 11 − 8, s = 24 − 21
= (A) rqsp (B) psrq
x ac + bc + ba + ca + bc + ab
(C) pqrs (D) psqr
x 2 ac + 2 ab + 2 bc
= =1
x 2 ab + 2 ac + 2 bc Sol. Given p = 26 − 23 , q = 18 − 15,
Hence, the correct option is (C). r = 11 − 8 and s = 24 − 21 .
3 1 In the given surds, the difference between the terms
3. Simplify: +
5+ 2 6 + 5 of each of the surds is the same at 3. Then the surd
containing the greater terms is the smallest and smaller
(A) 6 − 2 (B)
6+ 2 terms is the greatest.
(C) 5 + 6 (D)
2− 6 Hence p < s < q < r.
Hence, the correct option is (D).

Sol.
3
=
3 ( ) 5− 2
= 5− 2
7. If (3x+1) + (4y–1) = 73 and 4(3x) + 3(4y–2) = 60, find x + y.
(A) 5 (B) 4
( 5) − ( 2)
2 2
5+ 2
(C) –1 (D) –2
1 6 − 5 ol. Let 3x = a and 4y - 2 = b
S
= = 6 − 5  3a + 4b = 73
( 6) − ( 5) Given (1)
2 2
6 + 5
 4a + 3b = 60 (2)
3 1 4 (1) ⇒ 12a + 16b = 292
+ = 6 − 2 3 (2) ⇒ 12a + 9b = 180
5+ 2 6 + 5
- -
Hence, the correct option is (A). ---------------------
9 Subtracting 7b = 112
4. 2 1 2
=
6 − 18 + 3
3 3 3 ⇒ b = 16
From (2), 4a = 60 - 3b = 60 - 48 ⇒ a = 3
( )
(A) 6 3 + 33
1 1

( ) (B) (1/3) 6 3 + 33
1 1

\ 3x = 3 and 4y - 2 = 16

(6 − 3 )
(C)
1
3
1
3
(D) (1/9) (6 + 3 )
1
3
1
3 ⇒ x = 1 and y - 2 = 2
\x+y=5
(6 ) + (3 ) 1 3 1 3
3 3
9 Hence, the correct option is (A).
Sol. =
(6 ) −18 + (3 )
2
2 1 2 1 2 1 1 2
⎛ a a ⎞
6 − 18 + 3 +
3 3 3
3 3 3
8. Simplify : ⎜ ⎟
⎝ b− c b + c⎠
1 1
= 6 3 + 33
(A)
2a 2 c (b − c )
(B)
2

Hence, the correct option is (A).


(b − c ) 2
4a 2 b
5. ( 324 + 2 323 − ) ( 324 − 2 323 = ) (C)
4a2 b
(D)
2a 2
(A) 2 (B) 1 ( b − c )2 b2 + c2 − a2
(C) 36 (D) 2 323
2
⎛ a a ⎞
+
( ) Sol. ⎜ ⎟
2
Sol. 324 + 2 323 = 323 + 1 = 323 + 1 ⎝ b− c b+ c⎠
2
⎛ b + c + b − c⎞
( )
2
324 − 2 323 = 323 −1 = 323 −1 =a ⎜2
⎟⎟
⎜ b −c
⎝ ⎠
Required value = 323 + 1 − ( 323 −1 ) a2 (2 b )2 4a2 b
= =
(b − c) (b − c)
2 2
= 323 + 1 − 323 + 1 = 2
Hence, the correct option is (A) Hence, the correct option is (C).

Unit I_Chapter 9.indd 98 27/04/2017 14:21:08


Chapter 9  •  Indices Surds Logarithms  |  1.99

1 7− 3 7− 3
9. If x =
3
55 + 12 21 then the value of x + is _____. (A) (B)
x 2 3
(A) 4 (B) 5 (C) 6 (D) 7
7− 3 7− 3
Sol. x = 3
55 + 12 21 (C) (D)
2 3
x3 = 55 + 12 21
= 3025 + 3024 Sol. Given x = 5 − 21 ⇒ x =
1
2
(
10 − 2 21 )
1
3 =
( 3025 ) −(
2
3024 )
2

= 3025 − 3024
x=
1
2
( 7− 3 )

x 3025 + 3024
32 – 2x = 22 + 2 21 \ 32 − 2 x = 1 + 21
1
x + 3 = 2(55) = 110
3

x
x
1
2
( 7− 3 )
Consider =
1 ⎞⎛⎛ ⎞ 2
⎛ 1⎞ 32 − 2 x − 21 21 + 1 − 21
110 = ⎜ x + ⎟ ⎜ ⎜ x + ⎟ − 3 ⎟ (3)
⎝ x ⎠ ⎜⎝ ⎝ x⎠ ⎟

Method 1 =
1
2
(7− 3 )
Only choice B satisfies this equation. Hence, the correct option is (C).
Method 2 1 2. Arrange a, b, c, and d in descending order if a = 13 + 9,
1
x is positive . x + is positive. The R.H.S. of (3) b = 19 + 3 , c = 17 + 5 , d = 12 + 10 .
x 1
increases with an increase in x + . \ Exactly one (A) b, c, a, d (B) d, b, a, c
x (C) d, b, c, a (D) d, a, c, b
1 1
solution exists for x + . When x + = 5, (3) is Sol. a2 = 22 + 2 117 ;  b2 = 22 + 2 57 ;
satisfied. x x
c2 = 22 + 2 85 ;  d2 = 22 + 2 120
\ This is the only solution.
\We get d2 > a2 > c2 > b2
Hence, the correct option is (B).
\Descending order is d, a, c, b.
10. If a = 4(b – 1) and b ≠ 2, then simplify
Hence, the correct option is (D).
b− a b+ a 4
+ − . 6
b+ a b− a (b − a ) (b + a ) 13. If x =
4+
6
, find the value of x.

6
(A) b (B) 2 2+
6
b 4+
(C) (D) (b – 2)1/2 2 + ¥
b−2
9 + 33
b− a +b+ a 4 (A) (B) 1
Sol. Given expression = − 2
b −a 2
b −a 2
9 − 43 9 − 33
(C) (D)
2 (b − 2) 2 (b − 2) 2 4
= = ;
b2 − a b 2 − 4 ( b − 1) 6
Sol. Given x =
since a = 4(b – 1) 6
4+
2 (b − 2) 2 (b − 2) 2 (b − 2) 6
= = = 2+
b − 4b + 4 2
(b − 2)
2
(b − 2) 4+
6
2 + .....∞
= 2,
as it is given that b ≠ 2. 6
⇒  x =
\From the choices answer is (B) 6
4+
Hence, the correct option is (B). 2+ x

x 6 (2 + x) 6 (2 + x)
11. If x = 5 − 21, find the value of . ⇒ x= =
32 − 2 x − 21 4 (2 + x) + 6 4 x + 14

Unit I_Chapter 9.indd 99 27/04/2017 14:21:13


1.100  |  Unit 1  •  Quantitative Aptitude

Squaring both sides = 7(2 + 3 + 2 – 3 ) (2 + 3 – 2 + 3 )


6 x + 12 + 11(2 – 3 ) (2 + 3 ) = 7(4) (2 3 ) + 11(4 – 3)
x2 =
4 x + 14
= 56 3 + 11
⇒ 4x3 + 14x2 - 6x - 12 = 0
Hence, the correct option is (B).
By observation x = 1 satisfies the above equation.
17. If x = 3 2 + 3 4 , what is the value of x3 – 6x?
⇒ (x - 1) (4x2 + 18x + 12) = 0 (A) 0 (B) 4 (C) –6 (D) 6
⇒ x = 1 (or) 4x2 + 18x + 12 = 0
Sol. x = 3 2 + 3 4 ; x − 3 2 = 3 4
If 4x2 + 18x + 12 = 0, then the value of x must be nega-
tive. But from the given question it is clear that x cannot

x 3 − 2 − 3x 3 2 x − 3 2 = 4 ( )
be negative.
Hence x = 1 is the only solution. x 3 − 2 − 3x 3 2( 4) = 4 3


  x − 2 − 3 x 8 = 4
Hence, the correct option is (B). 3 3

..∞
..
 x3 – 2 – 6x = 4
1
x
x
14. If x = , find x.  x3 – 6x = 6
4
Alternate method:
1 1 1 1
(A) 12 (B) 10
(C) 16
(D) x = 3
2 + 3 4 ; on cubing both sides,
2 2 2 214
( 2) +( 4) ( 2 )( 4 ) ( )
3 3
..
.∞ x3 = 3 3
+ ( 3) 3 3 3
2+34
x
1
x
Sol. x = ⇒ x3 = 2 + 4 + (3) 3 8 (x)
4
1
1 ⇒ x3 = 6 + 6x; ⇒ x3 – 6x = 6
⇒ x4 =
4 Hence, the correct option is (D).
4
⎛1⎞ 18. If ( 6 + 5 )x² – 2 + ( 6 – 5 )x² – 2 = 22, then x =
⇒ x =⎜ ⎟
⎝4⎠ (A) ± 2 , 0 (B) ±2, ±1
8 (C) 0, ±2 (D) ±2, 1
⎛1⎞ 1
⇒ x = ⎜ ⎟ = 16
( )
x2 − 2
⎝4⎠ 2 Sol. Let 6+ 5 =a
Hence, the correct option is (C).
( ) 1 1
x2 − 2
15. Find the value of 7 − 3 5 . 6 − 5 = =
( )
x2 − 2
a
3− 5 6+ 5
(A) 7 − 2 3 (B)
2 1
\ the equation becomes a + = 22
3− 7 5+ 2 a
(C) (D) ⇒ a2 - 22a + 1 = 0
2 2 3
14 − 2 45 22 ± 484 − 4
Sol. 7 − 3 5 = ⇒a=
2 2
22 ± 4 30
9+5−2 9 5 9 − 5 3− 5 = = 11 ± 2 30
= = = 2
2 2 2
Case (i) a = 11 + 2 30
Hence, the correct option is (B).
1 1
( ) ( )
x2 − 2 2
16. If a = ,b= , what is the value of 7b2 + ⇒ 6+ 5 = 11 + 2 30 = 6+ 5
2+ 3 2− 3
11ab – 7a2? ⇒ x2 - 2 = 2 ⇒ x2 = 4 ⇒ x = ± 2
(A) –14 + 21 3 (B) 11 + 56 3 Case (ii)

(C) 49 + 8 3 (D)
3 + 11 a = 11 - 2 30

( ) ( ) ( )
x2 − 2 2 −2
1 1 ⇒ 6+ 5 = 6− 5 = 6+ 5
Sol. a = =2− 3 ;b = = 2+ 3
2+ 3 2− 3 ⇒ x2 - 2 = -2 ⇒ x = 0
7b² + 11ab – 7a² = 7(b + a) (b – a) + 11 (a) (b) Hence, the correct option is (C).

Unit I_Chapter 9.indd 100 27/04/2017 14:21:20


Chapter 9  •  Indices Surds Logarithms  |  1.101

1 9. If x = 3 + 5, then find the value of x3 – 9x2 + 22x. Sol. Taking logarithms to the base 5, the given equation
(A) –15 (B) 12 (C) 42 (D) 45 becomes, log51000 = log101000, log510 = 3log510 = y
Sol. x = 3 + 5 log0.5 1000 = 3log0.5 10 = x
⇒ (x - 3) = 5 1 1 − log10 0.5 log10 5 log10 10
− + = + = = 1/3
⇒ (x - 3)3 = 5 5 x y 3 3 3
⇒ x3 – 9x2 + 27x – 27 = 5 5 = 5 (x – 3) Alternate method:
⇒ x3 – 9x2 + 22x = 12 Given that 5y = 1000,
Hence, the correct option is (B). ⇒ 5 = 10(3/y)(4)
a×10 −3 (0.5)x = 1000, ⇒ 10(3/x) = 0.5,
20. If a × 52 = 2020.20, what is the value of ?
10 4 5
(A) 0.0000808080 ⇒
= 10( 3/ x )
10
(B) 0.00000808080
⇒ 5 = 10(3/x + 1) (5)
(C) 0.000000808080
(D) 0.0000202020 (4) and (5) are equal; 10(3/y) = 10(3/x + 1);
Sol. a × 52 = 2020.20 3 3 ⎛1 1⎞ 1
⇒ = + 1, ⇒ ⎜ − ⎟ =
a × (10/2)2 = 2020.20 y x ⎝ y x⎠ 3
a × (10)2 = 8080.80 Hence, the correct option is (A)
a = 8080.80/100 = 80.8080 23. If logr 6 = a, logr 3 = b, what is the value of logr (r/2) in
terms of a and b?
a × 10 −3 a a (A) 1 – a + b (B) ab – a – b
= 4 = 7
10 4
10 × 10 3
10 (C) a + b (D) 1
80.8080 Sol. logr (r/2) = 1 – logr 2
 = = 0.00000808080
10 7 1 – logr 6 + logr 3 = 1 – a + b
Hence, the correct option is (B). Hence, the correct option is (A).
21. Given that x, y, z are consecutive positive integers, 24. If log(x+y) (x – y) = 3, then what is the value of

)( )
which of the following is the value of log(xz + 1)?
log x 2 − y 2 x 2 + 2 xy + y 2 ?
(A) log(x + y + z) (B) 0 (
(C) 1 – log y (D) 2 log y (A) 0 (B) 2
Sol. x = y – 1 3
(C) (D) 1/2
y = y 2
z=y+1
(
Sol. log x 2 − y 2 x 2 + 2 xy + y 2 )
log (xz + 1) = log [y2 – 1 + 1] = 2log y
Alternate method: 2 log ( x + y )
[( logba = (loga)/(logb)]
Such questions can be solved by numerical method. log ( x + y ) + log ( x − y )
Assume the smallest numerical values satisfy the given
Considering the logarithm to the base (x + y) the given
conditions and substitute in the function.
2 2 2 1
1, 2 and 3 are three consecutive positive integers. function is = = =
1 + log( x + y ) x − y 1 + 3 4 2
log(xz + 1) = log [(1 x 3) + 1] = log4
Hence, the correct option is (D).
For the values x = 1, y = 2, z = 3, first option log (x + y
25. Which of the following can never be a value of
+ z) = log 6; 2x 3y 1
Second option is zero; third option is (1 – log2); and log 2 x + log3 y given that 2x ≥ 3y and y > ?
3y 2x 3
fifth option is 2log2, which is log2² = log 4. This is (A) –2.5 (B) –2 (C) 1 (D) –1.5
equal to log (xz + 1).
2x 3y
Hence, the correct option is (D). Sol. log 2 x + log3 y
3y 2x
⎛ 1 1⎞ = 1 - log2x3y + 1 - log3y2x
22. 5y = (0.5)x = 1000, what is the value of ⎜ − ⎟ ?
⎝ y x⎠ = 2 - (log2x3y + log3y2x)
(A) 1/3 (B) 0.95 (C) 10 (D) 1 let log3y 2x = p

Unit I_Chapter 9.indd 101 27/04/2017 14:21:23


1.102  |  Unit 1  •  Quantitative Aptitude

1  For a strong base (> 1) the log increases with number


It is given that y >
3 ⇒ (4x - 15)2 > 5x
⇒ the base 3y > 1
⇒ 16x2 - 120x + 225 > 5x
Also 2x ≥ 3y
⇒ 16x2 - 125x + 225 > 0
⇒ log3y2x ≥ 1
⇒ 16x2 - 80x - 45x + 225 > 0
⇒p≥1
⇒ 16x (x - 5) - 45 (x - 5) > 0
Consider 2 - (log2x 3y + log3y2x)
⇒ (16x - 45) (x - 5) > 0
⎛1 ⎞
= 2 - ⎜ + p⎟ ⎛ 45 ⎞
⎝ p ⎠ ⇒ x ∈ ⎜ −∞, ⎟ ∪ ( 5, ∞ )
⎛ 1⎞ ⎝ 16 ⎠
since p ≥ 1, the value of ⎜ p + ⎟ ≥ 2
⎝ p⎠ Since 4x - 15 > 0
⇒ x > 3 \ x ∈ (5, ∞)
⎛ 1⎞
Hence the maximum value of 2 - ⎜ p + ⎟ = 0 Hence x can take values from 6 to 99 i.e. a total of 94
⎝ p⎠
values.
From the given options 1 cannot be the value of the
given expression. Hence, the correct option is (D).
Hence, the correct option is (C). 29. Find the total number of digits in 176450, if log 2 =
0.301, log 3 = 0.4771 and log 7 = 0.845.
26. If a = b = 3 c = 4 d = 5 e , then find the value of loga (A) 161
(abcde). (B) 163
(A) 15 (B) 13 (C) 14 (D) 10 (C) 162
Sol. Given a = b = 3 c = 4 d = 5 e (C) 164
⇒ b = a2, c = a3 d = a4 and e = a5 Sol. N = 176450
\ loga (abcde) N = (42)100
= loga (a) (a2) (a3) (a4 ) (a5) N = [(2 × 3 × 7)]100
= loga a15 = 15 log N = 100 [log 2 + log 3+ log 7]
Hence, the correct option is (A). log N = 100 [1 × 6231]
27. If log2 = 0.3010, then find the value of log6250. log N = 162 × 31
(A) 3.294 (B) 3.486 Hence the number of digits in N is 163.
(C) 3.664 (D) 3.796 Hence, the correct option is (B)
Sol. log 6250 1 1 1
30. If = log bc a, = log ac b and = log ab c,
= log 54 (10) x −1 y −1 z −1
= 4 log 5 + log 10 then which of the following is true?
10
= 4 log + log 10 1 1 1 1 1 1
2 (A) + = (B) + + =1
x y z x y z
= 4 (log 10 - log 2) + log 10
x + y + z = 1
(C) (D) xy + yz + zx = 1
= 4 (1 - 0 × 301) + 1
= 4 (0 × 699) + 1 1
Sol. Given = log bc a
= 2 × 796 + 1 = 3.796 x −1
⇒ x - 1 = loga bc
Hence, the correct option is (D).
⇒ x = 1 + loga bc = logaa + logabc
28. For how many positive integer values of x less than 100,
1 ⇒ x = loga abc
is log5x (4x – 15) > ?
2 Similarly y = logb abc and
(A) 96 (B) 90 (C) 95 (D) 94 z = logc abc
Sol. As x = 1, 2, . . . . 99, 5x ≥ 5.
1 1 1
1 \ + + = log abc abc = 1
Given log5 x ( 4 x − 15) > x y z
2

( )
⇒ ( 4 x − 15 ) > 5 x 2
1 Hence, the correct option is (B).

Unit I_Chapter 9.indd 102 27/04/2017 14:21:25


Chapter 10  Quadratic Equations
Hints/Solutions
Practice Problems 1 Option (B):
2m  + 3m − k
1. 2x2 - 7x + 2 = 0 Sum = 1 + =
 + m − k  + m −k
Discriminant = (-7)2 - 4 × 2 × 2 = 33 > 0 but 33 is not
Sum not satisfied
perfect square.
Option (C):
\ the roots are irrational.
k+m− 2m
Hence, the correct option is (D). Sum = 1 + =
+m−k +m−k
2. Discriminant = 62 – 4(2) (–5) = 76.
Sum satisfied
This is positive but not a perfect square.
k +m− k+m−
\ The roots are conjugate surds. Product = 1 × =
+m−k +m−k
Hence, the correct option is (C).
Alternate Solution:
3. The equation whose roots are p times the roots of ax2 +
2
Sum of the coefficients = m – k +  – 2m + m –  + k = 0
⎛x⎞ ⎛x⎞ \ 1 is the one roof of the equation we know that 1 is
bx + c = 0 is given by a ⎜ ⎟ + b ⎜ ⎟ + c = 0.
Here p = 1/3. ⎝ p⎠ ⎝ p⎠ one root of ax2 + +bx + c = 0, then the other root is c/a
m−+k
\ The required equation is (3x)2 + 6(3x) + 10 = 0 \ Hence second root is .
m+−k
i.e. 9x2 + 18x + 10 = 0
Hence, the correct option is (C).
Hence, the correct option is (C).
4 ac − b 2
4. Let the equation be ax2 + bx + c = 0. 7. The expression represents the maximum
4a
As the roots are reciprocals of each other, the product value of the quadratic expression ax2 + bx + c when
of the roots is 1. a < 0 and represents the minimum value of the quad-
c ratic expression ax2 + bx + c when a > 0.
\ = 1 ⇒ c = a  b = 2a
a \ Neither (A) nor (B) is true.
\ ax2 + 2ax + a = 0 Hence, the correct option is (D).
a(x + 1)2 = 0 8. The given equation has the sum of its roots as –1 and
As a ≠ 0, x = –1, –1 the product of its roots as –420. As the sum of the roots
\ The sum of the squares of its roots is (–1)­2 + (–1)2 = 2 as well as the product of the roots are negative, the
roots are of opposite signs with the numerically larger
Hence, the correct option is (D).
root being negative.
5. Let one root be a, other root is 3a.
Hence, the correct option is (D).
Sum = 4a = 2
9. (k2 – 3k + 2) (k2 – 7k + 12) = 120
1 3 (k – 1) (k – 2)(k – 3) (k – 4) = 120  (5)
⇒a= , 3a =
2 2
= (5) (4) (3) (2)
k 1 3
Product = = × ⇒ k = 3 Comparing the two sides, k – 1 = 5
4 2 2
Hence, the correct option is (A). k=6
6. From the quadratic equation Alternate method:
2m Substituting the choices in place of k in the equation (5)
Sum = above, we see that only choice (B) satisfies it
m−k +l
Check out with each of the options as for which of them Hence, the correct option is (B).
has the same sum and product. 10. Since A copied the coefficient of x wrongly he copied
Option (A): the constant term correctly, hence constant term = 12 ×
 + m− k 2m 6 = 72. B copied the constant term wrongly and hence
Sum = 1 + = he copied the coefficient of x correctly, hence coeffi-
k + m − k + m − 
cient of x = -(1 + 26) = -27. Hence, the equation is x2
Sum not satisfied

Unit I_Chapter 10.indd 103 27/04/2017 14:16:49


1.104  |  Unit 1  •  Quantitative Aptitude

- 27x + 72 = 0 14. x + y = 4
Thus ⇒ (x - 24) (x - 3) = 0 x2 + y2 = x2 + (4 - x)2 = 2x2 - 8x + 16
Thus x = 24 or x = 3 Coefficient of x2 = 2 which is positive
Hence, the correct option is (C). \Minimum value exists.
11. (x - k1) (x - k2) = - 1; 4 ac − b 2
Minimum value =
Given that roots are integers. 4a
As, a = 2, b = –8 and c = 16, the minimum value is
⇒ x has integer values.
128 − 64
given that k1, k2 are integers. =8
8
Hence, (x - k1) as well as (x - k2) are integers. Hence, the correct option is (A).
The only way -1 can be resolved into the product of 15. For the equation 2x2 + 8x + p = 0 to have rational roots,
integers is -1 = 1x -1. Hence, b2 - 4ac = 82 - 4(2)(p) = 64 - 8p, should be a perfect
if, (x - k1) = + 1 square.
Case I
then (x - k2) = -1 64 - 8p is a perfect square.
or ⇒ 8 (8 - p) is a perfect square.
if, (x - k1) = - 1 ⇒ 8 (8 - p) = 0, 1, 4, 9, 16, 25, 36, 49, 64.
Case II
then (x - k2) = +1 As p is an integer, the value selected should be an inte-
Case I: gives ger such that 8(8 – p) is divisible by 8. i.e. 8 (8 - p) = 0,
16, 64, …
k1 + 1 = k2 - 1  or  k2 - k1 = 2
When 8(8 - p) = 0; p = 8
Case II: gives
8(8 - p) = 16; 8 - p = 2, p = 6
k1 - 1 = k2 + 1 or, k1 - k2 = 2
8(8 - p) = 64, 8 - p = 8, p = 0; not admissible.
Hence, the correct option is (D).
8(8 - p) = 144, 8 - p = 18, p = -10
12. As the equations given in the options contain coeffi-
cients a, b, and c, consider the equivalent of px² + qx + Hence, p = 8 or 6 are the possible values.
r = 0 with co-efficients a, b and c. Hence, the correct option is (D).
px2 + qx + r = 0 is equivalent to 16. Let the common root be a.
a(x - k)2 + b(x - k) + c = 0 (6) x2 – 9x + 18 = 0
because the equation whose roots are ‘k’ more than x = 6, 3
those of equation (6) is obtained by replacing; ‘x’ by (6 + a) (3 + a) = 40
‘x - k’.
(a – 2) (a + 11) = 0 as a > 0, a = 2
a(x - 2k)2 + b(x - 2k) + c = 0
Hence, the correct option is (B).
Hence, the correct option is (B).
17. The given equation is x3 – 11x2 + 37x – 35 = 0  (7)
13. x2 - 10x + 16 = 0
One root is 3 − 2 . As the coefficients are rational,
(x - 8) (x - 2) = 0
The irrational roots occur in pairs, 3 + 2 is also a root
x = 8 or x = 2.
of (1)
If x = 8 is the root of x2 - 10x + 16 which is half of one
root (a) of x2 - 4Rx + 8 = 0 then a = 2 × 8 = 16. \ Sum of the roots = –(–11) = 11 from (7)
Then the other root b of x2 - 4Rx + 8 = 0 would be Sum of two of the roots is 3 + 2 + 3 − 2 = 6
8 1 \ The third root is 11 – 6 = 5
= . Since this is not an integral root this value is
16 2 \ The roots of the equation are 3 ± 2 and 5.
not acceptable. If x = 2 which is the root of x2 - 10x +
16 = 0, which is half of one root (a) of x2 - 4Rx + 8 = 0 Hence, the correct option is (C).
8 18. |x|2 + 6|x| – 55 = 0
then, the second root b is = 2 .
4 (|x| + 11) (|x| – 5) = 0

The sum of the roots = −


( −4 R) = 4 R = 4 + 2 = 6, |x| cannot be negative. \ |x| = 5. \ x = ± 5
1 \ (a, b) = (5, –5) or (–5, 5)
6 3
R= = Let the roots of the second equation be y1 and y2.
4 2
Hence, the correct option is (C). Let y1 = aby2

Unit I_Chapter 10.indd 104 27/04/2017 14:16:50


Chapter 10  •  Quadratic Equations  |  1.105

y1 + y2 = (ab + 1) y2 and y1y2 = ab y22 9a2 - 3a - 15a + 5 = 0;


3a(3a - 1) - 5(3a - 1) = 0
( y1 + y2 ) 2 (αβ + 1)
2

= a = 1/3 or 5/3
y1 y2 αβ
Hence 3x = 1/3 = 3-1 or 5/3, thus x = -1 or log3 (5/3)
2
⎛ −q ⎞ Hence, the correct option is (A).
⎜⎝ p ⎟⎠ ( −25 + 1)
2

⇒ = 21. x 2 − 2 x − 3 + x 2 + 5 x − 24 = x 2 + 7 x − 30
r −25
p

( x − 3) ( x + 1) + ( x + 8) ( x − 3) = ( x + 10) ( x − 3)
q 2
−576 We can see that (x - 3) is common to all the terms of the
=
rp 25 above equation.

Hence, the correct option is (A). If x - 3 = 0, we will have
19. 0 ( x + 1) + ( x + 8)(0 ) = 0 + 0 = ( x + 10 )(0 ) = 0
A
Thus the equation is satisfied.
Hence x - 3 = 0, i.e. x = 3
Other values in the choices do not satisfy the equation.
c b
Hence, the correct option is (B).
22. N P A
B C
a Ranjan x pa x . pa
Let the sides be as represented in the figure. Raman y pb y . pb
From the data, b + a = 2c (8) When N, P and A stand for the number, the price and
b the amount respectively.
and c = 9 + ; ⇒ 2c = 18 + b (9)
2 Given x + y = 108
From (8) and (9); a = 18 (10) y Pa = 722 (12)
Applying the theorem of Pythogoras, x Pb = 578 (13)
b² = c² + a² and substituting a = 2c - b, y pa 722 361
⇒ = = and xpa = ypb
b² = c² + (2c - b)²; ⇒ b² = c² + 4c² - 4bc + b², x pb 578 289
⇒ 5c² - 4bc = 0; ⇒ 5c = 4b(11) 2 2
pa y ⎛ y ⎞ ⎛ 19 ⎞ y 19
Substituting in (8) \ = or ⎜ ⎟ = ⎜ ⎟ ⇒ =
2
pb x ⎝ x ⎠ ⎝ 17 ⎠ x 17
⎛ 4b ⎞
b + a = 2c ⇒ b + 18 = ⎜ ⎟ 17
⎝ 5⎠ The number of floppies with Ranjan = ×108 = 51
⇒ 5b + 90 = 8b  ⇒ b = 30 36
Hence, the correct option is (A).
Alternate method:
23. Let the roots of E be a and b
From the first two equations, a = 18 is obtained.
a = a + b
(3 : 4 : 5) is a ratio which satisfies the conditions b + a
= 2c, because (5 + 3) = 2(4). b = a b
Hence, 3 parts of the ratio of (3 : 4 : 5) is 18. Required equation has its roots as
Hence, 5 parts of the ratio is 5(18/3) = 30; ab  2 and ba2
i.e., b = 30. ab  2 + ba2 = ab (a + b)
Hence, the correct option is (A). (ab 2) (ba 2) = (ab )3
20. 2(32 · 32x) - 4(32 · 3x) + 10 = 0 required equation is x2 – abx + b3 = 0
2(9)(32x) - 4(9)(3x) + 10 = 0 Hence, the correct option is (A).
18(32x) - 36(3x) + 10 = 0; 24. x2 + ax + b = 0 (14)
Dividing by 2 and substituting a for 3x, x2 + bx + a = 0 (15)
we have 9a2 - 18a + 5 = 0 Let ‘k’ be a common root, then
k2 + ak + b = 0

Unit I_Chapter 10.indd 105 27/04/2017 14:16:52


1.106  |  Unit 1  •  Quantitative Aptitude

k2 + bk + a = 0 ⇒ a + b + 1 = 0.
⇒ k (a - b) + (b - a) = 0  (on subtraction), Hence, the correct option is (C).
1
(k - 1) (a - b) = 0 25. If the roots of the given equation are a and ; (a)
α
a - b ≠ 0, \ if a = b the two equations become identical ⎛ 1⎞ 2 ( 2m − 1)
and they will have two common roots. ⎜⎝ ⎟⎠ = 1, = 2m - 1 = 1
α 2
\ k = +1 ⇒ 1 + 1 = 2m, ⇒ m = 1
⇒ 1² + a + b = 0 Hence, the correct option is (C).

Practice Problems 2 Sol. Let the three consecutive positive integers be (x - 1), x,
(x + 1) then, (x - 1)2 + x2 + (x + 1)2 = 869
1. If the sum of the roots and the product of the roots of a
quadratic equation are 13 and 30, respectively, find its ⇒ 3x2 + 2 = 869
roots. ⇒ x = ±17
(A) 10, 3 (B) –10, –3 As x cannot be negative, x = 17
(C) 10, –3 (D) –10, 3 The numbers are 16, 17, 18
Sol. Let the equation be Hence, the correct option is (C).
x2 – 13x + 30 = 0 143
5. An integer exceeds its reciprocal by . Find the
x2 – 10x – 3x + 30 = 0 12
integer.
x(x – 10) – 3(x – 10) = 0 (A) 6 (B) –12 (C) 12 (D) –6
(x – 10) (x – 3) = 0
ol. Let the integers be x,
S
x = 10 or 3 1 143
Hence, the correct option is (A). then x − =
x 12
2. Find the sum and the product of the roots of the equa- ⇒ 12x2 - 143x - 12 = 0
tion 5 x2 + 25x + 2 5 = 0. ⇒ (12x - 1)(x - 12) = 0
(A) –5 5, 2 (B) 5 5, 2 −1
⇒x= ,12
(C) 25 5, 2 (D) –25 5, 2 12
As x is integer, x = 12
25
Sol. Sum = − =−5 5 Hence, the correct option is (C).
5
6. Construct a quadratic equation whose roots are recipro-
2 5 cals of the roots of the equation 2x2 + 8x + 5 = 0.
Product = =2
5 (A) 5x2 + 8x + 2 = 0 (B) 8x2 + 5x + 2 = 0
Hence, the correct option is (A). (C) 2x + 5x + 8 = 0
2
(D) 8x2 + 2x + 5 = 0
3. The sum of the squares of two consecutive positive Sol. The equation whose roots are reciprocals of the roots
integers added to their product is equal to 331. Find the of the equation ax2 + bx + c = 0 is given by cx2 + bx
two integers. + a = 0.
(A) 9, 10 (B) 10, 11 (C) 11, 12 (D) 12, 13 \ The required equation is 5x2 + 8x + 2 = 0.
Sol. Let the two consecutive positive integers be (x - 1), x. Hence, the correct option is (A).
then, (x - 1)2 + x2 + x(x - 1) = 331 7. The sum of the roots of a quadratic equation is 33
⇒ 3x2 - 3x - 330 = 0 and the product of its roots is 90. Find the sum of the
⇒ x2 - x - 110 = 0 squares of its roots.
⇒ (x - 11) (x + 10) = 0 (A) 909 (B) 8034 (C) 36 (D) 729
⇒ x =11 and x = -10 Sol. A quadratic equation whose sum of the roots is S and
As x cannot be negative, the integers are 10, 11 whose product of the roots is P has the sum of the
squares of its roots given by S2 – 2P.
(or) alternatively substitute the options and check.
The sum of the squares of the roots = 332 – 2(90) = 909.
Hence, the correct option is (B).
Hence, the correct option is (A).
4. The sum of squares of three consecutive positive
integers is 869. Find the numbers. 8. A quadratic equation in x has the sum of its roots as 19
(A) 14, 15, 16 (B) 15, 16, 17 and the product of its roots as 90. Find the difference of
(C) 16, 17, 18 (D) 17, 18, 19 its roots.

Unit I_Chapter 10.indd 106 27/04/2017 14:16:54


Chapter 10  •  Quadratic Equations  |  1.107

(A) 9 (B) 10 (A) a – b = 3 (B) a – b = –3


(C) 1 (D) 7739 (C) a – b = –21 (D) a – b = 21
Sol. A quadratic equation whose sum of the roots is 5 and Sol. x – 3x – 108 = 0 ⇒ (x – 12) (x + 9) = 0
2

whose product of the roots is P has the difference of its x = 12 or – 9


roots given by S 2 − 4 P . As α > β , α = 12, β = − 9, α − β = 12 −( −9) = 21
The difference of the roots is 192 − 4 (90) = 1 Hence, the correct option is (D).
Hence, the correct option is (C). 12. If a and b are the roots of the equation ax2 + bx + c = 0
9. The square of the sum of the roots of a quadratic equa- where c3 + abc + a3 = 0, which of the following is true?
tion E is 8 times the product of its roots. Find the value (A) ab 2 = 1 or a 2b = 1 (B) ab 3 = 1 or a 3b = 1
of the square of the sum of the roots divided by the (C) a = b  or a  = b (D)
2 2
a = b 3 or a 3 = b
product of the roots of the equation whose roots are Sol. c3 + abc + a3 = 0
reciprocals of those of E. 3
3 ⎛
c ⎞ ⎛ c ⎞⎛ b ⎞
(A) 8 (B) 1/8 (C) 1 (D) 4 Dividing each side by a , ⎜ ⎟ − ⎜ ⎟ ⎜ − ⎟ + 1 = 0
⎝ a ⎠ ⎝ a ⎠⎝ a ⎠
Sol. Let E be ax + bx + c = 0.
2
(ab )3 – (ab ) (a + b ) + 1 = 0
2
⎛ b⎞ c
Given: ⎜ − ⎟ = 8 dividing each side by ab,
⎝ a⎠ a
1
b2 = 8ac (ab)2 –(a + b) + =0
αβ
The equation whose roots are the reciprocals of the
roots of E is cx2 + bx + a = 0 ⎛ 2 1⎞ ⎛ 2 1⎞ 1 1
⇔ ⎜ α − ⎟ ⎜ β − ⎟ = 0 ⇔ a2 = or b  2 =
As b2 = 8ac ⎝ β⎠ ⎝ α⎠ β α
b 2 8ac Hence, the correct option is (A).
2
= 2
c c 13. Find the equation whose roots are thrice the roots of the
2 equation 2x2 – 15x + 18 = 0.
⎛ b⎞ a
\⎜− ⎟ = 8 (A) x2 + 45x + 324 = 0 (B) 2x2 – 45x + 81 = 0
⎝ c ⎠ c
(C) x + 45x – 324 = 0
2
(D) 2x2 – 45x + 162 = 0
2
⎛ b⎞ Sol. 2x - 15x + 18 = 0
2
⎜− c ⎟ ⎛ −15 ⎞ 15
⎝ ⎠ The equation has sum of its roots = − ⎜ ⎟= and
\ =8 ⎝ 2 ⎠ 2
a 18
product of the roots = = 9
c 2
We don’t need the coefficients of either of the equations. The equation whose roots are thrice the roots of the
Let the roots be a.b. above equation will have the sum of the roots being
Given : (a + b)2 = 8ab. ⎛ 15 ⎞ 45
thrice its sum of roots and have 3 ⎜ ⎟ = as sum of
For the second equation, the roots are 1/a, 1/b. We need ⎝ 2⎠ 2
to evaluate roots. It will have the product of its roots as nine times
the product of the roots of the original equation and
(α + β ) (α + β )
2 2
⎛1 1⎞ 1 hence 9 × 9 = 81 as product of roots.
2 (
⎜ + ⎟ = αβ ) = =8
⎝ α β ⎠ (1 α )(1 β ) (αβ ) αβ Hence, the required equation whose roots are thrice the

45 x
Hence, the correct option is (A). roots of 2x2 - 15x + 18 = 0 is x2 - + 81 = 0
2
10. The quadratic expression ax2 + bx + c has its maximum/
minimum value at or 2x2 - 45x + 162 = 0
b b −2b 2b Alternate method:
(A) – (B) (C) (D)
2a 2a a a In order to find out the equation whose roots are thrice
x
Sol. The maximum/minimum value of the quadratic expres- the roots of the given equation, substitute for x, in
−b the given equation. 3
sion ax2 + bx + c occurs at x = . 2
2a ⎛ x⎞ ⎛ x⎞
Hence, the correct option is (A). 2 ⎜ ⎟ − 15 ⎜ ⎟ + 18 = 0
⎝ 3⎠ ⎝ 3⎠
11. The roots of the equation x2 – 3x – 108 = 0 are a and b, ⇒ 2x2 - 45x + 162 = 0
where α > β . Which of the following holds true?
Hence, the correct option is (D).

Unit I_Chapter 10.indd 107 27/04/2017 14:16:57


1.108  |  Unit 1  •  Quantitative Aptitude

14. A person bought a certain number of oranges for `70. D can be equal to zero, if and only if either k = 0 or
If the price of each orange was `2 less, he would have k = –1
bought 4 more oranges for the same amount. Find the As k is the coefficient of x2, it cannot be zero; i.e. k ≠ 0.
number of oranges he bought originally.
As k is given to be a perfect square, it cannot be equal
(A) 12 (B) 10 (C) 18 (D) 15
to (–1); i.e. k ≠ –1.
Sol. Assume that the person bought x oranges for `70.
Hence D ≠ 0. ⇒ D > 0
70
Hence price of each orange is . If he bought 4 more As k and (k + 1) are two consecutive numbers and k is
x
oranges for `70, the price of each orange would be a perfect square, (k + 1) cannot be a perfect square.
70 70 Hence, D is positive, but not a perfect square. Hence,
which is 2 less than . the roots are always irrational.
x+4 x
70 70 Hence, the correct option is (C)
Hence = -2
x+4 x 17. Find the values of k for which the roots of x2 +
x(14 – k) – 14k + 1 = 0 are equal integers.
70 70
⇒ – = 2; (A) –11, –13 (B) –12, –16
x x+4 (C) –13, –15 (D) –11, –12
70 ( x + 4 ) − 70 x 70 x + 280 − 70 x Sol. If the roots of x2 + x(14 - k) - 14k + 1 = 0 are equal,
= = 2 ; x(x + 4) (14 - k)2 - 4(-14k + 1) = 0;
x ( x + 4) x ( x + 4)
⇒ 196 - 28k + k2 + 56k - 4 = 192 + 28k + k2 = 0.
= 140 ⇒ x2 + 4x - 140 = 0; (x + 14) (x - 10) = 0 Hence (k + 16) (k + 12) = 0 ⇒ k = -12 and -16
Hence, x = -14 or x = 10. To check which of the values of k, leads to equal, inte-
Since the number of oranges bought cannot be -ve, ger roots, substitute the value of k = -12, we get the
x cannot be -14, so x = 10. Hence 10 oranges were equation as x2 + 26x + 169 = 0
bought originally. (x + 13)2 = 0
Hence, the correct option is (B). Both the roots are integers and equal.
15. Find the value of p in the equation x2 + qx + p = 0, If k = -16, the equation is x2 + 30x + 225 = 0
where one of the roots of the equation is (2 + 3 ) and (x + 15)2 = 0
q and p are integers.
(A) 3 (B) 2 (C) 1 (D) –2 Both the roots are integers and equal.
Sol. Given p, q are integers, and one root of the equation is Hence, the correct option is (B).
18. If the roots of 2mx2 + 8x + 64m = 0 are real and equal,
2 + 3, the equation must have the conjugate 2- 3 as
find m.
the second root.
(A) 2/3 (B) 1/2 (C) 7/4 (D) 4/7
The product of the roots = ( 2 + 3 ) ( 2 − 3 ) = 4 - 3 = 1
Sol. Since the roots of the given equation are equal, the dis-
Hence the product of the roots = p = 1 criminant b2 - 4ac = 0
Hence, the correct option is (C). b2 - 4ac = 82 - 4(2m) (64)m = 26 - 222m26m
16. If k is a perfect square, the roots of the equation 4kx2 + ⇒ 26 - 22+7m = 0, Hence 26 = 22+7m i.e., 2 + 7m = 6 and
4 k x – k = 0 are 4
(A) always rational. hence 7m = 6 - 2 = 4; m =
7
(B) rational for only some of the values of k.
(C) always irrational. Hence, the correct option is (D).
(D) always complex. 19. If the roots of the equation ax2 + bx + c = 0 are a and b,
α β ⎛ 1 1⎞
Sol. The given equation is 4kx2 + 4 k x – k = 0 find the value of + − 2 ⎜ + ⎟ + 2αβ .
β α ⎝ α β⎠
The statements presented as options relate to the nature (A) (b2 + 2ac)/ac (B) (b2 – 2ac)/ac
of the roots. Hence, discriminant is to be considered. (C) (b2 + 4ac)/ac (D) None of these
( )
2
Discriminant (D) = 4 k – 4(4k) (–k) = 16k + 16k2
α β ⎛1 1⎞
= 16k(k + 1). Sol. + − 2 ⎜ + ⎟ + 2αβ
β α ⎝α β ⎠
As per data, k is a perfect square, i.e.+ k ≥ 0. Hence
D ≥ 0. Hence, roots are definitely real. It is to be decided α2 +β2 (β + α )
whether the roots are rational or irrational. = −2 + 2αβ
αβ αβ

Unit I_Chapter 10.indd 108 27/04/2017 14:16:59


Chapter 10  •  Quadratic Equations  |  1.109

(α + β ) 2 − 2αβ 2( β + α ) 23. Solve for x: 2 x + 3 + 4 x + 13 = 8


= − + 2αβ (A) 2 (B) –3 (C) 3 (D) 11
αβ αβ
2 Sol. Squaring on both sides, 2x + 3 + 4x + 13 – 82
⎛ −b ⎞ c ⎛ b⎞
⎜ a ⎟ − 2 a 2⎜ − a ⎟ = –2 2 x + 3 4 x + 13
⎝ ⎠ ⎝ ⎠ c
= − +2 6 x − 48
c c a = 2 x + 3 4 x + 13
a a 2
Squaring on both sides,
b2 a 2b 2c b 2 2b 2c 9x2 – 144x + 576 = 8x2 + 38x + 39
= × − 2 + + = + + −2
a2 c c a ac c a x2 – 182 x + 537 = 0
Hence, the correct option is (D). (x – 179) (x – 3) = 0
20. If 31 is split up into two parts such that the sum of x = 179 or 3
the squares of the two parts is 481, find the difference But x = 179 does not satisfy the equation.
between the two parts. \ x = 3.
(A) 7 (B) 5 (C) 3 (D) 1 Hence, the correct option is (C).
Sol. If 31 is split into parts a and b, 24. The roots of the equation 4x2 + 14x + 3 = 0 are a and b.
31 = a + b and 481 = a2 + b2 a b
Find + .
312 = (a + b)2 b a

Hence a2 + b2 + 2ab = 961 7 3 7 3 7 3 7 3
(A) – (B) – (C) (D)
2ab = 961 - 481 = 480 6 3 3 6
4ab = 480 × 2 = 960 Sol. Both the roots of the given equation are negative, i.e.,
a2 + b2 + 2ab - 4ab = a2 + b2 - 2ab a < 0, b < 0.
= (a - b)2 = 961 - 960 = 1⇒ (a - b) = 1 = ±1 \ a/b > 0 and a / b + b / a > 0
The difference of (a - b) = |(a - b)| = 1 a b
a b
Hence, the correct option is (D). + = +
b a b a
21. Solve for x: x4 – 42x2 + 216 = 0
a+b −( a + b)
± 6 , ±6 (B)
(A) ±2 6 , ±6 = =
ab ab
(C) ±3 6 , ±6 (D) ±4 6 , ±6
Sol. x – 42x + 216 = 0
4 2 Sum of the roots 14 4 7 3
= = =
⇒ (x2) – 42x2 + 216 = 0 Product of the roots 4 3 3
⇒ (x2 – 6) (x2 – 36) = 0 Hence, the correct option is (C).
⇒ x2 – 6 = 0 or x2 – 36 = 0 25. The equation x2 – 2x – 8 = 0 will have
⇒x=± 6,±6 (A) the numerically larger root as positive.
(B) the numerically larger root as negative.
Hence, the correct option is (A).
(C) both roots as negative.
1 ⎛ 1⎞ 5 (D) both the roots as positive.
22. If x2 + − 2 ⎜ x − ⎟ − = 0, which of the following ⎛ 2⎞
x 2
⎝ x⎠ 4 Sol. x2 - 2x - 8 = 0 The sum of the roots = ⎜ − ⎟ = 2 and
1 ⎝ 1⎠
can be the value of x – ? the product of the roots = -8. Since the product of the
x roots is negative, one of the roots is positive and the
(A) 7/2 (B) 1/2 other negative. Since the sum of the roots is positive,
(C) –1/2 (D) –3/2 the numerically larger root is positive.
2
⎛ 1⎞ ⎛ 1⎞ 5 Hence, the correct option is (A)
Sol. ⎜ x − ⎟ + 2 − 2 ⎜ x − ⎟ − = 0
⎝ x ⎠ ⎝ x⎠ 4
1
Substituting the choices in place of x – in the equa-
x
tion above, we see that only choice (B) satisfies it
Hence, the correct option is (B).

Unit I_Chapter 10.indd 109 27/04/2017 14:17:02


Chapter 11 Inequalities
Hints/Solutions
Practice Problems 1 7. 6x + 8 > 7x – 9 ⇒ 17 > 7x – 6x
⇒ 17 > x
1. When a < b and c < 0
⇒ x < 17
Then ac > bc is true.
4x – 7 < 6x – 3 ⇒ 4x – 6x < 7 – 3
Hence, the correct option is (C).
⇒ – 2x < 4
2. For any two real numbers, p, q
p ⇒x>–2
1. <1
q \ Solution set is (–2, 17).
⇒ p < q is not true always. Hence, the correct option is (C).
p
2. p > 0, q > 0 and > 1 8. Given x − 5 < 9
q
⇒ p > q is true. We know that if x < a ⇒ – a < x < a
p x − 5 < 9 ⇒ – 9 < x – 5 < 9
3. if > 1
q ⇒ – 4 < x < 14
⇒ p > q is not true always. Hence, the correct option is (B).
Hence, the correct option is (B). x +5
3. Given 5x – 8 < 2x + 9 9. ≥0
x−2
⇒ 3x < 17; ⇒ (x + 5) (x – 2) ≥ 0
17 Critical points are – 5, 2
x < ;(5)
3
 × 
4x + 7 > 7x – 8
−∞ −5 2 ∞
⇒ 15 > 3x
⇒ x < 5 (6) When x = 0, the inequation is not satisfied.
The common solution for (5) and (6) is x < 5; (– ∞, 5) The solution set is R – (– 5, 2]
Hence, the correct option is (B). \ The integral values of x that do not satisfy the
4. Given 5x2 – 3x – 2 ≥ 0 ­inequation is – 4, – 3, –2, –1, 0, 1, 2.
(x – 1) (5x + 2) ≥ 0 \ i.e., 7
−2 Hence, the correct option is (A).
The critical points are, 1,
5
10. Given (x + 5) (x + 9) (x + 3)2 < 0
Ι ΙI ΙΙΙ
Since (x + 3)2 is always positive
 ∞ 
(x + 5) (x + 9) < 0
2/3 1
⇒–9<x<–5
when x = 0, the inequality is not true Solution set is (– 9, – 5)
I and II regions satisfies the inequality Hence, the correct option is (B).
−2 11. 8 - 3x ≤ 5 ⇒ 3x ≥ 8 - 5
\ solution set is ( −∞, ] ∪ [1, ∞)
5 ⇒ 3x ≥ 3 ⇒ x ≥ 1 (7)
⎛ −2 ⎞ 4x + 5 ≤ -7 ⇒ 4x ≤ -7 - 5
\ R – ⎜ ,1 ⎟
⎝ 5 ⎠ ⇒ 4x ≤ -12 ⇒ x ≤ -3(8)
Hence, the correct option is (B). There is no x that satisfies both (7) and (8)
5. x2 - 9x - 36 < 0 \ The range is f, the empty set.
⇒ (x - 12) (x + 3) < 0 Hence, the correct option is (D).
⇒ x ∈ (-3, 12) ⎛ 1⎞
x

12. For any x ≥ 1 we have 2 ≤ ⎜1 + ⎟ < 2.8


Hence, the correct option is (A). ⎝ x⎠
6. The given options are properties of modulus, Consider choice (A)
30 30
\ all are true. 3130 ⎛ 31 ⎞ 1 ⎛ 30 + 1 ⎞ 1
31 = ⎜ ⎟ ⋅ =⎜ ⎟ ⋅
Hence, the correct option is (D). 30 ⎝ 30 ⎠ 30 ⎝ 30 ⎠ 30

Unit I_Chapter 11.indd 110 27/04/2017 14:17:10


Chapter 11  •  Inequalities  |  1.111

⎛ 1 ⎞
30
1 2 ⋅8 But ac = bd = 2
= ⎜1 + ⎟ ⋅ < <1
⎝ 30 ⎠ 30 30 \ a2 + b2 + c2 + d 2 ≥ 2(2 + 2) = 8
\ The minimum value of a2 + b2 + c2 + d 2 is 8.
3130
i.e., < 1 i.e., 3130 < 3031. Hence, the correct option is (C).
3031
Choice (A) is false 16. Given: x , y > 0 and x + y = 3
x+ y
Similarly, it can be seen that (2) is false Now ≥ xy i.e., (x + y)2 ≥ 4xy
2
Consider choice (C) But x + y = 3
30
(155) 29 ⎛ 155 ⎞ 1 9
=⎜ ⋅ \ 4xy ≤ 9 or xy ≤ i.e., xy ≤ 2.25.
30 ⎟ 4

(150) ⎝ 150 ⎠ 155
Hence, the correct option is (C).
30
⎛ 1 ⎞ 1 2 ⋅8 17. Case (i):- x ≥ 16
 = ⎜1 + ⎟ ⋅ < <1
⎝ 30 ⎠ 155 155 \ |x - 16| = x - 16
(155) 29 So the relation becomes
\ < 1 i.e., (155)29 < (150)30.
(150)30 x - 16 > x2 - 7x + 24
\ Choice (C) is true. ⇒ x2 - 8x + 40 < 0
Hence, the correct option is (C). ⇒ x2 - 8x + 16 + 24 < 0
21 21 ⇒ (x - 4)2 + 24 < 0
13. -|x - 3| + has the maximum value of when
2 2 But (x - 4)2 + 24 is positive for all real x.
|x - 3| = 0 i.e., when x = 3. So there cannot be any solution in this domain.
Hence, the correct option is (D). Case (ii): x < 16
⎧ −5 |x - 16| = 16 - x
⎪⎪ 3 x + 5 if x ≥ 3 The relation becomes
1 4. |3x + 5| = ⎨
16 - x > x2- 7x + 24
⎪ −3 x − 5 if x < −5
⎪⎩ 3 ⇒ x2 - 6x + 8 < 0 ⇒ (x - 4) (x - 2) < 0
−5 ⇒ x ∈ (2, 4)
Case 1: If x ≥ . which is consistent with x < 16
3
The inequality is 3x + 5 < 5x - 11 i.e., 2x > 16 Hence the range is (2, 4).
⇒x>8 Hence, the correct option is (D).
\ x > 8 is a possible set of solutions 18. Let a = x2 + x, b = x3 + 1
−5 We need to find a range where a < b
Case 2: If x < . i.e., x2 + x < x3 + 1.
3
The inequality is -3x - 5 < 5x -11 ⇒ x3 - x2 - x + 1 > 0 or x2 (x - 1) - 1 (x - 1) > 0
3 ⇒ (x2 - 1) (x - 1) > 0 or (x - 1)2 (x + 1) > 0
⇒ 8x > 6 ⇒ x >
4 as (x - 1)2 > 0 except at x = 1, x + 1 > 0 i.e., x > -1.
This is not a possible set of solutions as this contradicts Hence, the range is (-1, 1) ∪ (1, ∞).
−5 Hence, the correct option is (C).
the assumption, x <
3 19. Consider 4x2 + x + 4
\ The solution set is (8, ∞).
⎛ ⎛ 1⎞ ⎞ ⎛ 1 ⎞
Hence, the correct option is (A). = x ⎜ 4 ⎜ x + ⎟ + 1⎟ ≥ 9x ⎜∵ x + x ≥ 2 ⎟
⎝ ⎝ x⎠ ⎠ ⎝ ⎠
a2 + c2
15. We know that ≥ ac (A.M of a2, c2 compared to similarly 5y2 + y + 5 ≥ 10y and 7z2 + z + 7 ≥ 15z
2
G.M of a2, c2)

(4x 2
)( )(
+ x + 4 5 y2 + y + 5 7z2 + z + 7 )
i.e., a2 + c2 ≥ 2ac xyz
Similarly b2 + d2 ≥ 2bd (9 x )(11 y )(15 z )
≥ = 1485
Adding both these inequalities we get xyz
a2 + b2 + c2 + d2 ≥ 2(ac + bd) Hence, the correct option is (D).

Unit I_Chapter 11.indd 111 27/04/2017 14:17:12


1.112  |  Unit 1  •  Quantitative Aptitude

2 0. 2x2 – 5x – 8 ≤ |2x2 + x|(9) 1 6


=1+ =
2x2 + x = 0 ⇒ x = 0 or x = –1/2 5 5
If x ≤ –1/2 or x ≥ 0, then 2x2 + x ≥ 0 and |2x2 + x| Hence, the correct option is (C).
= 2x2 + x. 23. x = |a|b and 5 ≤ |b|
If –1/2 < x < 0, then 2x2 + x < 0 and |2x2 + x| = –2x2 – x \ xb = |a|b2 ≥ 25 |a|
Let x ≤ –1/2 or x ≥ 0 Consider a – xb
(9) ⇒ 2x2 – 5x – 8 ≤ 2x2 + x From a, we are subtracting a quantity that is greater than
⇒ 6x + 8 ≥ 0 ⇒ x ≥ –4/3 or equal to 25 |a|. If a = 0, this could be 0 or negative.
\ x ∈ [–4/3, a) – (–1/2,0) (10) But if a ≠ 0, this would be negative.
Let –1/2 < x < 0 \ a – xb ≤ 0.
(9) ⇒ 2x2 – 5x – 8 ≤ –2x2 – x Hence, the correct option is (D).
⇒ 4x2 – 4x – 8 ≤ 0 ⇒ x2 – x – 2 ≤ 0 24. Given; x − x − 2 =6
⇒ (x – 2) (x + 1) ≤ 0 ⇒ –1 ≤ x ≤ 2
When x < 2, x − 2 = - (x - 2)
\ x ∈ (–1/2, 0) (11)
From (10), (11) x ∈ [–4/3, ∞) \ |x–(–(x – 2) = 6 ⇒ (2x - 2) = ± 6
Hence, the correct option is (A). x - 1 = ± 3
x −5 x = 4 or - 2
21. > 4;
x+7 since x < 2; so x = - 2 is the only solution.
x −5 when x > 2 the equation is not true.
-4>0 Hence, the correct option is (B).
x+7

x − 5 − 4 x − 28 − 3 x − 33 x 2 y + y 2 z + z 2 x + xy 2 + yz 2 + zx 2
⇒ >0⇒ >0 25.
x+7 x+7 xyz

x + 11 ( x + 11) ( x + 7) x2 y + x2 z + y2 z + y2 x + z2 x + z2 y
⇒ <0⇒ <0 =
x+7 ( x + 7) 2 xyz
⇒ (x + 7) (x + 11) < 0
x 2 ( y + z) + y 2 ( z + x) + z 2 ( x + y)
When -11 < x < -7, the above inequation is true. =
xyz
The number of integral values between -11 and -7 is 3.
\ Hence the number of integral solutions is 3. x ( y + z) y ( z + x) z ( x + y)
= + +
Hence, the correct option is (D). yz xz xy
22. Given: 1 ≤ x ≤ 3, 2 ≤ y ≤ 5. x x y y z z
= + + + + +
x z y x z y x
is minimum when x is minimum and y is maximum,
y
x 1 ⎛x z⎞ ⎛x y⎞ ⎛ z y⎞
\ The minimum value of = = ⎜ + ⎟+⎜ + ⎟+⎜ + ⎟
y 5 ⎝ z x⎠ ⎝ y x⎠ ⎝y z⎠
x+ y Since the sum of a number and its reciprocal is ≥2.
The minimum value of
y \ The minimum value of the sum is = 2 + 2 + 2 = 6
x
=1+
y Hence, the correct option is (A).

Practice Problems 2 –7 ≤ y ≤ –3 and 2 ≤ x ≤ 5


1. If 2 ≤ x ≤ 5 and –7 ≤ y ≤ –3, then the minimum value of \ 9 ≤ y2 ≤ 49
which of the following is the least? \ 4 ≤ x2 ≤ 25
(A) x2y (B) xy2 (C) 3xy (D) –2xy2 We note that xy2 ≥ 0.
Sol. The 4 expressions in the options involve x, x2, y, y2. \ We need to look at only choices (C), (D), (A). The
\ We first determine the range of values for these 4 range of possible values for these are tabulated below.
quantities.

Unit I_Chapter 11.indd 112 27/04/2017 14:17:15


Chapter 11  •  Inequalities  |  1.113

–175 ≤ x2y ≤ –12, and 18 ≤ xy2 ≤ 245 x < 2 or x > 3


⇒ –490 ≤ –2xy2 ≤ –36 x2- 3x + 2 > 0; (x - 2) (x - 1) > 0
–35 ≤ xy ≤ –6 x < 1 or x > 2
⇒ –105 ≤ 3xy ≤ –18 \ The common solution is x < 1 or x > 3
\ The minimum value of –2xy2 is the least. Hence, the correct option is (B).
Hence, the correct option is (D). x −3
5. For how many integral values of x, is < 0?
2. The solution set for the inequation (x2 + x + 1)x < 1 is x+2
(A) (0, ∞) (B) (–1, ∞) (A) 3 (B) 5 (C) 4 (D) 7
(C) (–∞, 0) (D) (–∞, –1) x −3
Sol. Given: <0
Sol. Given: (x + x + 1)x < 1
2
x+2
Taking logarithms on both sides to a strong base,
( x − 3) ( x + 2)
log (x2 + x + 1)x < log 1 ⇒ <0
( x + 2) 2
⇒ x log (x2 + x + 1) < 0 ⇒ (x + 2) (x - 3) < 0
This could be true if x > 0 and log(x2 + x + 1) < 0, i.e. -2 < x < 3
when x > 0 and x2 + x + 1 < 1.
The number of integral values of x that satisfy is 4.
But this is not possible.
Hence, the correct option is (C).
\ We consider only x < 0 and log(x2 + x + 1) > 0, i.e.,
x2 + x + 1 > 1 ⇒ x(x + 1) > 0 i.e., x < –1. 6. Solve the system of inequations 3x + 17 < 5x – 19 and
4x + 15 > 9x + 21.
\ The solution set is (–∞, –1). (A) (18, ∞) (B) R
Hence, the correct option is (D). ⎛ − 6⎞
(C) f (D) ⎜⎝ − ∞, ⎟⎠
3. The number of integral values of x, that satisfy the 5
­inequation x − 3 + x − 4 ≤ 7 is Sol. Given: 3x + 17 < 5x - 19 and 4x + 15 > 9x + 21
(A) 7 (B) 6 (C) 8 (D) 9 3x + 17 < 5x - 19
Sol. Given: x − 3 + x − 4 ≤ 7 3x - 5x < - 19 - 17 ⇒ -2x < -36
Case 1: When x ≥ 4, x − 3 = (x – 3) and x − 4 = x - 4 x > 18  (12)
x − 3 + x − 4 ≤ 7 ⇒ x - 3 + x- 4 ≤ 7 4x + 15 > 9x + 21

2x ≤ 14 4x - 9x > 21 - 15
x ≤ 7 -5x > 6
\ x ∈ [4, 7] 6
x < − (13)
Case 2: When x ≤ 3, x − 3 = 3 - x and x − 4 = 4 - x 5
From (12) and (13) we see that there is no common
x − 3 + x − 4 ≤ 7 ⇒ 3 - x + 4 - x ≤ 7 - 2x ≤ 0
solution.
x ≥ 0
The solution set is { }
\ x [0, 3]
Hence, the correct option is (C).
When x ∈ (3, 4), x − 3 = x - 3; x − 4 = 4 - x
7. For which of the following values of x does the expres-
x −3 + x −4 ≤ 7 ⇒ x - 3 + 4 - x ≤ 7 sion x + 3 + x − 5 + 7 have its least value?

⇒ 1 ≤ 7 is always true (A) 4 (B) –6 (C) 7 (D) –5
\ The solution set = x ∈ [0, 7] Sol. Let f(x) = x + 3 + x − 5 + 7
The number of integral solutions of the given inequa-
tion is 8. When f(x) = x > 5, x + 3 = x + 3 and
Hence, the correct option is (C). x−5 = x - 5

4. The solution of the system of inequation x2 – 5x + 6 > 0 \ f(x) = x + 3 + x - 5 + 7 = 2x + 5
and x2 – 3x + 2 > 0 is when x < - 3, x + 3 = -(x + 3) and x − 5 = -(x - 5)
(A) (- ∞, 2) (B) (- ∞, 1) ∪ (3, ∞) \ f(x) = - x - 3 - x + 5 + 7 = 9 – 2x
(C) (1, 3) (D) (1, ∞)
when - 3 < x < 5, x + 3 = x + 3
Sol. Given: x2 - 5x + 6 > 0
x − 5 = -(x - 5)
(x - 2) (x - 3) > 0

Unit I_Chapter 11.indd 113 27/04/2017 14:17:17


1.114  |  Unit 1  •  Quantitative Aptitude

f (x) = x + 3 - x + 5 + 7 = 15. 11. If p, q, r are positive and


\ The minimum value of f (x) is 15. ( 2 p 2 + p + 2)(3q 2 + q + 3)( r 2 + r + 1)
x =  , then x
f (x) is minimum when x ∈ [-3, 5] 15 pqr
cannot be
Hence, the correct option is (A). (A) 8 (B) 10 (C) 7 (D) 6
8. The maximum value of the expression 10 - 3 x + 5 is
⎛ ⎛ 1⎞ ⎞
−5 Sol. Consider 2p2 + p + 2 = p ⎜ 2 ⎜ p + ⎟ + 1⎟ = p ( 2 ( ≥ 2 ) + 1)
(A) 10 (B) (C) 20 (D) 0 ≥ 5p ⎝ ⎝ p⎠ ⎠
3
Sol. Let f (x) = 10 - 3 x + 5 3q2 + q + 3 ≥ 7q and r2 + r + 1 ≥ 3r

We know that 3 x + 5 is always positive (2 p 2


)( )(
+ p + 2 3q 2 + q + 3 r 2 + r + 1) ≥ 5 p × 7q × 3r = 7

15 pqr 15 pqr
10 - 3 x + 5 ≤ 10
\ The maximum value of f (x) = 10 ∴ x cannot be 6.
Hence, the correct option is (D).
Hence, the correct option is (A).
12. The range of values of k for which –x2 + 3kx + 5k
9. For how many integral values of x, is the expression
+ 1 < 0 is
9 – 4x – 5x2 non-negative?
⎡ 9 ⎤
(A) 7 (B) 6 (C) 5 (D) 3 (A) (∞, –2) ∪ (–2/9, ∞) (B) ⎢ − 2 , −2⎥
⎣ ⎦
Sol. Given: 9 - 4x - 5x ≥ 0
2
(C) (–2, –2/9) (D) (-∞, ∞)
5x2 + 4x - 9 ≤ 0 Sol. Given –x2 + 3kx + 5k + 1 < 0
(x - 1) (5x + 9) ≤ 0 ⇒ x2 – 3kx – 5k – 1 > 0
−9
\ Critical points are ,1 The expression is always positive, if b2 – 4ac < 0
5
 X
⇒ 9k2 + 4(5k + 1) < 0
X
−9 ⇒ 9k2 + 20k + 4 < 0
1
5 ⇒ 9k2 + 18k + 2k + 4 < 0
when x = 0 the above inequation is true. (k + 2)(9k + 2) < 0
⎛ −2 ⎞
⎡−9 ⎤ k ∈ ⎜ −2,
The solution is ⎢ ,1⎥ ⎝ 9 ⎟⎠
⎣ 5 ⎦
Hence, the correct option is (C).
The number of integral values of ‘x’ is 3.
13. The range of x for which |x2 + x – 2| ≤ x2 – x is
Hence, the correct option is (D).
(A) [–1, 1]
10. Find the complete range of values of x for which (B) (–∞, –1]
|x + 4| < 3x – 7. (C) [1, ∞)
(A) (11/2, ∞) (B) (3/4, ∞) (D) No such value of x exists
(C) (– ∞, –3/4) (D) (3/4, 11/2)
Sol. |x2 + x – 2| ≤ x2 – x (14)
Sol. |x + 4| < 3x - 7.
x + x – 2 = ( x +2) (x – 1)
2
As the modulus of any quantity is non-negative and
If x ≤ – 2 or x ≥ 1, then x2 + x – 2 ≥ 0 and |x2 + x – 2|
3x - 7 being more than the modulus of a quantity would
= x2 + x – 2.
imply that.
If – 2 < x < 1, then x2 + x– 2 < 0 and |x2 + x – 2| = –x2 –
3x - 7 > 0
7 x + 2.
⇒x> Let x ≤ –2 or x ≥ 1.
3
Now when x > 7/3 (14) ⇒ x2 + x – 2 ≤ x2 – x
|x + 4| = x + 4 ⇒x–1≤0
So the relation is reduced to x + 4 < 3x - 7 \ x ∈ (–a, –2] (15)
⇒ 2x > 11 Let – 2< x < 1
11 (14)  ⇒ –x2 – x + 2 ≤ x2 – x
⇒x>
2 ⇒ –2x2 + 2 ≤ 0
⎛ 11 ⎞
Thus the range of x is ⎜ , ∞ ⎟ .
⎝2 ⎠ ⇒ 2x2 – 2 ≥ 0 ⇒ x2 ≥ 1
Hence, the correct option is (A). ⇒ x ≤ –1 or x ≥ 1

Unit I_Chapter 11.indd 114 27/04/2017 14:17:19


Chapter 11  •  Inequalities  |  1.115

\ x ∈ (–2, –1] (16) 7 7


x < − and x > − such a value of x does not exist.
From (15), (16), x ∈ (–∞, –1] 5 5
−2
Hence, the correct option is (B). \ The solution set of the inequation is <x<3
3
14. If x, y, z are positive real numbers and xyz = 216, which Hence, the correct option is (A).
of the following is not a possible value of xy + yz + zx? x 2 − 7 x + 10
16. The solution set for the inequality 2 < 1 is
(A) 326 (B) 433 (C) 291 (D) 96 x + 6 x − 40
Sol. For positive numbers, the GM is less than or equal to ⎛ 70 50 ⎞ ⎛ 50 ⎞
the AM. ⎜⎝ , ⎟⎠ (B)
(A) ⎜⎝ −10, ⎟⎠ ∪ (4, ∞)
13 3 13
xy + yz + zx
\ 3 ( xy ) ( yz ) ( zx ) ≤ (C) (4, ∞) (D)
R – (–4, 0)
3
Sol. Given:
i.e., xy + yz + yx ≥ 3(xyz)2/3 = 3 (216)2/3 = 108
\ 96 is not a possible value. x 2 − 7 x + 10
<1
x 2 + 6 x − 40
Hence, the correct option is (D).
15. The solution set for the inequation 3x + 2 < |2x + 5| x 2 − 7 x + 10
⇒ -1<0
< 8x + 9 is x 2 + 6 x − 40
⎛ 2 ⎞ ⎛2 ⎞
(A) ⎜⎝ − , 3⎟⎠ (B) ⎜⎝ , 3⎟⎠ x 2 − 7 x + 10 − x 2 − 6 x + 40
3 3 ⇒ <0
x 2 + 6 x − 40
⎡ 2 ⎤ ⎛ −2 ⎞
⎢ − 3 , 3⎥ (D)
(C) ⎜⎝ , 4⎟⎠ −13 x + 50
⎣ ⎦ 3 ⇒ <0
x 2 + 6 x − 40
Sol. Given: 3x + 2 < 2 x + 5 < 8x + 9
(13 x − 50)( x 2 + 6 x − 40)
⇒ >0
−5 ( x 2 + 6 x − 40) 2
Case 1: x >
, 2 x + 5 = 2x + 5
2
⇒ (13x - 50) (x + 10) (x - 4) > 0
3x + 2 < 2x + 5 < 8x + 9
50
consider 3x + 2 < 2x + 5 The critical points are , -10, 4
13
x < 3 (17)
2x + 5 < 8x + 9 X  X 
⇒ - 6x < 4 −10 50 4
−4 13
x > (18)
6
−2 when x = 0 the inequation is true
From (17) and (18) <x<3
3 \ The values of x lying in the 2nd and fourth regions
−5 satisfy the above inequations.
Case: 2x < then 2 x + 5 = -(2x + 5)
2 ⎛ 50 ⎞
Solution set is ⎜ −10, ⎟ ∪ (4, ∞)
3x + 2 < - (2x + 5) < 8x + 9 ⎝ 13 ⎠
Consider 3x + 2 < -(2x + 5) Hence, the correct option is (B).
3x + 2x < -5 - 2 17. If x ≤ 4, y ≥ –2, which of the following is necessarily
5x < -7 true?
7 (A) xy ≥ –8 (B) x + y > 2
x < − (19) (C) x – y ≤ 6 (D) (A) and (B)
5
-(2x + 5) < 8x + 9 Sol. Given: x ≤ 4 (20)
-10x < 9 + 5 and y ≥ -2
-10x < 14 y ≥ -2 ⇒ - y ≤ 2 (21)
−14 Adding (20) and (21) we get
⇒x>
10 x - y ≤ 4 + 2 = x - y ≤ 6  [\ (C) is true]
7 If x = –10, y = 1, xy = –10  [\ (A) and (B) are false]
x > −
5 Hence, the correct option is (C).

Unit I_Chapter 11.indd 115 27/04/2017 14:17:23


1.116  |  Unit 1  •  Quantitative Aptitude

18. Let a1, a2¸ …, a3n be 3n positive numbers such that their = x –3 for all values of x
product is 1. The minimum value of a1 + a2 + a3 + … Hence, the correct option is (C).
+ a3n is
22. If a > b and c > 0, then which of the following ­statements
(A) n (B) 31/n
is/are true?
(C) 3 n
(D) 3n
(A) a + c > b + c (B) a–c>b–c
Sol. If the product of several numbers is constant, their sum (C) ac > bc (D) All the above
is minimum when they are all equal.
Sol. When a > b and c > 0
As a1a2. . . a3n = 1, the minimum value of a1 + a2 + … +
⇒ a + c > b + c is true
a3n is 1 + 1 + … + 1 (3n times) = 3n.
a - c > b - c is also true
Hence, the correct option is (D).
ac > bc is also true.
19. If a, b, c, d are positive real numbers, the minimum
Hence, the correct option is (D).
⎛ 1 1 1 1⎞
value of the product (a + b + c + d) ⎜ + + + ⎟ is 23. Solve: 2x – 3 ≥ 7 and 5x – 7 < 3.
⎝ a b c d⎠
(A) f (B) [5, ∞)
(A) 16 (B) 32 (C) 24 (D) 36 (C) (–∞, 2) (D) [2, 5]
ol. We know that
S Sol. 2x - 3 ≥ 7 ⇒ 2x ≥ 10 ⇒ x ≥ 5. (22)
AM (a, b, c, d) ≥ HM (a, b, c, d) 5x - 7 < 3 ⇒ x < 2 (23)
a+b+c+d 4 From (22) and (23), there are no common values of x.

4 1 1 1 1 \ The solution is the empty set.
+ + +
a b c d Hence, the correct option is (A).
⎛1 1 1 1⎞ 24. Which of the following is always true?
(a + b + c + d) ⎜ + + + ⎟ ≥ 16
⎝a b c d⎠ (A) x > y ⇒ x2 > y2.
\ The minimum value is 16. (B) x > y ⇒ x2 < y2.
(C) If x > 0, y > 0 and x > y, then x2 > y2.
Hence, the correct option is (A).
(D) None of the above.
20. At what value of x is |2x – 7| – 8 minimum?
Sol. –2 > – 5 but (– 2)2 < (– 5)2. Option (A) is not true
7 always.
(A) –8 (B)
2 5 > 2 but (5)2 > (2)2. Option (B) is not true always.
7
(C) – (D) 15 But when x > 0, y > 0, and x > y ⇒ x 2 > y2.
2
Option (C) is always true.
Sol. |2x - 7| - 8 is minimum when |2x - 7| is minimum. This
7 Hence, the correct option is (C).
happens when x = . 25. If x + 7 < 10, then the solution set for the inequality is
2
Hence, the correct option is (B). (A) (– ∞, –17) ∪ (3, ∞) (B) (3, 17)
21. Max [ min( x + 6, x − 3), min( x + 5, x − 7)] = (C) (– 17, – 3) (D) (–17, 3)
(A) x + 6 (B) x + 5 Sol. x + 7 < 10
(C) x – 3 (D) x – 7 ⇒ – 10 < x + 7 < 10 ⇒ –17 < x < 3
Sol. min (x + 6, x – 3) = x – 3, for all values of x. \ solution set is (– 17, 3)
min (x + 5, x – 7) = x – 7, for all values of x. Hence, the correct option is (D).
\Required value = max(x – 3, x – 7)

Unit I_Chapter 11.indd 116 27/04/2017 14:17:24


Chapter 12 Progressions
Hints/Solutions
Practice Problems I 7. Sn = 5n2 + 2n
⇒ Sn - 1 = 5n2 + 2n - 1
1. Let the first term and the common difference of the
series be a and d respectively. nth term = Sn – Sn– 1
Then, T6 = a + 5d = 30 (9) = 5 n2 + 2n – {5(n – 1)2 + 2 (n – 1)
T11 = a + 10d = 55 (10) = 10n­– 3
Solving (9) and (10) Hence, the correct option is (B).
a = 5, d = 5 8. a = 4r = 2
T21 = a + 20d ⇒ 5 + 20 (5) = 105
let 64 2 be the nth term of the geometric progression.
Hence, the correct option is (B).
2. Let the first term of the arithmetic progression be a. Tn = 4 ( 2 )n–1 = 64 2
a + 2a = 9 ⇒ 2n –1 = 29/2 ⇒ n = 10
a = 3 Hence, the correct option is (C).
15th term = a + 14a 9. Sixth term = 2 (3)5 = 486
15a = 45 Hence, the correct option is (B).
Hence, the correct option is (C). 10. The sum of the first n terms of a geometric progression
whose first term is a and whose common ratio is r is
3. x + 4, 6x – 2 and 9x – 4 are three consecutive terms in
an A.P.
given by
(
a rn − 1 )
⇒ 6x – 2 – (x + 4) = 9x – 4 – (6x – 2) r −1
6x – 2 – x – 4 = 9x – 4 – 6x + 2
Sum of the first 4 terms =
(
6 24 − 1 ) = 90
5x – 6 = 3x – 2 ⇒ 2x = 4
2 −1
x = 2
Hence, the correct option is (A).
Hence, the correct option is (A).
11. Let the common ratio of the geometric progression be r.
4. a = 32, d = –4
r 3 = 8; r = 2
tn = 32 +(n – 1) (–4) = 4
⇒n=8 Sum of the first terms =
(
1 27 − 1
= 127
)
8(32 + 4) 2 −1
sum of the series = = 144
2 Hence, the correct option is (C).
Hence, the correct option is (A). 12. Sum of the terms of an A.P. = (number of terms) × (the
5. Sum of the first n terms of an arithmetic progression middle term of the A.P.), if number of terms (n) is odd.
n Since, n is 37 (odd), we have sum of the terms of the
whose first term is a and common difference is d =
2 A.P. = 703 = 37(middle term of the A.P.). Middle term
[2a + (n – 1) d]
of the A.P. = (703/37) = 19.
Sum of the first 31 terms of the arithmetic progression
Hence, the correct option is (D).
31 ⎛8⎞ 31
= [2 (6) + (31 – 1) ⎜ ⎟ ] = [92] = 1426 13. Given series is 1, (1 + 2), (1 + 2 + 3), (1 + 2 + 3 + 4) …
2 ⎝3⎠ 2
\ The nth bracket is
Hence, the correct option is (C).
n( n +1)
6. Let the five terms of the arithmetic progression be 1 + 2 + 3 + 4 + … + n i.e. n =
2
a – 2d, a – d, a, a + d, a + 2d.
sum = a – 2d + a – d + a + a + d + a + 2d = 70 n( n +1)
Sn = Stn = S
⇒ 5a = 70 ⇒ a = 14 2
Product of extremes = (14 – 2d) (14 + 2d) = 132 1
= [Sn2 + Sn]
⇒ 196 – 4d2 = 132 ⇒ d = ±4 2
The five terms are 6, 10, 14, 18, 22 or alternatively, 1 ⎡ n( n + 1)( 2n + 1) n( n + 1) ⎤
substitute the options and check. = +
2 ⎢⎣ 6 2 ⎥⎦
Hence, the correct option is (C).

Unit I_Chapter 12.indd 117 27/04/2017 14:17:33


1.118  |  Unit 1  •  Quantitative Aptitude

n( n + 1)( 2n + 1) n( n +1) a( r 4 − 1)
Sn = + and = 30 (14)
12 4 r −1
( 20)( 21)( 41) ( 20)( 21) a( r 8 − 1) a( r 4 − 1)
\ S20 = + Dividing (13) by (14) we have /
12 4 r −1 r −1
= 1435 + 105 = 1540 510
= r4 + 1 = = 17.
Hence, the correct option is (A). 30
14. As a, c and b are in geometric progression, r4 = 17 - 1 = 16

 c2 = ab c4 = a2b2 r = ± 4 16 = ± 2
As a2 + b2, a2 + c2 and b2 + c2 are in geometric 510 ( r − 1)
progression, First term of the G.P., a =
r8 − 1
(a2 + c2)2 = (a2 + b2) (b2 + c2) As first term is positive, r = 2 is taken.
a4 + 2a2c2+c4 = a2b2 + a2c2 + b2c2 + b4
510 ( 2 − 1) 510 (1)
⇒ a4 + a2c2 + c4 = a2b2 + b2c2 + b4 a = = = 2.
2 −1
8
255
since c4 = a2b2 Hence, the correct option is (A).
a2 (c2 + a2 + b2) = b2 (a2 + b2 + c2) 18. 2(y + 3) - 2y = 2y + 6 - 2y = 6 = Common difference.
a2 = b2 As per the G.P. given,
⇒ a, b have the same sign. 5 y − 1 2( y + 1)
= ⇒ (5y - 1) (x + 2) = [2 (y + 1)]2
\ a = b and c = ±a 2( y + 1) x+2

Hence, the correct option is (A).
(5y - 1) (6 - 2y + 2) = 4 (y + 1)2
15. The first number between 450 and 950 which is divis-
⇒ 14y2 - 34y + 12 = 0 ⇒ y = 2.
ible by both 3 and 7 i.e., 21 is 462. The last number
between 450 and 950 which is divisible by 21 is 945. Hence, the correct option is (A).
Hence, from 21 × 22 to 21 × 45, there are a total of 24 19. Both progressions are A.P.’s. The series of common
terms which are divisible by both 3 and 7. terms of two A.P.’s is also an A.P. Its common differ-
Solving using the concept of Progressions: ence is the L.C.M of the common differences of the two
progressions.
First term between 450 and 950 divisible by 21 (both 3
and 7); a = 462 (between 450 and 950). The last term First common term of the two progressions is 14.
divisible by 21 between 450 and 950 () = 945. The The Nth common term of the two progressions
common difference between successive terms which = 14 + (N – 1) L.C.M. (6, 4) – 14 + (N – 1) 12
are divisible by 3 and 7 is 21.
14 + (N – 1) 12 < the smaller of the last terms of the two
945 − 462 483 A.P.’s
Number of terms required = +1= +1
21 21 = 14 + (N – 1) 12 = 98
= 23 + 1 = 24.
⇒N=8
Hence, the correct option is (B).
Hence, the correct option is (C).
16. S = 2 + 4x + 6x2 + 8x3 + … (11)
20. Let S = 2 + 3x + 4x2 + 5x3 + … (15)
Multiplying by x both sides
Then, Sx = 2x + 3x2 + 4x3 + … (16)
Sx = 2x + 4x2 + 6x3 + … (12)
Subtracting (16) from (15),
Subtracting (12) from (11),
S(1 - x) = 2 + x + x2 + x3 + …
S(1 – x) = 2 + 2x + 2x2 + 2x3 + … x
2 S(1 - x) = 2 +
1− x
2
S = 1− x = 2 (1 − x ) + x 2− x
1 − x (1 − x ) 2 S(1 - x) = =
1− x 1− x
Hence, the correct option is (C).
17. If the first term of the G.P. is a and the common ratio 2− x
S=
(1 − x )
2
is r, we have
a( r 8 − 1) Hence, the correct option is (A).
= 510 (13)
r −1

Unit I_Chapter 12.indd 118 27/04/2017 14:17:35


Chapter 12  •  Progressions  |  1.119

21. The given sequence is 3, 8, 17, 30, …, 1227 = (–1 + 4) + (–9 + 16) + (–25 + 36) + … + (–361 + 400)
The sequence formed by the differences of two suc- = 3 + 7 + 11 + … + 39
cessive terms of the above sequence is 8 – 3, 17 – 8, This is in arithmetic progression.
30 – 17, … a = 3, nth term = 39
i.e, 5, 9, 13, …., which is an arithmetic progression Number of terms = 20/2 = 10 (as the given terms are
\ The nth term of the given sequence can be taken as grouped into pairs)
tn = an2 + bn + c 10
Sum of these 10 terms = (3 + 39) = 5 × 42 = 210
2
n = 1, t1 = a + b + c = 3 (17)
Hence, the correct option is (A).
n = 2, t2 = 4a + 2b + c = 8 (18)
n = 3, t3 = 9a + 3b + c = 17 (19) 1 1 1 1
23. + + +
Solving (17), (18), and (19) we have a = 2, b = –1 and 1+ 3 3+ 5 5+ 7 7+ 9
c=2 1
+… +
\ tn = 2n2 – n + 2 = 1227 119 + 121
2n2 – n – 1225 = 0 3 −1 5− 3
= + +
(2n + 49) (n – 25) = 0 ( 3 − 1)( 3 + 1) ( 5 − 3 )( 5 + 3 )
n = 25 as n > 0
7− 5 121 − 119
Sum of the terms is +…+
n ( 7 − 5 )( 7 + 5 ) ( 121 − 119 )( 121 + 119 )
∑ t n = S(2n2 – n + 2)
n =1 3 −1 5− 3 7− 5 121 − 119
= 2Sn2 – Sn + 2S1 = + + +… +
2 2 2 2
2n ( n + 1) ( 2n + 1) n ( n + 1)
= − + 2n 121 −1
6 2 = =5
2
⎡ 2 ( n + 1) ( 2n + 1) − 3 ( n + 1) + 12 ⎤ Hence, the correct option is (B).
= n⎢ ⎥
⎣ 6 ⎦ 24. log3x + log3 1 x + log3 1 x + … + log3 1 x = 432
n 3 5 23
= ⎡⎣ 4 n2 + 6 n + 2 − 3n − 3 + 12 ⎤⎦
6 log3 x + 3log3 x + log3x + … + 23log3 x = 432
n ⇒ 144 log3x = 432 ⇒ x = 27
= ⎡⎣ 4 n2 + 3n + 11⎤⎦
6 Hence, the correct option is (B).
25 25. Let the first term and the common difference of S1 be a1
When n = 25 sum of terms = ⎡ 4 × 252 + 3 ( 25 ) + 11⎤⎦
6 ⎣ and d1 respectively. Let the first term and the common
difference of S2 be a2 and d2 respectively.
25 [ 2500 + 75 + 11] h
Mean of 25 terms = = 431 [2a1 + ( n − 1)d1 ]
6 25 2 11n − 17
=
Hence, the correct option is(B). n 5n − 21
[2a2 + ( n − 1)d2
22. -12 + 22 - 32 + 42 - 52 + 62 - 192 + 202 2
= (-12 - 32 - 52 - … -192) + (22 + 42 + … + 202) n −1
a1 + d
= - (12 + 32 + 52 + … + 192) + (22 + 42 + … + 202) = 2 1 = 11n − 17
n −1 5n − 21
= -(12 + 22 + 32 + … + 202) + (22 + 42 + … + 202) + a2 + d2
2
(22 + 42 + … + 202) a + 15d1
The ratio of the 16 terms of S1 and S2 = 1
= 2(22 + 42 + … + 202) - (12 + 22 + 32 + … + 202) a2 + 15d2
= 2 [22 (12 + 22 + … + 102] - (12 + 22 + 32 + … + 202) The 16th term is the average of the first 31 terms. The
8(10) (11) ( 21) ( 20) ( 21) ( 41) ratio of 16 terms in equal ratio of the sum of the 31
= − times of the two series.
6 6
210 (88 − 82) 210 (6) \ The ratio of the 16th terms of S1 and S2
= = = 210.
6 6 11(31) − 17 341 − 17 324 162
= = = =
Alternate method:
5(31) − 21 155 − 21 134 67
–12 + 22 – 32 + 42 – 52 + 62 + … + 152 + 202 Hence, the correct option is (B).

Unit I_Chapter 12.indd 119 27/04/2017 14:17:38


1.120  |  Unit 1  •  Quantitative Aptitude

Practice Problems 2 Sol. The given series represents a geometric progression


3
Solutions for questions 1 to 25: whose first term is 1 and common ratio is .
4
1. Thirteen times the thirteenth term of an arithmetic pro- The sum to infinity of a geometric progression whose
gression is equal to seven times the seventh term of the a
arithmetic progression. What is the twentieth term? first term is a and whose common ratio is r =
(| r | < 1) 1− r
(A) –1 (B) –3 (C) 0 (D) 4
Sol. Let the first term and the common difference of the 1
The sum to infinity = =4
arithmetic progression be a and d respectively. 3
1−
13 + t13 = 7 × t7 4
⇒ 13 (a + 12d) = 7 (a + 6d) ⇒ a = –19d Hence, the correct option is (B).
T20 = a + 19 d = –19d + 19d = 0 6. If x, y, and z are three natural numbers in arithmetic
progression, then the xth term, the yth term, and the zth
Hence, the correct option is (C). term of any arithmetic progression A are in
2. Find the sum of the terms of the arithmetic progression (A) Arithmetic progression
whose first term, last term, and common difference are (B) Geometric progression
3, 101, and 7, respectively. (C)  Not necessarily in arithmetic progression or
(A) 750 (B) 720 (C) 780 (D) 810 ­geometric progression.
Sol. Let the number of terms be n. (D) Arithmetic as well as geometric progression
101 = 3 + 7 (n – 1) Sol. Let A be an arithmetic progression whose first term is a
15 = n and whose common difference is d.
15 xth term of A = a + (x – 1) d
Sum of the terms = [3 + 101] = 780
2 yth term of A = a + (y – 1) d
Hence, the correct option is (C). zth term of A = a + (z – 1) d
3. Find the sum of the terms of an arithmetic progression As x, y and z are in arithmetic progression, (x – 1) d,
whose first term, last term, and number of terms are –9, (y – 1) d and (z – 1) d are in arithmetic progression….
51, and 21, respectively. the xth term of A, the yth term of A and the zth term of A
(A) 420 (B) 441 are in arithmetic progression.
(C) 462 (D) 483
Hence, the correct option is (A).
n 7. If x, y, and z are three natural numbers in arithmetic
Sol. Sum to n terms = [ a +1 ] progression, then the xth term, the yth term, and the zth
2
term of any geometric progression G, are in
21
Sum of the terms = [–9 + 51] = 441 (A) Arithmetic progression
2 (B) Geometric progression
Hence, the correct option is (B). (C) Not necessarily in arithmetic progression or geo-
4. Find the first four terms of a geometric progression metric progression.
whose nth term is 4(–5)n. (D) Arithmetic as well as geometric progression
(A) –20, 100, –500, 2500 Sol. Let G be a geometric progression whose first term is a
(B) 20, –100, –500, 2500 and whose common ratio is r.
(C) –20, –100, –500, –2500 xth term of G = arx – 1
(D) 20, 100, 500, 2500
yth term of G = ary – 1
Sol. Putting n = 1, 2, 3, 4 we can get the terms
zth term of G = arz – 1
t1 = 4(-5)1 = –20 x+z
As x, y and z are in arithmetic progression, y =
t2 = 4(-5)2 = 100 x+z−2
2
Finding two terms is enough to get the answer from \ ary – 1 = ar 2

options. (ary – 1)2 = a2 rx – 1 + z – 1


Hence, the correct option is (A). = (arx – 1) (arz – 1)
3 9 27 \ The xth term, the yth term and zth term of G are in
5. Find the sum to infinity of 1, , , , ..
4 16 64 geometric progression.
(A) 2 (B) 4 (C) 4/3 (D) 2/3 Hence, the correct option is (B).

Unit I_Chapter 12.indd 120 27/04/2017 14:17:39


Chapter 12  •  Progressions  |  1.121

8. The sum of the squares of three terms in arithmetic pro- B( N )


for to be an integer, N must be in the form
gression is 365. The product of the first and the third A( N )
terms is 120. Find the sum of the squares of the second 3k + 1 where k is a whole number, i.e., 6p + 1 or 6p + 4
term and the common difference.
\ N – 1 or N + 2 would be divisible by 6.
(A) 145 (B) 170
(C) 122 (D) 197 Hence, the correct option is (C).
Sol. Let the second term and the common difference be a 11. Find the smallest of the three numbers in arithmetic
and d respectively. progression, if the product of the first and the third
numbers is 252 and the sum of the three numbers is 48.
First term = a – d (A) 10 (B) 12 (C) 14 (D) 16
Third term = a + d Sol. Let the three numbers in A.P. be a - d, a and a + d;
(a – d)2 + (a + d)2 + a2 = 365 a - d + a + a + d = 3a = 48
3a2 – 2d2 = 365 (20) 48
⇒a= = 16.
(a – d) (a + d) = 120 3
Given that 162 - d2 = 252
a2 – d2 = 120  (21)
256 - 252 = d2
solving (20) and (21),

4 = d2
a2 = 121 and d2 = 1
Thus, d = ± 4 = ± 2
a2 + d2 = 122
Hence, the smallest of the three numbers is 16 - 2 = 14,
Hence, the correct option is (C). Even if d = –2 is considered, the smallest number will
9. The sum of four numbers in an ascending arithmetic be 14 only.
progression. is 160 and the product of the extremes is Hence, the correct option is (C).
1564. Find the smallest of the numbers.
12. If the sum of the first 37 terms of an arithmetic progres-
(A) 28 (B) 34
sion. is 703, then find the sum of the first 10 terms of
(C) 42 (D) 43
the arithmetic progression, given the first term of the
Sol. Let the four terms be a - 3d, a - d, a + d and a + 3d arithmetic progression is 1.
a - 3d + a - d + (a + d) + a + 3d = 160 (A) 55 (B) 65
(C) 75 (D) 85
4a = 160 ⇒ a = 40;
37
Given (a - 3d) (a + 3d) = 1564; a2 - 9d2 = 1564; Sol. Sum upto the first 37 terms is [2a + (37 - 1) d]
2
9d2 = a2 - 1564 = 402 - 1564 37
36 = [2a + 36d] where a is the first term of A.P and d is
d2 = = 4; d = ±2. 2
9 the common difference.
Since the A.P. is ascending d = 2. = 37[a + 18d] = 703.
Smallest number = a - 3d = 40 - 3(2) = 34. 703
Hence, the correct option is (B). a + 18d = = 1 + 18d = 19
37
10. Let A(N) denote the sum of the first N natural num- ⇒d=1
bers and B(N) denote the sum of the squares of the first Sum of the first 10 terms of the A.P.
N natural numbers. If B(N) is a multiple of A(N), then 10
which of the following could be true? = [2 (1) + (10- 1) 1] = +5 [2 + 9] = 55.
2
(A) N is divisible by 6
Hence, the correct option is (A).
(B) N + 1 is divisible by 6
(C) N + 2 is divisible by 6 13. The first term of an arithmetic progression consisting
(D) N – 3 is divisible by 6 of 30 terms is 10 and the common difference is 5. Find
the ratio of the sum of the 30 terms of the arithmetic
N ( N +1)
Sol. A(N) = progression. to the sum of the last 20 terms of the AP.
2 (A) 99 : 13 (B) 96 : 17
N ( N + 1)( 2 N + 1) (C) 93 : 19 (D) 99 : 86
B(N) =
6 30
Sol. Sum of the first 30 terms of the A.P. = [2 (10)
B( N ) 2 N + 1 + 29 (5)] 2
=
A( N ) 3 = 15 [20 + 145] = 15 [165] = 2475.

Unit I_Chapter 12.indd 121 27/04/2017 14:17:40


1.122  |  Unit 1  •  Quantitative Aptitude

10 1 1
Sum of the first 10 terms of the A.P. = [2 (10) + 9(5)] Sol. 1 + + + is an infinite series in geometric
2 2 4
= 5[20 + 45] = 5 [65] = 325. Ratio of the sum of the first progression.
30 terms of the A.P. to the sum of the last 20 terms of a = 1, r = 1/2
the A.P. = (2475) : (2475 - 325) = 2475 : 2150 a 1
Sum to infinity = = =2
= 5 [495] : 5 [430] = 495 : 430 = 99 : 86 1− r 1 −1 / 2
Hence, the correct option is (D). Hence, required value is 72 = 49.
14. The product of three terms in geometric progression. is Hence, the correct option is (A).
1728 and the sum of the products of two of them taken
at a time is 1032. Find the smallest of the three terms. 17. An athlete runs a race, and after every hour, his speed
(A) 1 (B) 2 reduces to half the speed with which he travelled for the
(C) 3 (D) 6 previous hour. Find the time taken to cover the race, if
the person started the race with a speed of 16 km/hr and
a the length of the race was 31.5 km.
Sol. Let the three terms in GP be , a and ar
r (A) 4 hours (B) 6 hours
(a) (C) 8 hours (D) 5 hours
\ ( a ) ( ar ) = 1728; a3 = 1728
r Sol. In the first hour, the distance covered by the athlete =
⇒ a = 12 16(1) = 16 km. In the second hour, distance covered by
a a ⎛1 ⎞ 1
( a ) + a ( ar ) + ( ar ) = a2 ⎜ + r ⎟ + a 2 = 1032 the athlete = (16) (1) = 8 km.
r r ⎝ r ⎠ 2
Assuming that the person travelled for a total of t hours,
⎛1 ⎞ we have
∵ a = 144; 144 ⎜ + r ⎟ + 144 = 1032
2

⎝ r ⎠
⎛ ⎛ 8⎞ t

⎛1 ⎞ 1 888 111 37 16 ⎜1 − ⎜ ⎟ ⎟⎟
144 ⎜ + r ⎟ = 888 , + r = = = . ⎜ ⎝ 16 ⎠
⎝ ⎠
⎝r ⎠ r 144 18 6 = 31.5
8
1 37 1 1−
+ r = =6+ 16
r 6 6
1 ⎛ ⎛ 8 ⎞t ⎞
⇒ r = 6 or 16 ⎜1 − ⎜ ⎟ ⎟
6 ⎜ ⎝ 16 ⎠ ⎟
⎝ ⎠
So, 2, 12, 72 or 72, 12, 2 is the G.P. and 2 is the smallest  = = 31.5
1
number. 1−
2
Hence, the correct option is (B).
t
15. The first term of an arithmetic progression is 6 and the ⎛ 1 ⎞ 31 ⋅ 5
1- ⎜ ⎟ =
common difference is 4. The nth term of the series is ⎝2⎠ 32
250. Find the value of n.
(A) 60 (B) 62 t
⎛1⎞ 31 ⋅ 5 32 − 31 ⋅ 5 0 ⋅5 1
(C) 66 (D) 64 ⎜ 2 ⎟ = 1 − 32 = = =
Sol. If a is the first term of an arithmetic progression and the ⎝ ⎠ 32 32 64
common difference is d, the nth term of the progression t 6
is given by a + (n - 1)d. The nth term is given as 250, ⎛1⎞ ⎛1⎞
⇒⎜ ⎟ =⎜ ⎟
a as 6 and d as 4. ⎝2⎠ ⎝2⎠
Hence, 250 = 6 + (n - 1)4. ⇒ t = 6 hours.
(n - 1)4 = 244 Alternate method:
61 = n - 1. The sum required is 31.5. First term is 16 and r is 1/2.
Hence, n = 61 + 1 = 62. 1
Hence, writing down the terms upto the value , can
Hence, the correct option is (B). be a good method of solving. 2
1 1
1+ + +∞
1 6. 7 2 4 evaluates to The terms are 16, 8, 4, 2, 1, 1/2, 1/4 …
(A) 49 (B) 34 S6 = 31.5; hence t = 6 hours.
(C) 2401 (D) 343 Hence, the correct option is (B).

Unit I_Chapter 12.indd 122 27/04/2017 14:17:43


Chapter 12  •  Progressions  |  1.123

18. If Sn is the sum of the first n terms of the series 40 + 22. If the sum of three numbers in a geometric progression
38 + 36 + …, then find the maximum value of Sn. is 38 and their product is 1728, then find the smallest
(A) 450 (B) 420 number.
(C) 390 (D) 410 (A) 6 (B) 4 (C) 2 (D) 8
Sol. The given series 40, 38, 36, …, is cm A.P. with com- Sol. Let the first term and the common ratio be a and r
mon difference of –2. respectively.
The sum of the first 20 terms (S20) is 2 (20) (21)/2 = 420 Second term = ar and Third term = ar2
The 21st term is 0. a + ar + ar2 = 38 and (a)(ar)(ar2) = 1728
\ S21 = 420 (ar)3 = 123 ⇒ ar = 12
12
For n > 21 the terms would be negative Sn < 420. The first 3 terms are , 12, 12r
r
\ The maximum value of Sn = 420. 12 2 3
+ 12 + 12r = 38 ⇒ 6r2 – 13r + 6 = 0 ⇒ r = or
Hence, the correct option is (B). r 3 2
19. 13 + 23 + 33 + … + m3 = 3025. Find the value of m. 2
(A) 8 (B) 9 (C) 10 (D) 11 if r = , the numbers are 18, 12 and 8.
3
2
⎛ m( m + 1) ⎞ 3
ol. 1 + 2 + 3 + …+ m = ⎜
S 3 3 3 3
⎟⎠ = 55
2 If r = , the same numbers are obtained in the reverse
⎝ 2 2
order.
m( m +1) In either case, 8 is the smallest number
Hence, = 55; m2 + m = 110
2
Hence, the correct option is (D).
m2 + m - 110 = 0
23. The common ratio of geometric progression is a posi-
Hence, m = -11 or m = 10. The number of terms cannot tive number less than 1. The first term is 18. The dif-
be negative. Thus, m = 10. ference of the third and second terms is 4. Find the
Hence, the correct option is (C). common ratio.
20. Find the sum of the squares of the first 10 even numbers. 1 2
(A) (B)
(A) 840 (B) 1540 3 3
(C) 1260 (D) 1370
3 1 2
Sol. Sum of the squares of the first 10 even numbers (C) (D) or
4 3 3
= 22 + 42 + 62 + 82 + 102 + … + 202
Sol. Let the common ratio be r. 0 < r < 1,
= 22 (12 + 22 + 32 + 42 + 52 + … + 102)
\ r > r2. The first 3 terms are 18, 18r, 18r2
4(10) (11) ( 21)
= 2 1
6 18 r – 18r2 = 4 ⇒ r = or
3 3
n( n + 1) ( 2n + 1) Hence, the correct option is (D).
[Applying the formula for the sum of
6
24. Find the sum of the first 12 terms of Sn, where Sn =
the squares of first n natural numbers] 1 + 3 + 7 + 13 + 21 + 31 + 43 + … .
4 × 210 × 11 (A) 472 (B) 584
= = 140 × 11 = 1540.
(C) 591 (D) 572
6
Hence, the correct option is (B). Sol. Sn = 1 + 3 + 7 + 13 + 21 + 31 + 43 + … +tn
21. Find the sum of the terms of the series Sn = 1+ 3 + 7 + 13 + 21 + 31 + … + tn-1 + tn
(1) × (20), (2) × (19), (3) × (18), …(20) × (1). Subtracting, we get
(A) 1750 (B) 1640
0 = 1 + [2 + 4 + 6 + 8 + 10 + … upto n - 1 terms] - tn
(C) 1540 (D) 1430
⇒ tn = 1 + [2 + 4 + 6 + 8 + … up to n - 1 terms]
Sol. The terms of the series are in the form x (21 – x)
n −1
20
⇒ tn = 1 + [2(2) + (n – 2)2]
Required sum = ∑ x (21 – x)
x =1
2
( n −1)
( 21)( 20)( 21) 1 = 1 + [2 n] = 1 + n2 - n
= − (20)(21)(41) = 1540 2
2 6 N

Hence, the correct option is (C). \ SN = ∑t


n =1
n = Σ( n2 − n + 1) = Σ n2 − Σn + Σ 1

Unit I_Chapter 12.indd 123 27/04/2017 14:17:45


1.124  |  Unit 1  •  Quantitative Aptitude

N ( N + 1) ( 2 N + 1) N ( N + 1) 2
= − +N ⎛ side of S1 ⎞ 1
6 2 Sol. Side of S2 =⎜ ⎟ ( 2) = (side of S1)
⎝ 2 ⎠ 2
12(13)( 25) 12(13) It follows that side of Sn+1 = 1 (side of Sn)
\ S12 = − + 12
6 2 2
= 650 - 78 + 12 Where n is any natural number
= 584 ⎛ 32 32 ⎞
Sum of the perimeters = 4 ⎜ 32 + + + …⎟
Hence, the correct option is (B). ⎝ 2 2 ⎠
25. S1 is a square. By joining the midpoints of sides of S1 4(32)
another square S2 is formed. By joining the midpoints = = 128 2 ( 2 + 1) cm
1
of S2, another square S3 is formed, and so on. The side 1−
of S1 is 32 cm. Find the sum of the perimeters of all the 2
squares (in cm). Hence, the correct option is (B).

(A) 48 2 ( )
2 +1 (B) 128 2 ( 2 +1 )
(C) 72 2( 2 + 1) (D) 36 2 ( )
2 +1

Unit I_Chapter 12.indd 124 27/04/2017 14:17:46


Chapter 13 Permutations and Combinations
Hints/Solutions
Practice Problems 1 Men Women Number of
6 4 selections
1. The total number of ways that 12 blazers, 10 shirts and
5 ties can be worn is 12(10)(5) = 600 (by fundamental 3 2 6
C34C2
rule). 4 1 6
C44C1
Hence, the correct option is (D). 5 0 6
C54C0
2. The given word INSTITUTE contains a total of 9
occurrences - 3T ’s, 2I’s and the rest four are distinct. \total number of ways that the committee formed is
The total number of different words that can be formed, 6
C3. 4C2 + 6C4. 4C1 + 6C5. 4C0
9! = 20(6) + 15(4) + 6(1) = 120 + 60 + 6 = 186
is
3! 2 ! Hence, the correct option is (B).
Hence, the correct option is (D). 9. We treat 10 girls as 1 unit.
3. Since one player is in the team and one player is not in Then total number of students = 11
the team, we have to select 10 players from 13. This can
\ 11 students can be arranged in a row in 11! ways
be done in 13C10 or 13C3 or 286 ways.
Again the 10 girls can be arranged among themselves
Hence, the correct option is (C).
in 10! ways.
4. The number of ways of selecting 6 students from n stu-
\required number of ways = 10! 11!
dents is nC6. The number of ways of selecting 9 students
from n students is nC9. Hence, the correct option is (D).
Given nC6 = nC9 ⇒ n = 15 10. Since the number of boys is greater than the number
of girls, the first and the last place has to be occupied
The number of ways of selecting 4 students from 15
by boys. 6 boys can be arranged in a row in 6! ways.
15(14)(13)(12) There are five gaps in between them. The 5 girls can be
students is 15C4 = = 1365
4(3)( 2)(1) arranged in these gaps in 5! ways.
Hence, the correct option is (A). \Total number of arrangements = 6! 5!.
5. When two books are to be together, we assume those Hence, the correct option is (C).
two books as 1 unit. With this one unit, there are 9
11. The number of derangements of n objects is
books which can be arranged in 9! ways. But these two
books can be arranged internally in 2! ways ⎡1 1 1 1 n 1 ⎤
Dn = n! ⎢ − + − + … ( −1)
\ Required number of arrangements = 9! 2! ⎣ 2 ! 3! 4 ! 5! n ! ⎥⎦
Hence, the correct option is (A). Out of seven letters any two letters are placed into its
6. Number of ways of inviting at least one of n people is corresponding envelopes and remaining 5 letters, no
given by 2n – 1 Here n = 6 letter is placed into its corresponding envelope.
\ Required number of ways 26 – 1 i.e. 7C2 (D5)
Hence, the correct option is (B). ⎛1 1 1 1⎞
= 21 ( 5! ) ⎜ − + − ⎟
7. The word EQUATION contains 5 vowels and 3 ⎝ 2 ! 3! 4 ! 5! ⎠
consonants. = 21 × (44) = 924
Since the words begin with a consonant, the first place Hence, the correct option is (B).
can be filled in 3 ways and last place can be filled in 12. Number of ways of choosing at least one green dye is
5 ways. The remaining 6 places can be filled with the 23 - 1.
remaining 6 letters in 6! ways.
Number of ways of choosing at least one yellow dye is
\The total number of words that can be formed is 3(5) 22 - 1.
(6!) = 10800.
Number of ways of choosing a red dye is 24.
Hence, the correct option is (B).
8. The possible number of men and women and the corre- (
\ Required number of ways = 23 −1 22 −1 24 )( )( )
sponding number of ways in which the committee can = (7) (3) (16) = 48 (7) = 336
be selected are tabulated below. Hence, the correct option is (A).

Unit I_Chapter 13.indd 125 27/04/2017 14:17:53


1.126  |  Unit 1  •  Quantitative Aptitude

13. Prahaas can select the questions in the following Th H T U


combinations 3 (5) (7) (7) 245
  (i) 3, 3, 2    (ii)  3, 2, 3 4 (7) (7) (7) 343
(iii) 2, 3, 3 (iv)  4, 2, 2 5 (7) (7) (7) 343
  (v) 2, 4, 2 (vi)  2, 2, 4 7 (3) (7) (7) 147
The number of selections in each of the cases (i), (ii) -------
and (iii) is 6C3 × 6C3 × 6C2 while 1078
The number of selections in each of the cases (iv), (v), -------
(vi) is 6C4 × 6C2 × 6C2 These 1078 numbers include 3200, but not 7300.
Hence total selections are \ The number of numbers between 3200 and 7300 is
3 × 6C3 × 6C3 × 6C2 + 3 ×6C4 × 6C2 × 6C2 1077.
= 3 [6000 + 3375] = 28125 Hence, the correct option is (D).
Hence, the correct option is (C). 17. The six-digit number may start with 15 or 26 and also it
is an even number. We can have the following possible
14. The word has 7 letters I, I, N, N, K, L, G. The following cases.
are the possibilities while selecting 4 letters. Case 1:
Case (i): All 4 are distinct. 15–––6
Case (ii): Two are alike and two are distinct. If the last digit is 6, then in the remaining 3 places, one
Case (iii): Two are alike of one kind and two alike of place can be filled by 6 and the other two places can be
the other. filled in 9(9) ways.
Now the number of selections and arrangements in \ Hence, the number of trials = 9(9) (3) = 243
each case is given below. Case 2:
Combinations Permutations 15––––
Case (i) 5
C4 = 5 5 × 4! = 120 If last position is filled by one of the digits 0, 2, 4, 8,
4! then in the remaining 3 places, two of the place can be
Case (ii) 2
C1 × 4C2 = 12 12 × = 144 filled by 6 and third place can be filled by 9 ways.
2!
\ Hence, required number of trials = 4(9) (3) = 108
4! Case 3:
Case (iii) 2C2 = 1 1× =6
2! 2! 26___6
Hence total Combinations are 18. If the last position is filled by 6, then the remaining 3
Hence, the correct option is (D). positions can be filled in 9(9) (9) ways.
\ Hence, required number of trials = 9 × 9 × 9 = 729
Solutions for questions 15 to 25: Case 4:
15. The number of ways that the man can invite at least 0, 2, 4, 6, 8
n + 1 friends for a dinner is
2 6 6 ↓9 ↓9 ↓
(2n + 1)
Cn + 1 + … + 2n +1C2n +1 = 4096 (1) If the last position is filled with 0, 2, 4, 8, then in
\ 2n + 1
C0 + 2n + 1
C1 + … + 2n +1
Cn = 4096  (2) remaining 3 positions one position has to be filled by 6
(1) + (2) ⇒ and the other two positions can be filled in 9(9) ways.
2n + 1C0 + 2n + 1C1 + … + 2n + 1C2n–1 = 8192 \ Hence, the required number of trials = 9(9) (3) (4)
= 972
⇒ 22n + 1 = 8192 (∵ nC0 + nC1 + … + nCn = 2n)
\ At the most, Raju has to make 243 + 108 + 729 + 972
⇒ 22n + 1 = 213
or 2052 trials to succeed.
⇒ 2n + 1 = 13 Hence, the correct option is (B).
\ the number of friends = 13 18. A 4 × 3 matrix has 12 elements. Each element can be 0
Hence, the correct option is (D). or 1 or 2.
16. The digits in the thousands place and the possible The total number of matrices 0, 1, 2 as the elements is
number of digits in the hundreds, tens and units places 312
(enclosed in brackets) are tabulated below. Hence, the correct option is (C).

Unit I_Chapter 13.indd 126 27/04/2017 14:17:54


Chapter 13  •  Permutations and Combinations  |  1.127

19. One postcard can be dropped into 8 letter boxes, in 8 Note: A group of (p + q + r) items can be divided into
ways, 4 postcards can be dropped in 84 ways. groups of p, q, r items in
Hence, the correct option is (C). ( p + q + r) !

20. 12 pens can be distributed among 3 children and each p!q!r !
one gets 4 pens.
23. In an 8 × 8 chess board there are 8 rows and 8 columns.
\ Hence, required number. of ways = 12C4 × 8C4 × 4C4 In every row and column there are four white squares
12 ! 8! 12 ! each. Number of ways of selecting two white squares
= × =
8! 4 ! 4 ! 4 ! ( 4 !)3 which are in same row or column = 8 × 4C2 + 8 × 4C2 =
8 [6 + 6] = 96
Hence, the correct option is (D). Hence, the correct option is (C).
21. The initial digits and the number of numbers are tabu- 24. \ Required number of ways = (total 4 digited tel-
lated below. ephone numbers) - (The number of 4 digited numbers
Initial Digits Number of numbers without repetition)
3 24 = (9) (103) - 9 (9P3)
4 24
6 24 = 9 (1000 - (9) (8) (7))
------------------------------------------------------ = 9 (1000 -504)
8 24 = 9 (496) = 4464
------------------------------------------------------ Hence, the correct option is (B).
The 96th number is 89643 25. We know that, if p things are alike of one kind, q things
The 95th number is 89634 are alike of second kind and r things are alike of third
Hence, the correct option is (C). kind, then one or more things can be selected in (p + 1)
22. We need to divide a group of 50 into groups of 25, 15 (q + 1) (r + 1) – 1 ways.
50 ! \ Required number of ways = (4 + 1) (3 + 1) (2 + 1) - 1
and 10 and this can be done in .
25!15!10 ! = (5) (4) (3) - 1 = 59
Hence, the correct option is (B). Hence, the correct option is (B).

Practice Problems 2
1. A five lettered word is formed using some of the 6 6 6 6
­letters {a, b, h, i, p, r, s}. How many of them will be
\ Required number of ways = 6 4
palindromes?
(A) 125 (B) 225 (C) 343 (D) 729 Hence, the correct option is (D).
3. Find the number of even numbers formed using all the
Sol. A palindrome is a word which when read from left to
digits 1, 2, 3, 4, 5 when each digit occurs only once in
right or right to left, remains the same.
each number.
In a palindrome, only in the first half the letters are dif- (A) 5
P4 (B) 4! 2 (C) 5! (D) 4!
ferent. The same letters that appear in the first half are
Sol. A number is even if its units place is divisible by 2
repeated in the second. i.e. in a five letter palindrome
word, first three letters will be different. \ units place can be filled by either 2 or 4 i.e. in 2 ways.
The remaining 4 places can be filled by the remaining 4
First place can be filled in 7 ways, digits in 4! ways
Similarly second and 3rd places can be filled in 7 ways Total number of even numbers that can be formed
\ required number of palindrome words formed is = 2 (4!) = 48
= 7 × 7 × 7 = 343 Hence, the correct option is (B).
Hence, the correct option is (C). 4. In how many ways can ten students be seated around a
2. Find the number of four-digit numbers that can be circular table so that three students always sit together?
formed using the digits 1, 2, 3, 4, 5, 6 when each digit (A) 7! (B) 7! 3! (C) 2.7! (D) 3 (7!)
can occur any number of times in each number. Sol. Assume that the three students have to sit together as
(A) 46 (B) 6
P4 (C) 6
P6 (D) 64 one unit. Now there are 8 units (7 students + 1 unit
of three students) and they can be arranged at a cir-
Sol. We have to fill four blanks. Each blank can be filled by cular table in 7! ways. Again the three students can be
any of the six digits. arranged among themselves in 3! ways.

Unit I_Chapter 13.indd 127 27/04/2017 14:17:54


1.128  |  Unit 1  •  Quantitative Aptitude

\The total number of ways = 7! 3!. 10. In how many ways can the letters of the word
Hence, the correct option is (B). ‘SUBJECT’ be placed in the squares of the figure given
5. In how many ways can four letters be posted into 5 post below so that no row remains empty?
boxes? (A) 5 × 6! (B) 10 × 6!
(A) 350 (B) 125 (C) 625 (D) 1024 (C) 11 × 5! (D) 13 × 8!
Sol. Each letter can be posted in 5 ways. Sol. In all 10 squares in which 7 letters to be arranged and it
can be done is 10P7.
Four letters can be posted in 5(5)(5)(5)(5) or 625 ways.
Hence, the correct option is (C). Top row vacant, this can be done in 8P7 = 8! ways.
6. In how many ways can 7 boys and 6 girls be arranged in Middle row vacant, this can be done in 8P7 = 8! ways.
a row so that no two girls sit together? \ Required arrangements = 10P7 – 8! × 2
(A) 13! (B) 8P6 × 7! 10 ! ⎛ 90 ⎞
= - 8! 2! = 8! ⎜ − 2 ⎟ = 13 × 8!
(C) 6! P7 8
(D) 12! 6 ⎝ 6 ⎠
Sol. First arrange the 7 boys. Hence, the correct option is (D).
× B1 × B2 × B3 × B4 × B5 × B6 × B7 × 11. In how many ways can the letters of the word EUROPE
Now there are 8 gaps between boys, marked by ×. be arranged so that no two vowels are together?
(A) 12 (B) 24
7 boys can be arranged in 7 places in 7! ways
(C) 360 (D) Not possible
The 6 girls can be arranged in these gaps in 8P6 ways
Sol. In the given word, there are 4 vowels and 2 consonants.
\Required number of ways = 8P6 × 7! \ It is not possible to arrange the letters as required.
Hence, the correct option is (B). Hence, the correct option is (D).
7. A man has 7 friends. In how many ways can he invite at 12. A four-digit number is formed using the digits 0, 2, 4,
least one of his friends for a dinner? 6, 8 without repeating any one of them. What is the sum
(A) 63 (B) 120 (C) 127 (D) 256 of all such possible numbers?
Sol. The number of ways (or combinations) of selecting (A) 519960 (B) 402096
atleast one of n different things is (C) 133320 (D) 4321302
n
C1 + nC2 + … + nCn = 2n – 1 Sol. The first digit (thousands) can be selected in 4 ways.
\ The number of ways that the man can invite at least The other 3 places can be filled in 4P3 ways.
one of his friends for dinner We can form a total of 4 4P3 four-digited numbers by
= 27 – 1 = 128 – 1 = 127 using all the even digits. We have to add all these num-
bers. Let us look at the contribution of each of the digit.
Hence, the correct option is (C).
3 (3!) numbers contain 2 in the units place
8. The number of squares that can be formed on a 8 × 8
chessboard is 3 (3!) numbers contain 2 in the tens place
(A) 204 (B) 220 (C) 240 (D) 210 3 (3!) numbers contain 2 in the hundreds place
Sol. Number of squares (S) that can be formed in an n × n Similarly, we can work out the value contributed by the
chess board other digits. The digits and the total contribution of the
digits is tabulated below.
n
n( n + 1) ( 2n + 1)
= ∑ i 2 = Digit Contribution
1 6
2 24 (2000) + 18 (222)
8(9) (17)
As n = 8, S = = 204 4 24 (4000) + 18 (444)
6
6 24 (6000) + 18 (666)
Hence, the correct option is (A).
8 24 (8000) + 18 (888)
9. The number of positive integral solutions to the equa-
tion x + y + z = 20 is The total sum is 24 (20000) + 18 (20) (111)
(A) 131 (B) 110 (C) 55 (D) 171 = 480000 + 39960 = 519960
Sol. We know that, Hence, the correct option is (A).
\ The number of positive integral solutions of 13. How many four-digit odd numbers can be formed such
x1 x2 + … + xr = n is n – 1Cr – 1 that every 3 in the number is followed by a 6?
(A) 108 (B) 2592 (C) 2696 (D) 2700
19 ×18
\ Required answer is 19C2 = = 171 Sol. The possible digits in the different places and the corre-
2
sponding number of numbers are tabulated below. The
Hence, the correct option is (D). units digit or the tens digit cannot be 3.

Unit I_Chapter 13.indd 128 27/04/2017 14:17:55


Chapter 13  •  Permutations and Combinations  |  1.129

Possible Digits No. of Number \ The total number of arrangements is 13.


A. 3 6 __ odd 9 (4) = 36 Hence, the correct option is (C).
B. 3 __ 6 odd 10 (4) = 40 16. How many 4-digit numbers can be formed such that
C. __ 3 6 odd 9 (4) = 36 the digit in the hundreds place is greater than that in the
D __ __ __ odd 8 (9) (9) (4) = 2592 tens place?
(A) 9000 (B) 10000
-------
(C) 4500 (D) 4050
2704
Sol. ↓ _ _ ↓
9 10

-------
As there is no restriction on the units place, this place
The numbers 3361, 3365, 3367 and 3369 have been
can be filled by any of the 10 digits. The, thousands
counted in B as well as C.
place can be filled by any of the 9 digits. (all except 0)
The numbers 3661, 3665, 3667, 3669 has been counted
in A as well as B. To fill the other two places, we have to select two dis-
tinct digits. This can be done in 10C2 ways.
\ The required number of numbers is 2696.
Required number of ways = 10C2 (10) (9) = 4050
Hence, the correct option is (C).
14. How many times does the digit 5 appear in the numbers Hence, the correct option is (D).
from 9 to 1000? 17. In how many ways can 5 boys and 3 girls sit around a
(A) 300 (B) 257 (C) 256 (D) 299 table in such a way that no two girls sit together?
Sol. Case 1: (A) 480 (B) 960 (C) 320 (D) 1440
The number of 2-digit and 3-digit numbers having only Sol. The 5 boys can be seated around a table in 4! ways. In
one 5 is 3 (9) (9) = 243 between them, there are 5 places.
Case 2: The 3 girls can be placed in the 5 places in 5P3 ways.
5 5 ↑9 B1
The number of 2-digit and 3-digit numbers having B5
exactly two 5’s is 3 (9) = 27
Case 3: B2
B4
555
The number of numbers having exactly three 5s is 1. B3
\ The total number of times 5 occurs, in all possible
\ Required number of ways = 4! 5P3 = 24 × 60
natural numbers less than 1000 is 243 + 2 (27) + 3(1)
= 300 = 1440
The number of times 5 occurs in between 9 to 1000 is Hence, the correct option is (D).
300 - 1 = 299 18. Find the maximum number of ways in which the letters
Hence, the correct option is (D). of the word MATHEMATICS can be arranged so that
15. There are 5 bowls numbered 1 to 5, 5 green balls all M’s are together and all T ’s are together.
and 6 black balls. Each bowl is to be filled by either 11!
(A) 11! (B)
a green or black ball and no two adjacent bowls can 2! 2! 2!
be filled by green balls. If the same colour balls are 9! 9!
(C) (D)
indistinguishable, then the number of different possible 2! 2! 2! 2!
arrangements is
Sol. In the given word, there are 2 M’s, 2 T ’s, 2 A’s and 5
(A) 8 (B) 7 (C) 13 (D) 256
single letters.
Sol. The number of black and green balls and the number of
Taking the 2 M’s as one unit and 2 T ’s as one unit, with
ways they can be arranged in the 5 bowls, so that no two
9!
adjacent bowls have green balls are tabulated below. remaining 7 letters can be arranged ways. (There are
2 A’s) 2!
b g No. of 9!
(6) (5) arrangements Position of g balls \ Required number. of ways =
2!
5 0 1 Hence, the correct option is (D).
4 1 5 1, 2, 3, 4, 5 19. Consider the word INSTITUTE.
3 2 6 1, 3 or 1,4 or 1,5 or 2,4 In how many ways can 5 letters be selected from the
or 2, 5 or 3, 5
word?
2 3 1 1, 3, 5 (A) 41 (B) 30 (C) 36 (D) 40

Unit I_Chapter 13.indd 129 27/04/2017 14:17:56


1.130  |  Unit 1  •  Quantitative Aptitude

Sol. The word is INSTITUTE 22. In a certain question paper, a candidate is required to
answer 5 out of 8 questions, which are divided into two
Letter N S U E I T
parts containing 4 questions each. He is permitted to
Number of times repeated 1 1 1 1 2 3
attempt not more than 3 from any group. The number
The distribution of the 5 letters, combination and of ways in which he can answer the paper is
­permutations are tabulated below. (A) 24 (B) 96 (C) 48 (D) 84
Distribution Combinations Permutations
Sol.

1, 1, 1, 1, 1 6
C5 = 6 6 (5!) = 720 Section (1) Section (2) Required
(4 Qns) 4 (Qns) combinations
1, 1, 1, 2 2 ( C3) = 20
5
⎛ 5! ⎞
20 ⎜ ⎟ = 1200 2 3 4
C2 4C3
⎝ 2! ⎠ 3 2 4
C3 4C2
1, 1, 3 5
C2 = 10
⎛ 5! ⎞ Required number of ways = 2 (4C3) (4C2)
10 ⎜ ⎟ = 200
⎝ 3! ⎠ = 2 (4) (6) = 48
1, 2, 2 4
C1 = 4
⎛ 5! ⎞ Hence, the correct option is (C).
4⎜ ⎟ = 120
⎝ 2! 2! ⎠ 23. The sides PQ, QR, and RS of ∆PQR have 4, 5, and 6
2, 3 1
points (not the end points), respectively, on them. The
5! number of triangles that can be constructed using these
= 10
2 !3! points as vertices is
41 2250
(A) 455 (B) 34 (C) 425 (D) 421
Sol. We know that,
5 letters can be selected in 41 ways.
Hence, the correct option is (A). The number of triangles formed with ‘n’ non collinear
points is nC3.
20. The letters of the word AGAIN are permuted in all pos-
sible ways and are arranged in dictionary order. What is \ Here, number of triangles = 15C3 - 4C3 - 5C3 - 6C3
the 28th word?   = 455 - 4 - 10 - 20
(A) GAIAN (B) GAINA
(C) GANIA (D) NGAIA  = 455 - 34 = 421
Sol. Arranging the letters of the word ‘AGAIN’ in diction- Hence, the correct option is (D).
ary order is A, A, G, I, N. 24. There are 8 different books and 2 identical copies of
The letters and the number of words are tabulated each in a library. The number of ways in which one or
below. more books can be selected is
Initial Letters Number. of words (A) 28 (B) 38 – 1 (C) 28 – 1 (D) 38
A 24 Sol. For each book, 0 or 1 or 2 copies can be selected.
GAA 2 Hence, the required number of ways = 38 - 1
GAIAN 1 (At least 1 book has to be selected)
GAINA 1 Hence, the correct option is (B).
\ The 28 word GAINA
th
25. The number of sequences in which 7 players can throw
Hence, the correct option is (B). a ball, so that the youngest player may not be the last is
21. In how many ways can 12 distinct pens be divided (A) 4000 (B) 2160
equally in to 3 parcels? (C) 4320 (D) 5300
12 ! 12 ! Sol. Not younger player
(A) 4 (B)
( 4 !) ( 4 !)3 _ _ _ _ _ _ ↑
12 ! 12 ! The last ball can be thrown by any of the remaining
(C) (D)
3! 4 ! ( 4 !)3 3! 6 players. The first 6 players can throw the ball in 6P6
Sol. 12 pens can be distributed into 3 parcels in ways.

12 ! \ The required number of ways = 6 (6!) = 4320


ways
( 4 !) 3!
3
Hence, the correct option is (C).

Hence, the correct option is (D).

Unit I_Chapter 13.indd 130 27/04/2017 14:17:57


Chapter 14  Data Interpretation
Hints/Solutions
Practice Problems 1 6. The number of projects undertaken in 1998
Solutions for question 1: = 350 + 400 = 750.
1. Percentage increase for the student Hence, the correct option is (C).
7. The average number of projects undertaken in Mexico
⎛ 1295 − 997 ⎞
P= ⎜ ⎟ × 100 = 29.8% and Texas
⎝ 997 ⎠ 250 + 200
Percentage increase for the student in 1995 = = 225.
2
⎛ 876 − 664 ⎞ 300 + 250
Q= ⎜ ⎟ × 100 = 31.9% in 1996 = = 275.
⎝ 664 ⎠ 2
Percentage increase for the student 350 + 400
in 1998 = = 375.
⎛ 732 − 585 ⎞ 2
R = ⎜ ⎟ × 100 = 25.12%
⎝ 585 ⎠ 400 + 350
in 1999 = = 375
Percentage increase for the student 2
350 + 500
⎛ 689 − 480 ⎞ in 2000 = = 425.
S= ⎜ ⎟ × 100 = 43.5% 2
⎝ 480 ⎠
Hence, the correct option is (A).
\ Clearly only student ‘S’ has percentage increase
more than 35%. Solutions for questions 8 to 10:
Hence, the correct option is (A). 8. The least percentage of colleges offering courses in
medicine is for New Jersey and the least percentage of
Solutions for questions 2 to 4: colleges offering Biochemistry is for Michigan.
2. On March 1, 1867 – 37th state joined the Union and on \ The least number of colleges offering Biochemistry
Feb 14, 1912 – 48th state joined the Union. Including in the four states
them a total of 48 - 37 +1 i.e., 12 states have joined.
⎛ 15 80 30 ⎞
Hence, the correct option is (B). = Min ⎜ . .2000, .68.2000 ⎟
⎝ 100 100 100 ⎠
124
3. Required percentage = ×100 = 23%. 15 80
535 = . 2000 = 240
100 100
Hence, the correct option is (D).
\ Each state has atleast 240 colleges offering
4. Number of representatives is not less than six for
Biochemistry i.e., more than 230 colleges offering
Washington, Texas, Virginia, Minnesota, Illinois,
Biochemistry.
Arizona, Indiana and Georgia. Of these, only Texas has a
population of more than 15 million. Hence, the remain- Hence, the correct option is (A).
ing seven states will satisfy both the given conditions. 9. We do not know whether each college offering
Hence, the correct option is (D). Biophysics in any of the four states offers Biochemistry
or not. \ we cannot answer the question.
Solutions for questions 5 to 7:
Hence, the correct option is (D).
5. The number of projects in Mexico and Texas
10. Total number of colleges offering Biostatistics in all the
Year Mexico Texas four states =
1995 250 > 200
⎛ 56 35 84 15 86 20 70 30 ⎞
1996 300 > 250 ⎜ 100 . 100 + 100 . 100 + 100 . 100 + 100 . 100 ⎟ . 2000
⎝ ⎠
1997 250 < 300
7 3 1 3
1998 350 < 400 = 56. . 20 + 84. . 20 + 86. .20 + 70. .20
20 20 5 10
1999 400 > 350
= 392 + 252 + 344 + 420 = 1408.
2000 350 < 500
Hence, the correct option is (B).
2001 400 < 550
During the three years, the number of projects under- Solutions for questions 11 and 12:
taken in Mexico are greater. According to the given data, the distribution of the
Hence, the correct option is (A). workers is as follows:

Unit I_Chapter 14.indd 131 27/04/2017 14:18:03


1.132  |  Unit 1  •  Quantitative Aptitude

Factory Total Men Women Boys


Solutions for questions 16 and 17:
16. The average sales over the given period (in 000’s of
J 4800 2400 1800 600
rupees)
K 6750 2700 2430 1620
3750 + 3500 + 4250 + 4500 + 4400
L 5250 1260 2940 1050 = = 4080
5
M 12000 4200 3000 4800
The average production over the given period (in 000’s
N 8500 3230 2550 2720
of rupees)
P 2700 1215 1080 405
4500 + 5100 + 4750 + 5000 + 5750
= = 5020
11. Required percentage 5
Therefore the average production over the given period
1080 − 600
= × 100 = 80%. was more than the average sales by
600
5020 − 4080
Hence, the correct option is (C). = × 100 = 23% .
4080
12. The required ratio = 4200 : 2940 Hence, the correct option is (B).
= 10 : 7. 17. Percentage change in sales from 2004 to 2005
Hence, the correct option is (B).
3750 − 3500
Solutions for questions 13 to 15: = × 100 = 6.66
3750
25 + 25 + 40 90 Percentage change in sales from 2005 to 2006
13. The required ratio = = = 3 : 4.
30 + 45 + 45 120 4250 − 3500
= × 100 = 21.43%
Hence, the correct option is (C). 3500
Percentage change in sales from 2006 to 2007
30 − 25 45 − 30 50 − 45 55 − 50 45 − 30 4500 − 4250
14. Of , , and , is = × 100 = 5.88
25 30 45 50 30 4250
the highest which is for the year 2002. Percentage change in sales from 2007 to 2008
Hence, the correct option is (C).
4500 − 4400
1 5. The required percentage increase = × 100 = 2.22%
4500
(50 + 55) − ( 45 + 50) × 100 Therefore the percentage change was the highest in the
=  10.5%. year 2006.
( 45 + 50)
Hence, the correct option is (D). Hence, the correct option is (B)..

Practice Problems 2 required to raise the temperature of 2.2 pounds (1 kg) of


water by 1.8°F (1°C).
Solutions for questions 1 to 3:
Note 2: 1 Joule = 0.2390 calories and
Type of energy Btu content
1 Btu = 1.06 kilojoules
1 barrel of crude oil (42 gallons) 5,838,000
1 gallon of gasoline 124,000 1. The Btu content in 42 gallons of heating oil is equal to
the Btu content in which of the following?
1 gallon of heating oil 139,000
(A) 60 gallons of propane
1 barrel of residual fuel oil 6,287,000
(B) 42 gallons of crude oil
1 cubic foot of natural gas 1,026 (C) 48 gallons of gasoline
1 gallon of propane 91,000 (D) 57 cubic feet of natural gas
1 short ton of coal 20,681,000 Sol. Btu content in 42 gallons of heating oil = 42 × 139,000
1 kilowatt hour of electricity 3,412 = 5838000 i.e., the same in 1 barrel of Crude oil or 42
gallons of crude oil.
Note 1: British Thermal Unit (Btu) represents the amount Hence, the correct option is (B).
of energy required to raise the temperature of 1 pound of
2. The Btu content in 60 gallons of propane is less than
water by 1°F. Also 1 calorie represents the amount of energy
that in one short ton of coal by

Unit I_Chapter 14.indd 132 27/04/2017 14:18:05


Chapter 14  •  Data Interpretation  |  1.133

(A) 15,221,000 units (B) 16,290,000 units 3. 1 Btu is equal to _____ calories
(C) 17,820,000 units (D) 18,630,000 units (A) 228.72 (B) 244.68
ol. Btu content in 60 gallons of propane = 60 × 91,000
S (C) 253.34 (D) 262.44
= 54,60,000 Btu content in one short ton of coal Sol. 1 Btu = 1.06 Kilo joules i.e., 1060 joules
= 20,681,000 But 1 joule = 0.2390 calories
\The required difference = 15,221,000 units. \ 1 Btu = 1060 × 0.2390 calories i.e., 253.34 calories.
Hence, the correct option is (A). Hence, the correct option is (C).

Directions for questions 4 to 6:  These questions are based on the graph and pie chart given below.
Distribution of US doctors across specializations in a certain year

General Physicians

Dentists

Oncologists

Psychiatrists

Gynecologists

Cardiologists

Orthopaedicians

Others

0 5 10 15 20 25 30 35 40 45
(Ten Thousands)

Distribution of professionals in USA that year

CPAs
Doctors
15%
20%

Engineers
Others
35%
30%

4. Gynaecologists constitute what percent of the total 35 ×180 63


number of doctors? = = .
40 × 20 8
(A) 11% (B) 25% (C) 20% (D) 15%
Hence, the correct option is (A).
20 1
Sol. Gynaecologists = × 100 = × 100 = 11%. 6. If the population of USA was 360 millions that year,
180 9 then how many doctors are available per 1000 people?
Hence, the correct option is (A). (A) 20 (B) 10 (C) 5 (D) 4
5. What is the ratio of the number of engineers to the Sol. (Number of doctors ÷ Total population) × 1000
number of general physicians?
180 × 10 4
63 47 33 23 = × 1000
(A) (B) (C) (D) 360 × 106
8 8 7 5
Sol. The number of engineers as a fraction of total number = 5 per thousand.
of general physicians Hence, the correct option is (C).

Unit I_Chapter 14.indd 133 27/04/2017 14:18:06


1.134  |  Unit 1  •  Quantitative Aptitude

Directions for questions 7 to 9:  The following bar graph shows the quantity (in million watts) of different types of power
generated in five states in a certain country in the year 2007 and 1997.
350

300

250

200

150

100

50

0
A B C D E A B C D E
2007 1997
Hydel Therm al Solar Nuclear Wind Bio

Solutions for questions 7 to 9: Wind as well as Bio is more in 2007 when compared to
7. For state A, we observe that wind power was not pro- that in 1997.
duced in 2007 and was produced in 1997. Similarly for Hence, the correct option is (A).
C and E also, atleast one type of power was produced 9. Of the total power generated in 2007 of the types wind
in 1997, but not in 2007. and solar, only 85% and 90%, respectively, were use-
Hence, the correct option is (B). ful for consumption. By how many million watts is
Sol. For how many of the given states can we say that at least the consumable power generated by state D in 2007
one particular type of power was not produced in 2007 using solar energy less than that generated using wind
while the same type of power was produced in 1997? energy?
(A) 4 (B) 3 (A) 20.25 (B) 19.50
(C) 2 (D) 1 (C) 18.75 (D) 17.50
8. For how many types is the total production in 2007 more Sol. Consumable power from solar energy in state D in
than that in 1997 for all the given states combined? 2007 = 90% of 50 = 45 MW. That from wind energy =
(A) 5 (B) 4 85% of 75 = 63.75 MW.
(C) 3 (D) 2 \ Required difference = 18.75 million watts.
Sol. Observing the given stacked bar, we can say that the Hence, the correct option is (C).
production of power from Hydel, Thermal, Nuclear,

Directions for questions 10 to 12:  These questions are based on the following data that show the number of representatives
of different political parties in the Parliament of UK, which has three wings—The Northern Ireland Assembly, The Scottish
Parliament and the National Assembly of Wales.
Northern National
ireland Scottish assembly of
Political assembly Political parliament Political wales
parties Males Females parties Males Females parties Males Females
DUP 36 12 Labour 48 8 Labour 12 4
UUP 10 3 Democrats 16 4 SDLP 15 2
SDLP 24 2 Conservative 24 6 Conservative 6 5
Others 18 3 Republicans 14 9 Others 9 7

Unit I_Chapter 14.indd 134 27/04/2017 14:18:08


Chapter 14  •  Data Interpretation  |  1.135

Solutions for questions 10 to 12: Classification of passengers in each of of the flights


10. Only among parties with at least 25 members in any First class
wing, the number of young parliamentarians is more 10%
than eight. What is the maximum number of parliamen-
tarians in the two wings—Northern Ireland Assembly
and Scottish Parliament who are not young? Business class
(A) 201 (B) 144 (C) 165 (D) 176 15%
Sol. In Northern Ireland Assembly, the maximum number
Economy class
of parliamentarians who are young = (48 - 9) + 13
75%
+ (26 - 9) + 21 = 90. The same for Scottish parliament
= (56 - 9) + (20) + (30 - 9) + 23 = 111.
\ 90 + 111 i.e. 201 is the required answers.
Hence, the correct option is (A). Solutions for questions 13 to 15:
11. If there are six Catholic parliamentarians for each party 13. How many airports handle more than 1500 flights per
in the Scottish Parliament, then how many Scottish day?
parliamentarians are not members of the Conservative (A) 5 (B) 6
party nor are Catholics? (C) 7 (D) 8
(A) 27 (B) 42 (C) 60 (D) 81 Sol. There are 7 Airports handling more than 1500 flights
Sol. The number of parliamentarians who are neither per day.
Catholic nor members of the Conservative party
Hence, the correct option is (C).
= (48 + 8 - 6) + (16 + 4 - 6) + (14 + 9 - 6) = 81.
Hence, the correct option is (D). 14. Which of the following airports carries the maximum
number of passengers per flight per day?
12. The number of parliamentarians who are graduates in
(A) Chicago (B) JFK
the National Assembly of Wales is 45. How many par-
liamentarians in the National Assembly of Wales are (C) Detroit (D) Atlanta
either females or graduates? Sol. Number of passengers per flight per day carried by the
(A) 27 given airports are as follows:
(B) 30 720 6
(C) 32 (A) Chicago = = = 0.24
3000 25
(D) Cannot be determined
Sol. The number of parliamentarians in the National 700 1
(B) JFK = = = 0.25
Assembly of Wales who are either females or graduates 2800 4
cannot be determined because there is no information
392 7
on the number of females who are not graduates. (C) Detroit = = = 0.28
1400 25
Hence, the correct option is (D).Directions for ques-
tions 13 to 15:  These questions are based on the table 560 1
and the pie chart given below. (D) Atlanta = = = 0.2
2800 5
  Details of Airports in USA As thousand is common in all the fractions, eliminating
Flights per day Number of that Detroit has the maximum number of passengers
passengers per flight.
Name of the per day (in Hence, the correct option is (C).
Airport Arrivals Departures hundreds)
15. How many passengers (in hundreds) travel in the
Atlanta 1420 1380 560
Economy class at JFK Airport?
JFK 1380 1420 700
(A) 310 (B) 490
Florida 1300 1500 360 (C) 525 (D) 595
Chicago 1440 1560 720
Sol. Number of passengers travelling in Economy class at
National 1200 1300 360 JFK Airport (in hundreds) = 75% of 700
Los Angeles 920 880 468
3
Dallas 800 700 405 = × 700 = 525.
4
Detroit 690 710 392
Hence, the correct option is (C).
Seattle 1100 1000 441

Unit I_Chapter 14.indd 135 27/04/2017 14:18:09


1.136  |  Unit 1  •  Quantitative Aptitude

Directions for questions 16 to 18:  These questions are based on the graph and the table given below.
Distribution of students who passed in different
subjects in the year 2000 in a school

Biological Science

Mathematics

History

Literature

Physics

Chemistry

0 5 10 15 20 25 30
(in hundreds)

  Percentage change in the number of students who passed over


the previous year in the school

2000 2001
Biological Science 10% 20%
Mathematics 15% 20%
History –25% –10%
Literature 30% –20%
Physics 20% 20%
Chemistry 15% 25%

Note: The negative sign indicates a decrease.

16. What is the ratio of the number of students who passed The increase in the percentage of students who passed
in History in 1999 to that in 2001? from 1999 to 2001
(A) 9 : 4 (B) 8 : 5 = 143.75 – 100K × 100 = 43.75%.
(C) 40 : 27 (D) 6 : 5
Hence, the correct option is (D).
Sol. Let the number of students who passed in History in
18. In which of the following pairs of subjects did an equal
1999 be 100 K.
number of students pass in the year 2001?
\ The number of students who passed in 2000 will be (A) Mathematics and Literature
75 K. (B) Literature and Chemistry
Also, the number of students in 2001 will be (C) Mathematics and Physics
90 135 (D) History and Physics
75 K × i.e., K Sol. The number of students of Mathematics who passed in
100 2
2001 = 120% of 15 = 18.
\ The required ratio will be 40 : 27.
Hence, the correct option is (C). The number of students of Literature who passed in
2001= 80% of 20 = 16.
17. By what percent did the number of students who passed
The number of students who passed in Chemistry
in Chemistry increase from 1999 to 2001?
2001=125%of 20 = 25.
(A) 35% (B) 37.5%
(C) 40% (D) 43.75% The number of students who passed in Physics 2001
= 120% of 15 = 18.
Sol. Let the number of students who passed in Chemistry in
1999 be 100 K. The number of students of History who passed in 2001
= 90% of 12 = 10.8.
The number of students who passed in 2000 = 115 K
125 The number of students of Biological sciences who
The number of students in 2001 = × 115 K passed in 2001 = 120% of 25 = 30.
100
The number of Maths students and the number of
125
= × 115 K = 143.75 K ­students of Physics is equal in 2001.
100
Hence, the correct option is (C).

Unit I_Chapter 14.indd 136 27/04/2017 14:18:10


Chapter 14  •  Data Interpretation  |  1.137

Directions for questions 19 to 21:  These questions are based on the graphs given below.

Sales of Coke and Pepsi


50

Billion gallons 40

30

20

10

0
North America Asia Europe Africa South America
and Others
Coke Pepsi

Populations in various continents


4
3.5
3
2.5
Billions

2
1.5
1
0.5
0
North America Asia Europe Africa South America
and Others

Solutions for questions 19 to 21: 4


(C) × 102 gallons/person
19. By what percent is the sales of Pepsi in Asia lower than 3
that in North America? 1000
(D) gallons/person
(A) 40% (B) 33% 7
(C) 25% (D) 20% Sol. Per capita consumption of coke in Europe
Sol. Sales or Pepsi in Asia and in North America are 25 × 10 9
15 ­billion gallons and 20 billion gallons respectively. = gallons/person
0.75 × 10 9
20 − 15
Therefore the required ratio = × 100 = 25%. 100
20 = gallons/person.
3
Hence, the correct option is (C).
Hence, the correct option is (B).
Total consumption 21. If the average (arithmetic mean) of the cost of Coke in
2 0. Per capita consumption =
Total population all the continents is US$11, per gallon, then what is the
total sales revenue on selling Coke in all the continents?
Using the above definition what is the per capita con-
(A) $1835 billion (B) $1675 billion
sumption of Coke in Europe?
(C) $1535 billion (D) $1100 billion
4 Sol. The total volume of Coke in all the continents
(A) × 10 gallons/person
3
= 100 billion gallons
100 Revenue = 100 × 11 i.e. $1100 billions.
(B) gallons/person
3 Hence, the correct option is (D).

Unit I_Chapter 14.indd 137 27/04/2017 14:18:12


1.138  |  Unit 1  •  Quantitative Aptitude

Directions for questions 22 to 24:  These questions are based on the graph, pie chart, and the table given below.
Interest rates in USA across 1995 to 1999.
7
6.5
6
5.5
5
Percentage Rate

4.5
4
3.5
3
2.5
2
1.5
1
0.5
0
1995 1996 1997 1998 1999

Distribution of loans availed of by various sectors of the


industry for each of the years from 1995 to 1999

Others Services
30° 60°

Power Agriculture
90° 45°

Manufacturing
135°

Total loan amount availed of by the industry


(In Billions of US$)
1995 36
1996 48
1997 72
1998 108
1999 144

22. In the years 1998 and 1999 combined, the total interest Interest paid by manufacturing sector in 1999
paid by exactly one sector as a percentage of the total 135
interest paid by all the sectors of the industry cannot be = 144 × × 5.5%
360
which of the following?
Total interest paid by all sectors of the industry in 1998
1 2 and 1999
(A) 8 % (B) 16 %
3 3 = 5.75% of 108 + 5.5% of 144
(C) 25% (D) 40%
\ Interest paid by manufacturing sector forms
Sol. Interest paid by manufacturing sector in 1998 135 135 135
135 108 × × 5.75 + 144 × × 5.5 100%
= 108 × × 5.75% 360 360 360
360

Unit I_Chapter 14.indd 138 27/04/2017 14:18:13


Chapter 14  •  Data Interpretation  |  1.139

108 × 5.75 + 144 × 5.5. 100% of the total interest paid 1 ⎛ 31.50 ⎞
by all the sectors of the industry = 37.5% = = $0.07875 billion = $78.75 million.
4 ⎜⎝ 100 ⎟⎠
From the above, we can see that interest paid by any
θ Hence, the correct option is (D).
sector forms .100% of the total interest paid by 24. The total interest paid in 1995 and 1996 was more than
360
all the sectors of the industry where q = central angle $390 million for
representing loan the amount which was availed by that (A) at least three sectors (B) exactly one sector
sector. (C) exactly two sectors (D) exactly three sectors
Maximum q was for the manufacturing sector. Sol. The total interest amount paid by ‘others’ in 1995 and
\ Maximum percentage also was for the manufactur- 1996
ing sector. 5.25 ⎛ 30 ⎞ 6 ⎛ 30 ⎞
= (36) + ( 48)
\ The required percentage cannot be 40%. 100 ⎜⎝ 360 ⎟⎠ 100 ⎜⎝ 360 ⎟⎠
Hence, the correct option is (D). 5.25 ⎛ 1 ⎞ 6 ⎛1 ⎞ 15.75 + 24
= ⎜ .36 ⎟ + ⎜ .48 ⎟ =
23. ENRON Inc. has availed 25% of the loan pertaining to 100 ⎝ 12 ⎠ 100 ⎝ 12 ⎠ 100
one of the sectors in 1997. What is the minimum possible = $0.3975 billion = $397.5 million.
amount of interest that they could have paid in 1997 in The total interest for the service sector, the power ­sector,
millions of dollars? [Consider others to be a single sector.] the agriculture sector, the agricultural sector and the
(A) 60.75 (B) 66.75 (C) 72.75 (D) 78.75 manufacturing sector will be more than $397.5 ­million
Sol. Interest paid by ENRON Inc and hence more than $390 million. (\ These sectors
5.25 ⎛ 25 ⎛ ⎛ 30° 60° 135° 90° ⎞ ⎞ ⎞ have a greater central angle than ‘­ others’). Others
= ⎜ 72 ⎜ or or or ⎟
360° 360° 360° 360° ⎠ ⎟⎠ ⎟⎠
⎜ ⎝ consist of one or more sectors. We do not know the
100 ⎝ 100 ⎝
breakup of the total interest amount paid by ‘­others’.
\ Minimum possible interest that ENRON Inc could We can only say that at least 4 sectors paid more than
have paid $390 ­million as interest.
5.25 ⎛ 25 ⎛ 30° ⎞ ⎞ 5.25 ⎛ 1 ⎞ Hence, the correct option is (A).
= ⎜ 72 ⎟= .6
100 ⎝ 100 ⎝ 360° ⎟⎠ ⎠ 100 ⎜⎝ 4 ⎟⎠

Directions for questions 25 to 27:  Answer the questions on the basis of the information given below.
In the bubble graph below, the horizontal axis shows the weight of the product in pounds and the vertical axis shows the
price in dollars. The relative size of the circles indicates how many of the products were sold. The largest circle indicates
4000 products, and there is a decrease of 10% in the number of products for an incremental decrease in the size of the circles.

P
30000

Q
R
S
25000
T U
Price in dollars

20000

15000 V

10000

2.1 2.2 2.3 2.4 2.5 2.6 2.7 2.8


Weight in pound

Unit I_Chapter 14.indd 139 27/04/2017 14:18:15


1.140  |  Unit 1  •  Quantitative Aptitude

25. A shop sells one unit each of all products having 50 min 60 min 20 min
a weight of 2.2 to 2.5 pound. The sales generated in F
A B
­dollars is ______
(A) 42500 (B) 70000
(C) 47500 (D) 72500
Sol. R, S and T corresponds to products whose weights lie in
C D E
the range 2.2 to 2.5 pounds
45 min 40 min 30 min
\ $24000 + $24000 + $22000 = $70000.
Hence, the correct option is (B). 28. Find the minimum time in which all the activities can
be completed.
26. The number of the heaviest product sold was ______
(A) 180 minutes (B) 175 minutes
that of any product shown and was the _______
(C) 195 minutes (D) 225 minutes
(A) greater than; most expensive
Sol. The minimum time that is required to complete all the
(B) greater than; least expensive
tasks is 50 + 60 + 40 + 30 i.e., 180 minutes.
(C) lesser than; most expensive
Hence, the correct option is (A).
(D) lesser than; most expensive
29. If the work is to be completed in the least possible time,
Sol. The heaviest product sold corresponds to circle V. The
then what is the maximum possible time gap between
size of bubble V is the largest in the graph
starting the activities D and F?
\ It was the least expensive ($12500). (A) 20 minutes (B) 40 minutes
Hence, the correct option is (B). (C) 50 minutes (D) 80 minutes
27. The total numbers of products of type U and T sold Sol. The maximum possible time gap between starting of
were ______. activities D and F is 50 minutes. Since F can be started
(A) 5400 (B) 6000 even after 50 minutes of starting of D.
(C) 6840 (D) 8000 Hence, the correct option is (C).
Sol. Product V = 4000 numbers 30. Now, two new activities—Y, taking 10 minutes, and Z,
90 taking 30 minutes—are included. If Y and Z are to be
Product U = × 400 = 3600 [10% decrease for a cor- completed before A and D, respectively, then what is the
100
responding decrease in size of all bubble] least possible time in which the entire work is completed?
(A) 205 minutes (B) 210 minutes
90 (C) 180 minutes (D) 190 minutes
Product T = × 3600 = 3240
100 Sol. If Y and Z are added then the minimum time required
\ Total number of products U and T = 3600 + 3240 increased by only 10 minutes, since Z can be completed
= 6840. before D begins (along with A and C). Therefore, 180 +
10 = 190 minutes.
Hence, the correct option is (C).
Hence, the correct option is (D).
Directions for questions 28 to 30: These questions are
based on the information given further. Directions for questions 31 and 32:  Answer the questions
Six activities—A, B, C, D, E and F—need to be per- on the basis of the information given below.
formed. Time required for each activity is given in the table, The table below gives the sector wise sales of XYZ Ltd.,
along with the list of activities that need to be completed which is a group of companies, for the financial year 2000–
before that activity is started. 2001. Study the table and answer the following questions.

Time to complete Activities to be Sales Percentage


Activity (in min.) performed before Sl. No Sector (`Million) share

A 50 – 1. Electronics (Es) 52,920 11.2

B 60 A, C 2. Financial Services (FS) 79,380 16.8


C 45 – 3. Consumer Goods (CG) 41,580 8.8
D 40 B, C 4. Information Systems (IS) 122,850 26
E 30 D 5. Pharmaceuticals (Ph) 40,635 8.6
F 20 B 6. Construction (Con) 100,170 21.2
7. Fertilizers (Fz) 34,965 7.4
Assume that two or more activities can be performed
simultaneously. Total 472,500 100.0

Unit I_Chapter 14.indd 140 27/04/2017 14:18:15


Chapter 14  •  Data Interpretation  |  1.141

Note: is 10. What is the average profit percentage across the


(A) Sales turnover at US $10.5 billion (`47,250 crore) rep- remaining sectors?
resents a year-on-year growth of 28 percent in dollar (A) 9.25 (B) 9.45
terms and 30.2 percent in rupee terms. (C) 9.65 (D) 9.85
(B) Profit percentage = Profit as a percentage of sales. 10
31. The group has decided to focus its attention on the min- Sol. Average profit across sectors = × 100 = 10
100
imum number of sectors contributing together at least
10
75% of the sales of the group. Identify the correct set Profit from electronics = × 11.2 = 1.12.
of sectors. 100
(A) IS, FS, Con, and Fz 15
Profit from consumer goods = × 8.8 = 1.32
(B) IS, FS, Con, and Es 100
(C) IS, Con, CG, Fz, and Ph \ Profit from other sectors = 7.56
(D) IS, FS, Es, and Fz Sales from other sectors = 80.
Sol. IS + FS + Con + Es = 26 + 16 × 8 + 21 × 2 + 11 × 2 = 75.2. 7.56
\ % Profit from other sectors = ×100 = 9.45 .
Hence, the correct option is (B). 80
32. The average profit percentage across the sectors is 10, Hence, the correct option is (B).
that for consumer goods is 15, and that for electronics

Directions for questions 33 to 35:  These questions are based on the following information.

The graph given below shows the savings, income from rent and
the expenditure of a family for each of the six years.
(in rupees thousands)
1400
1300
1200
1100
1000
900
800
700
600
500
400
300
200
100
0
1999 2000 2001 2002 2003 2004

Savings Income from rent Expenditure

Savings = Income from business + Income from rent – Expenditure

33. What is the approximate percentage growth in the aver- Annual savings in the last 3 years
age annual savings of the last three years compared to 950 + 1150 + 1350
that of the first three years? = = 1152
4
(A) 6% (B) 31% (C) 15% (D) 21%
Sol. Average annual savings in first 3 years 200 4
% growth = × 100 = × 100 ≅ 21%.
950 19
900 + 950 + 1000
= = 950 Hence, the correct option is (D).
3

Unit I_Chapter 14.indd 141 27/04/2017 14:18:16


1.142  |  Unit 1  •  Quantitative Aptitude

34. If the income from business is plotted on the graph, 35. Which of the following is the difference between the
then which of the following is true about the graph? average expenditure per annum and the average income
(A)  Increase followed by decrease in the alternate from rent per annum?
years (A) `50000 (B) `51660
(B)  Decrease followed by increase in the alternate (C) `41660 (D) `45000
years 5300
(C) Linear curve Sol. Avg. expenditure per annum = × 1000
6
(D) None of these
5000
Sol. The income from business for various years is Avg. income from rent per annum = × 1000
6
1999: 900 + 750 - 750 = 900 30
Required difference = × 1000 = `50000.
2000: 950 + 800 - 750 = 1000 6
2001: 1000 + 750 - 800 = 950 Hence, the correct option is (A).
2002: 950 + 900 – 700 = 1150
2003: 1150 + 1100 - 900 = 1350
2004: 1350 + 1000 - 1100 = 1250.
Hence, the correct option is (D).

Unit I_Chapter 14.indd 142 27/04/2017 14:18:17


Hints/Solutions | 1.143

Previous Years’ Questions y = 84.

1. The most consistent batsman is the batsman for whom the The smallest odd number = x = 75.
standard deviation of the scorers is the least. Therefore, K is The second largest even number = y + 4 = 88.
the most consistent batsman. Sum = 163.
Hence, The correct option is (A). Hence, The correct answer is 163.
2. Let the speed of the train be S. Let the length of the platform 7. We can straight away reject the items for which the revenue
be L. figure is less than or equal to the export figure. We have to
consider items 1, 3, 5 and 6 only. As all the percentage figures
280 + L 280 have to be multiplied by the same factors, to get the actual
= 60 and = 20
S S revenue and actual exports, we can ignore these factors. Of
280 + L = 60 S and S = 14 12 23 20 19 20
, , and the greatest is .
11 19 12 16 12
L = 60(14) – 280 = 560.
∴ Item 5 has generated the maximum revenue per kg.
Alternate Method:
Hence, The correct option is (D).
The train crosses its own length in passing a man standing on
the platform. 8. Let the rate at which water is pumped in, be f.
280 m/ sec Half the tank has to be filled in 10 minutes.
∴ Speed of the train = = 14 m/sec.
20 1
∴ In 60 seconds, the train will cover 840 m. Of this, 280 m is 1
∴ Effective rate of filling = 2 =
the length of the train. 10 20
∴ The remaining 560 m is the length of the platform. 12 1 1 1
f − = i.e., f − =
Hence, The correct answer is 560. 30 20 60 20
1
3. Trade deficit = (Exports) 1  1 
5 f =  = 4   = 4 (draining rate).
15    60 
1
Imports – Exports = (Exports) Hence, The correct option is (A).
5
9. According to the given chart,
6
Imports = (Exports) Hawk, Butterfly, Drongo, Bulbul are birds.
5
This relation is satisfied in 2006. ∴ Butterflies are birds
(i) is a correct conclusion
Hence, The correct option is (D). (iii) is also a correct conclusion.
4. The faces of one coin are H and H. ‘Tiger’ occupies a greater proportion of the ‘pie’ chart when
The faces of the other coin are H and T. compared with ‘Red ant’.
A coin is chosen randomly and tossed. It comes up H. ∴ There are more tigers than red ants in the forest.
The other face can be H, H or T. (ii) is correct.
1 Elephants are present in the largest number, they need not be
Probability (The other face is T) =
3 the largest mammals.
Hence, The correct option is (B). No information is given to support (iv).
Therefore, (iv) is not necessarily correct;
5. For an unbiased die thrown a very large number of times, the
(i), (ii), (iii) are correct.
frequency of occurrence of each of the numbers must be the
same. Hence, The correct option is (D).
As the given frequencies of occurrence are not the same, the 10. Let the stream velocity be V km/hr.
die must be biased. Upstream travel time = 3 (downstream travel time)
Hence, The correct option is (B).
d  d  ⇒ V = 4.
6. Let the eight consecutive odd numbers be = 3 
8 −V  8 +V 
x, x + 2, x + 4, x + 6, x + 8, x + 10 , x + 12, x + 14
Hence, The correct answer is 4.
x + (x + 2) + (x + 4) + (x + 6) + (x + 8) + (x + 10) + (x + 12) +
(x + 14) = 656 11. Expenditure on labour = 4,50,000
⇒ 8x = 600 ⇒ x = 75.
Total expenditure = 4, 50, 00 
100 
 = 3 lakhs
Let the four consecutive even numbers be y, y + 2, y + 4, y + 6  15 
y + (y + 2) + (y + 4) + (y + 6) = 87(4) Profit = 10 lakhs
⇒ 4y = 336 Total selling price = 40 lakhs

Unit I_Previous year Question.indd 143 27/04/2017 14:21:42


1.144 | Quantitative Aptitude

40 lakhs have each of the five digits in any of its places from 2nd to 5th.
Selling price of each purifier = = 20,000. 120
200 Sum of all the possible five digit numbers = (1(104 +
5
Hence, The correct answer is 20,000. 103 +102 + 10 + 1) + 3(104 + 103 + 102 + 10 + 1) + 5(104 + 103
12. Probability that the current batch is accepted = + 102 + 10 + 1) + 7(104 + 103 + 102 + 10 + 1) + 9(104 + 103 + 102
probability of none of the four bulbs tested being defective = + 10 + 1))
4
 5  = (24) (1 + 3 + 5 + 7 + 9) (11111) = 6666600.
1 − 100  = 0.8145
  Hence, The correct option is (B).
 5 
4
5  18. Sum
∵
1 −Probability
 of any bulb being non-defective is 1 − .
 100  100  2 −1 3− 2
= +
Hence, The correct answer is 0.8145. 2+ 1 3+ 2
n y
m
4−3 81 − 80
13. New value of f = ( 2 x )   = 2n–m xn ym = 2n–mP. + + 
2 4+ 3 81 + 81
Hence, The correct option is (A).
14. Let the 12 consecutive odd numbers be = 2 − 1 + 3 − 2 + 4 − 3 +
a, a + 2, a + 4, a + 6, a + 8, a + 10, a + 12, a + 14, a + 16, a +  81 − 80 = 81 − 1 = 8
18, a + 20, a + 22. a2 − b2
   Q 
=a − b
Sum of the first 5 numbers = 5 a + 20  a+b 
5 a + 20 = 425 ⇒ a = 81. Hence, The correct option is (B).
Sum of the last 5 numbers = a + 22 + a + 20 + a + 18 + a + 16 19. Probability (number selected being not divisible by 71) =
+ a + 14 = 103 + 101 + 99 + 97 + 95 = 495. 13
1 – probability (number selected being divisible by 7) = 1 –
Hence, The correct answer is 495. 90
15. Let the industrial consumption of power in 2000 – 2001 be P. (∵ of the 90, 2 – digit integers, there are 13 integers divisible
77
The industrial consumption of power in 2010–2011 = 2P. by 7. These are 14, 21, 28, ……..98) =
90
Let the rate be r % Hence, The correct option is (D).
n d
 r  20. Let the length of the journey be d km. The tourist covers km
2P = P 1 +  2
 100  d
at 60 km/hr,  d −  = km at 30 km/hr and the rest i.e.,
1 d
10 2 2 4
 r  d
2 = 1 + km at 10 km/hr.
 4
 100 
d
r 1 Average speed =
1+
= 210 ≈ 1.072  (1) d d d
100 + +
2 4 4
⇒ r = 7.2. 60 30 10
Hence, The correct option is (B). d
=
 1 1 1
16. Let the total cost in 2012 be x. d + +
 120 120 40 
Raw material expenses in 2012 = 0.2x.
1
Other expenses in 2012 = 0.8x. = = 24 km/hr i.e., 24 km/hr.
5
Raw material expenses in 2013 = (0.2x) (1.3).
120
= 0.26x.
Hence, The correct option is (C).
Other expenses in 2013 = (0.8x) (1.2) = 0.96x.
21. Erection cost = (labour wages per day) (number of working
Total cost in 2013 = 1.22x. hours).
Percentage increase in total cost Let the current labour wages per day and the current number
1.22 x − x of working hours be L and N respectively.
= × 100% = 22%.
x Current erection cost = LN. This is also = 13200.
Hence, The correct answer is 22%. LN = 13200.
Note:  Labour cost in 2012 is redundant data.  1  1
The new erection cost will be L 1 +  N 1 − 
17. Number of five digit numbers which can be formed using the  5   24 
digits 1, 3, 5, 7, 9 without repeating any of them = 5! = 120.  6   23 
= LN     = ` 15180.
Any five digit number formed is equally likely to begin with  5   24 
each of the five digits 1, 3, 5, 7, 9. It is also equally likely to Hence, The correct option is (B).

Unit I_Previous year Question.indd 144 27/04/2017 14:21:44


Hints/Solutions | 1.145

22. Let the number of units sold be q and the given selling price 25. We know that AM (x1 + k, x2 + k, x3 + k, . . . . xn + k) = AM (x1,
of each unit is ` 50. x2, x3, . . . xn) + k.
Selling price of q units = 50q AM (x1k, x2k, x3k, . . . xnk) = k Am (x1, x2, . . . xn)
Cost price of q units = 5q2 SD (x1 + c, x2 + c, x3 + c, . . . xn + c) = SD (x1, x2, x3, . . xn) = c
Profit function = 50q – 5q2 SD (x1, x2, x3, . . . xn).
∴ P = 50q - 5q2 ∴ P and R statements are true.
dp Hence, The correct option is (C).
= 50 - 10q.
dq 1 1
dp 26. Let LogkP = (LogkQ) = (LogkR) = c
For maximum of P, =0 2 3
dq
50 - 10q = 0 ⇒ P = kc, Q = k2c, R = k3c
∴q=5 Going from the choices, we can observed that only Q2 = PR is
d2 p true.
2 = - 10 < 0 Hence, The correct option is (B).
dq
∴ At q = 5, P is maximum. 27. There is P litres of pure liquid. If in each operation, x liters of
the liquid is replaced with x liters of water, then after n opera-
∴ The number of units produced is 5.
tion, the quantity of pure liquid present in P litres of the liquid
Hence, The correct option is (A). n
P−x
23. y = 2x - 0.1x2 is   × P.
 P 
dy Here, P = 10, x = 1 and n = 3.
= 2 - 0.2x
dx 3
dy  10 − 1 
for maximum value of y, =0 ∴ The quantity of spirit left =   × 10 = 7.29 litres.
dx  10 
2 - 0.2x = 0 Hence, The correct option is (D).
2
x= = 10
0.2 28. Given that, the danger to human beings (D) varies proportion-
d y 2 ately with toxicity (T), potency (P) and growth (G).
2 = - 0.2
dx Here, D varies directly as G because the bigger the circle, the
d2y greater the danger.
∴ At x = 10, <0
dx 2 D varies directly as P because the greater the potency, greater
At x = 10; y is maximum is the possibility that it will survive the human immunity
∴ y = 2 × 10 - 0.1 (10)2 system.
Maximum height  = 10 m. D varies inversely as T because, toxicity is defined as mil-
ligrams of microbe required to destroy half of the body mass.
Hence, The correct option is (B).
GP d2 p
24. Let A be the event that the shock absorber is supplied by X and ∴Da ⇒Da (G α d 2 where d is the diameter)
T T
B be the event that the shock the absorber is supplied by Y.
R be the event that the absorber in reliable. 50 2 ( 0.4 )
D(P) = k = 1.25k
60 3 40 2 800
Given P(A) = = ; P(B) = =
100 5 100 5 40 2 ( 0.5)
96 24 72 18 D(Q) = k = 1.33k
P(R/A) = = ; P(R/B) = = 600
100 25 100 25
30 2 ( 0.4 )
We find P(B/R). D(R) = k = 1.20 k
300
∴ By Baye’s theorem
20 2 ( 0.8)
P ( B) P ( R B) D(S ) = k = 1.60 k
P(B/R) = 200
P ( B ) P ( R B ) + P ( A) P ( R A ) ∴ S is the most dangerous microbe.
2 18 Hence, The correct option is (D).
× 36
5 25 36
= = = 100
2 18 3 24 36 + 72 108 29. V + F = 4q + = (say) T
× + × q
5 25 5 25
1 For T to be minimum,
= = 0.334. dT 100
3 = 0 ⇒ 4 - 2 = 0
Hence, The correct option is (B). dq q

Unit I_Previous year Question.indd 145 27/04/2017 14:21:47


1.146 | Quantitative Aptitude

⇒q=5 Combined rate of work of 2 skilled, 6 semi-skilled and 5


unskilled workers = 2(3) + 6  + 5 (1) = 20 units/day.
d T 2002 3
2 = 3 2
dq q 300
∴Time taken by them to complete the work = days
d 2T 20
For I = 5, > 0 i.e. T is minimum for q = 5. = 15 days.
dq 2
Hence, The correct option is (A). Hence, The correct option is (D).
30. Let the number of pending orders, the number of orders 32. If there is no restriction on the number of times each digit
received each day and the number of orders that each truck is used, we can form (3) (3) (3) numbers between 3000 and
can clear per day be p, e and n respectively. 4000 as well as between 4000 and 5000.
Given that ∴A total of 54 numbers could have been formed as there are
p + 4e = (7x) 4 = 28x (1) only 2 twos and 3 threes.
∴We cannot form 3222 and 3333. Also, we cannot form 4222.
p + 10e = (3x)10 = 30x (2)
A total of 51 numbers can be formed.
x x
(2) – (1) gives 6e = 2x ⇒ e = and from e = and (1), we Hence, The correct option is (B).
80x 3 3
get,  p 33. Let the base considered for the additions be b.
3
137 + 276 = 435
Total number of orders that will be accumulated at the end of
80x  x  85 x (137)b + (276)b = (435)b
the 5th day = +  5 = b2 + 3b + 7 + 2b2 + 7b + 6 = 4b2 + 3b + 5
3 3
  3
b2 - 7b - 8 = 0
∴ The number of trucks required to clear all the orders accu-
(b - 8) (b + 1) = 0
85 x 17
mulated till the end of the 5th day in 5 days = = (< 6) b = 8 (∵b must be positive).
3 3
i.e. a minimum of 6 trucks are required. 5x 7 3 1
Hence, The correct option is (C). + 6 7 2
31. Let the rate of work of each unskilled worker be 1 unit/day. 1 6 2 3

Ten unskilled workers can build the wall in 30 days.
We perform the addition on base 8.
∴Work = 1 (10) (30) = 300 units.
1 + 2 = 3 = 38
∴Rates of work of each skilled worker and each semi-skilled
300 300 3 + 7 = 10 = 128
worker are units/day and units/day, respectively
5 × 20 8 × 25 1 + 7 + 6 = 14 = 168
3
i.e. 3 units/day and units/day, respectively.
Hence, The correct option is (C).
2

Unit I_Previous year Question.indd 146 27/04/2017 14:21:48


Test
Hints/Solutions
1. Let the amounts with P, Q, R and S be p, q, r and s, respectively. 5. Let the weight of the diamond be W. Let the weights of the
p + q + r + s = 220 pieces be 3x and 4x. Initial value of the diamond equals kW 3,
where k is a constant.
p = q + 30
q W = 7x
s = i.e. q = 25. ∴ p = 25 + 30
2 ⇒ Initial value of the diamond (in `)
r = s + 10 = k(7x)3 = 343 kx3
2s + 30 + 2s + s + 10 + s = 220 Total value of the pieces (in `) = k(3x)3 + k(4x)3
6s = 180 = kx3 (33 + 43) = 91kx3
s = 30. Loss (in `) = 343 kx3 – 91 kx3 = 252 kx3 = 504000
Hence, The correct option is (C). ∴ kx3 = 2000
2. Let the number of girls and the number of benches be g and ∴ Initial value = 343kx3 = 343 × 2000
b, respectively. If 5 girls sit on each bench, a total of 5b girls
= ` 686000.
can sit.
Hence, The correct option is (B).
∴ g – 3 = 5b i.e. g = 5b + 3
6. Number of observations = 25
If there is one bench less, then on b – 1 benches 6(b – 1) girls
can sit. Average = 120
∴ g = 6 (b – 1) Correction = – (25)
25
g = 5b + 3 = 6 (b − 1) Corrected average = 120 −
25
b = 9. = 120 – 1 = 119.
Hence, The correct option is (D). Hence, The correct option is (B).
3. Let the number of questions correctly answered, wrongly 7. The average of 13 observations = 50
answered and unanswered by the candidate be C, W and U, ∴ x1 + x2 + x3 …… x13 = 650
respectively Average of the first seven observations
C + W + U = 60 (1) x1 + x2 …. x7 = 45 × 7 = 315 (2)
Candidate’s marks = C – W – U = 20 (2) Average of the last seven observations
(1) + (2) ⇒ 2C = 80 i.e. C = 40. x7 + x8 + ….. x13 = 7 × 52 = 364 (3)
Hence, The correct option is (D). By adding (2) and (3),
x7 + (x1 + x2 …. x13) = 679
a +b −c a −b + c c +b − a
4. = = − (1) From (1),
x y z
x7 + 650 = 679 ⇒ x7 = 29.
a c e a+c c +e a+e
If = = then, = = Hence, The correct option is (C).
b d f b +d d +f b +f
8. The concentration of milk after 3 replacements
a +b −c + a −b + c a −b + c + c +b − a
∴ (1) ⇒ =  100 − 10 
2

x +y y +z = = 0.729
 100 
a+b −c + c +b −a ∴ Concentration of water = 0.271
=
x +z
Required percentage = 27.1%.
2a 2c 2b Hence, The correct option is (D).
∴ = =
x +y y +z x +z
9. Total volume of the solution = 729 ml
Let each of these be k. 7
Quantity of acid = × 729 = 7 × 81 = 567 ml
Then 2a = k (x + y), 2c = k (y + z) and 2b = k (x + z) 9
x (b – a) + y (a – c) + z (c – b) Quantity of water = 162 ml
Let us assume that n ml of water is to be added to make the
xk yk zk ratio of acid to water as 7 : 3.
(z − y) + (x − z) + (y − x)
2 2 2 162 + x 3
= ⇒ 1620 + 10x = 2187 + 3x
k 729 + x 10
= {xz − xy + xy − yz + yz − xz}
2 7x = 2187 – 1620 = 567
k x = 81
= {0} = 0.
2 Therefore, 81 litres of water needs to be mixed.
Hence, The correct option is (A). Hence, The correct option is (D).

Part B_Unit 1_Test.indd 147 27/04/2017 14:16:23


1.148 | General Aptitude

10. Let the price of acid per gallon be ` 100. 1 1 1


⇒ = +
Let the quantity of water mixed per gallon of acid be x c 120 a
gallons. Substituting this in eg (3),
Cost price of (x +1) gallon = ` 100
1  1 1 1 2 5
100 + + = or =
Cost price of 1 gallon solution = a  120 a  20 a 120
x +1
or a = 48.
Selling price of 1 gallon solution = ` 100.
Hence, The correct option is (C).
100
∴ × 1.2 = 100 15. Let the time in which each man and each woman can com-
x +1
plete the job be m days and w days, respectively.
120 1
=x+1 Part of the job which can be completed by a man in a day = .
100 m

1.2 = x + 1 1
Part of it which can be completed by a woman in a day = .
0.2 gallon = x w
 1  1 
Volume of water mixed per gallon of acid = 0.2 gallons. Job = 3  + 4     (10)
 m w 
Hence, The correct option is (A).
11. Let the usual time taken be t hours.  1  1 
=  24  + 2     (2)
 m w 
 10   10  5
Distance = 6  t −  = 4  t +  t=
 60   60  6 30 40 48 4
+ = +
m w m w
Distance
Required speed = 36 18
t =
w m
 5 10 
6 −  2 1
 6 60  =
= = 4.8 kmph. w m
5
6
30 40  2  40 100
∴ Job = + = 30   + =
Hence, The correct option is (C). m w w  w w
12. Total distance (in km) = (30)(1) + (50)(5) + (75)(4) = 580 Part of the job which can be completed by 5 men and 10
Total time = 10 hours  1 1
580 women in a day = 5   + 10
Average speed (in kmph) = = 58.  m w
10
Hence, The correct option is (C).  2 1 20
= 5  + 10 =
w  w w
13. In an hour, the distance it can cover without stoppages = 54 km. 100
In this time, the distance it can cover with stoppages = 36 km. Required time (in days) = w = 5.
It does not cover 18 km because of its stoppages. 20
18 w
Stoppage time/hr = Time taken to cover 18 km = Hence, The correct option is (B).
54
hours = 20 minutes. 16. Let the initial income be 100x.

Hence, The correct option is (C). Rent = 25x
14. Let the time in which A, B and C can complete the job be a Food = 15x
days, b days and c days respectively.
Medical = 4.8x
1 1 1 Miscellaneous = 13.8x
+ = (1)
a b 40 Total expenses = 58.6x
1 1 1 Total savings = 41.4x = 82800
+ = (2)
b c 30 x = 2000.
1 1 1 Income = 200000.
+ = (3)
a c 20 Hence, The correct option is (A).
17. P(30) = S(10)
Subtracting (1) from (2),
3P = S
1 1 1 P 1
− = =
c a 120 S 3

Part B_Unit 1_Test.indd 148 27/04/2017 14:16:26


Test | 1.149

P 1 23. Logpq and logr q are given. We can invert the first expression
= and get logq p. Then, we get logr p. We can find x.
C 2
5 4
P% = 50. logpq = ⇒ logqp =
4 5
Hence, The correct option is (C).
5
18. 1036 – 12 = 1024 logrq =
6
1545 – 9 = 1536
 4  5 2
The H.C.F of 1024 and 1536 = 512. ∴logr p =     = = 3x
 5  6 3
Hence, The correct option is (B). 2
∴x=
19. Time interval for the bells to toll together is the LCM of 10, 9
15, 20, 25, 30 = 300 sec = 5 min.
Hence, The correct option is (D).
The next time the bells toll together is at 8:05 a.m.
Hence, The correct option is (C). 5 5 5
24. + +
1 + log p qr 1 + logq pr log r pq + 1
3 3 3 3
20. + + + ........
2+ 5 5+ 8 8 + 11 25 + 29 5 5 5
= + +
log p p + log p qr logq q + logq pr log r pq + log r r
3 5− 2 3 8− 3
= × + × 5 5 5
5+ 2 5− 2 8− 3 8− 5 = + +
log p pqr logq pqr log r pqr
3 11 − 8 3 29 − 26
+ × + ..... + ×
11 + 8 11 − 8 29 + 26 29 − 26  1 
⇒ 5(logpqrp + logpqrq + logpqrr) ∵logb a = 
3
=
( 5− 2 ) + 3( 8− 5 ) + 3( 11 − 8 )  loga b 
 3 3 3 ⇒ 5logpqr pqr ⇒ 5 × 1 = 5.

+..... +
3 ( 29 − 26 

) Hence, The correct option is (C).
3  25. Given, log102 = 0.3010
25 let P = 255
=  5 − 2 + 8 − 5 + 11 − 8 + ..... + 29 − 2 6 
  log10 p = log10255 = 55log2

= − 2 + 29 = 29 − 2 . = 55 (0·3010) = 16·555

Hence, The correct option is (A). ∴ The number of digits = 16 + 1 = 17.


Hence, The correct option is (A).
21. Given, a + b + c = 0
26. The given series is 45, 41, 37, ……..
If a + b + c = 0 then, a3 + b3 + c3 = 3abc → (1)
1 1
To get the maximum value of sum, the nth term is positive.
c8
(3 )
a8 a6 bc
⋅ (3 )
b 8 ab 6 c
⋅ abc6
3
tn > 0
45 + (n − 1) (−4) > 0
a2 b2 c8
⇒ 3bc . 3 ac . 3abc 6
∴ (am)n = amn −4n + 49 > 0
4n < 49
a2 b2 c2
+ +
= 3bc ac ab (∴am.an = am + n) 49
n< ⇒ n < 12.25
a3 + b 3 + c 3 3abc
4
=3 abc = 3 abc (∴ from (1)) n = 12
= 33 = 27. 12
S12 =
[2 + (11)4] = 276.
Hence, The correct option is (C). 2
22. Given, 3x + 3 – 3x+2 = 486 Hence, The correct option is (C).
⇒ 3x · 33 – 3x · 32 = 486 27. Let 1 + 3 + 32 + 33 + ……to n terms < 3000
⇒ 27 (3x) – 9(3x) = 486 3n − 1
=1· < 3000
⇒ 18 (3x) = 486 3 −1
486 = 3n − 1 < 6000
⇒ 3x = = 27
18 3n ≤ 37 < 6001
⇒ 3 = 3 ⇒ x = 3.
x 3
∴ The greatest value of n is 7.
Hence, The correct option is (A). Hence, The correct option is (C).

Part B_Unit 1_Test.indd 149 27/04/2017 14:16:28


1.150 | General Aptitude

28. S = 3 + 33 + 333 + …….. + upto n terms 4ac − b 2


The minimum value =
∴ 3S = 9 + 99 + 999 + ….. + upto n terms 4a
3S = (10 − 1) + (100 − 1) + (1000 − 1) +….(10n −1)
4 (1)(9) − ( −15)
2

=
= 10 + 100 + ……..+ 10 − (1 + 1 +…….upto n times)
n
4
36 − 225 −189
10(10 n − 1) = =
∴ 3S = −n 4 4
10 − 1
Hence, The correct option is (D).
10(10 n − 1) n
⇒S= − 30. Area of the shaded region = Area of the quadrant – Area of
27 3
the triangle =  p (64) − (8)(6) cm2 = (16p – 24) cm2.
1 1
Hence, The correct option is (D). 4 
2
29. Given, f(x) = x2 – 15x + 9 = 0. Hence, The correct option is (D).
Here, a = 1 > 0
∴ f(x) has a minimum value.

Part B_Unit 1_Test.indd 150 27/04/2017 14:16:29

You might also like